Sie sind auf Seite 1von 394

Return to question 1 Return to contents

200 Best of Five Questions


for Nephrology Specialty Certificate Exam with Revision Notes and Guidelines
Dr Ebadur Rahman
Dr Raees Farhan Mushtaq

Copyright Ebadur Rahman, Raees Farhan

All rights reserved. No part of this publication may be reproduced, stored in


any retrieval system or transmitted in any form or by any means, electronic,
mechanical, photocopying, recording or otherwise, without the prior written
permission of the copyright holder for which application should be addressed
in the first instance to the publishers. The views expressed herein are those
of the author and do not necessarily reflect the opinion or policy of Tricorn
Books or the employing organisation, unless specifically stated. No liability
shall be attached to the author, the copyright holder or the publishers for
loss or damage of any nature suffered as a result of the reliance on the
reproduction of any of the contents of this publication or any errors or
omissions in the contents.

Published in UK, 2015 by Tricorn Books

Tricorn Books,
131 High Street,
Old Portsmouth,
PO1 2HW

www.tricornbooks.co.uk
200 Best of Five Questions
for Nephrology Specialty Certificate Exam
with Revision Notes and Guidelines

Authors
Dr Ebadur Rahman
FRCP, FRCPl, FASN, MMedSci Nephrology (Sheffield,UK)
Renal SCE (UK)
Consultant Nephrologist
Prince Sultan Military Medical City, Riyadh
Regional Adviser RCPE, Riyadh

Dr Raees Farhan Mushtaq


FASN, MRCPl, Renal SCE (UK)
MSc Renal Medicine (Imperial College, University of London),
Diploma in Nephrology (Hammersmith Hospital, London)
Adult Nephrologist, Salford Royal Hospital, Manchester

Co Authors
Dr Ramesh Abdullah
Dr Roshni Rathore

Editors
Dr Puchimada Uthappa, FRCP
Nephrology Consultant, Salford Royal Hospital, Manchester
Dr Fancesco Rainone, MD
Consultant Renal Physician, Salford Royal Hospital
Contents

Foreword
Authors
Preface
Normal lab ranges
Questions & Answers
Radiology for SCE Exam

1, 2, 3, 4, 5, 6, 7, 8, 9, 10, 11, 12, 13, 14, 15, 16, 17, 18, 19, 20,

21, 22, 23, 24, 25, 26, 27, 28, 29, 30, 31, 32, 33, 34, 35, 36, 37,

38, 39, 40, 41, 42, 43, 44, 45, 46, 47, 48, 49, 50, 51, 52, 53, 54,

55, 56, 57, 58, 59, 60, 61, 62, 63, 64, 65, 66, 67, 68, 69, 70, 71,

72, 73, 74, 75, 76, 77, 78, 79, 80, 81, 82, 83, 84, 85, 86, 87, 88,

89, 90, 91, 92, 93, 94, 95, 96, 97, 98, 99, 100, 101, 102, 103,

104, 105, 106, 107, 108, 109, 110, 111, 112, 113, 114, 115,

116, 117, 118, 119, 120, 121, 122, 123, 124, 125, 126, 127,

128, 129, 130, 131, 132, 133, 134, 135, 136, 137, 138, 139,

140, 141, 142, 143, 144, 145, 146, 147, 148, 149, 150, 151,

152, 153, 154, 155, 156, 157, 158, 159, 160, 161, 162, 163,

164, 165, 166, 167, 168, 169, 170, 171, 172, 173, 174, 175,

176, 177, 178, 179, 180, 181, 182, 183, 184, 185, 186, 187,

188, 189, 190, 191, 192, 193, 194, 195, 196, 197, 198, 199, 200
Foreword

The Royal College of Physicians Nephrology Specialty Certificate Exam was


introduced in 2010 and provides a further hurdle for nephrology trainees
in their career path towards eligibility for a consultant post. For those
nephrologists working outside the UK, success with this exam can facilitate
acquisition of a more favourable career path. The exam is far from inexpensive
and it can be only attempted once in a calendar year, and so it follows that
candidates should try to maximise their chances of success. However, at the
current time there are only limited resources to help candidates prepare for
the exam.
This is where Dr Rahman and Dr Mushtaqs 200 Best of 5 questions for the
Nephrology Specialty Certificate Exam fills the gap. The questions provided in
this book have been carefully selected to represent the topics covered in the
postgraduate nephrology syllabus and are typical in structure and degree of
difficulty to the questions seen in the exam. What I find most impressive is the
style of the up to date, evidence based and comprehensive explanations that
underpin each topic, the regular use of images and tables making this resource
not only a reliable, but also enjoyable, source of reference and learning.
The authors should be congratulated for their efforts, and I do hope that the
reader benefits with success in the forthcoming specialty exam.

Philip A Kalra, MA (Cantab) MB BChir FRCP (UK) MD (Cantab)


Honorary Professor of Nephrology and Consultant Nephrologist
University of Manchester and Salford Royal NHS Foundation Trust, UK
(Editor of Essential Revision Notes for the MRCP, published by Pastest Ltd)

Return to contents
About the Authors
Dr Ebadur Rahman

FRCP, FRCPl, FASN, MMedSci Nephrology (Sheffield,UK)


Renal SCE (UK)
Consultant Nephrologist
Prince Sultan Military Medical City, Riyadh
Regional Adviser RCPE, Riyadh

Dr Ebadur Rahman graduated from the University of Dhaka in


1995 and went on to study for the Diploma in Internal Medicine (UK) in 1998
at the Imperial College of Science, Technology and Medicine, London. He
continued his academic career with a Diploma and Masters in Nephrology
from Sheffield University in 1999. He attained MRCP (UK) in 2003 and Specialty
Certificate in Nephrology RCP (UK) in 2011. He is a fellow of Royal college of
Physicians Edinburgh ,Ireland and American Association of Nephrology.
Dr Rahman is appointed as a Consultant Nephrologist at Prince Sultan Military
Medical City since 2008. He is Clinical Tutor in the Department of Nephrology,
(PSMMC) since 2009. He held the position of chairman of Riyadh Nephrology
and Transplant club (RNTC) in the year 2010. Recently he was appointed as the
Regional adviser of Royal college of Physicians Edinburgh for Riyadh region
KSA. Dr Rahmans area of interest is in Intervention Nephrology. He has done
more than 3000 successful Renal Interventions. He has more than 20 published
works as articles or abstracts.

Dr Raees Farhan Mushtaq

FASN, MRCPl, Renal SCE (UK)


MSc Renal Medicine (Imperial College, University of London),
Diploma in Nephrology (Hammersmith Hospital, London)
Adult Nephrologist, Salford Royal Hospital, Manchester

Education: Dr Farhan Mushtaq has an active interest in medical education and


is a member of the Royal College of Physicians. He has 8 years of experience in
teaching for the MRCP and undergraduate exams.

Nephrology
Dr Farhan Mushtaq graduated from Liaquat University Jamshoro, Hyderabad
Pakistan in 1992 securing 6th position on merit. He then went to the UK for
his postgraduate studies. With the blessing of his parents, he embarked on
his postgraduate training in Nephrology at Hammersmith Hospital, London
under the supervision of Dr Jeremy Levy (author of the best selling book, The
Oxford Handbook of Dialysis), and Prof Charles D Pusey. Dr Farhan gained
his Postgraduate Diploma and Masters Degree (MSc in Renal Medicine) from
Imperial College of Science, Technology and Medicine (University of London)
and his MRCP from the Royal College of Physicians of Ireland. He passed Renal

Return to contents
Specialty Certificate Exam from RCP (UK) in 2012.
Dr Mushtaq has full registration with the General Medical Council in the UK and
a license to practice. From 2009 to 2013 he worked as an adult Nephrologist
in Prince Sultan's Military Medical City, Riyadh, one of the top ranked tertiary
care hospitals in the kingdom. His major achievement was the setting up
of a Nephrology Clinic Service in the Primary Health Center, which was the
first of its kind in Saudi Arabia. With a profound interest in teaching, he and
his colleague, Dr Ebad made the Renal Specialty Exam Course website,
which consists of 750 Best of Five Questions, currently the largest bank of
Nephrology questions available on the Internet.
Since 2012 Dr Mushtaq and Dr Ebad have been running a very successful
interactive course in Dubai, preparing candidates for the difficult Renal
Specialty Exam.
Currently he is working as an adult Nephrologist at Salford Royal Hospital,
Manchester, the top ranked Tertiary Care NHS Trust in UK.
Dr Mushtaq's keen area of interest is in slowing down the progression of
Diabetic Kidney Disease, CKD disease and Haemodialysis.

Co Authors
Dr. Ramesh Nair (Abdullah)
Diplomate American Board of Pathology
Consultant Renal Histopathologist
Prince Sultan Military Medical City, Riyadh Saudi Arabia

Dr. Nair Graduated from the Kozhikode Medical College in India in 1989. He
received his Diplomate in the American Board of Pathology in 1999 and Renal
Pathology Fellowship Certification from Johns Hopkins University Hospital,
Baltimore, USA in 2000. He has more than a decade of experience as a Renal
Pathologist in the USA and has worked in prestigious institutions such as the
University of Iowa Hospitals where he served as Clinical Associate Professor. He
has received several teaching accolades and including best 2nd year Medical
Student Teacher of the Year award. He is currently a Consultant in Renal
Pathology at the Prince Sultan Military Medical City in Riyadh, Saudi Arabia.

Dr Roshni Rathore
MBBS, MRCP Nephrology

Dr Roshni Rathore graduated from Mumbai, India in 2004. She has


trained in Renal medicine and General medicine as a Specialist
Registrar in tertiary renal centres in North west of England namely
Royal Preston Hospital, Wythenshawe Hospital, Manchester Royal
infirmary and Salford Royal Hospital. Currently She is appointed as
Locum Consultant in Salford royal NHS foundation .

Return to contents
Editors

Dr P M Uthappa, FRCP
Consultant Renal Physician
Dr P M Uthappa completed his undergraduate medical training
in Bellary, India. He attained his membership from the Royal
college of Physicians, Edinburgh after general medical training at
Victoria Hospital Kirkcaldy, Scotland. He entered specialist training
in Nephrology and General Internal Medicine in West Midlands
with training posts at University Hospital Birmingham, University
Hospital Coventry and Warwickshire NHS Trust and Royal Wolverhampton
Hospital NHS Trust. After obtaining his CCT in renal medicine and General
Medicine in 2008, he was awarded Fellowship of the Royal College of Physicians,
London in 2012.
At present he is working as a Renal Physician at Salford Royal NHS Foundation
Trust and has been in the post since 2008. His special interests include, CKD,
Hypertension, Haemodialysis, access for dialysis, pregnancy and renal disease.
He has significant experience in Medical Education, training of the renal trainees
and is the lead for undergraduate Medical education in the renal department.

Dr Francesco Rainone, MD
Consultant Renal Physician
Salford Royal & WWL NHS Foundation Trusts
Dr Francesco Rainone was born and raised in Italy where he
graduated magna cum laude in 2007 at Vita-Salute San Raffaele
University, Milan. He subsequently entered the Nephrology
training programme at Vita-Salute University, Milan and obtained
the specialization title cum laude in March 2013. He spent 10
months of his training at Salford Royal NHS Foundation Trust where he stayed
as visiting research fellow. He eventually moved to the UK and is a Consultant
Nephrologist at Salford Royal NHS Foundation Trust. His clinical areas of interest
are transplantation, early CKD, glomerulonephritis and myeloma-associated
kidney damage.

Contributors
Dr Kishore Gopalakrishnan
Pathologist at NHS
Birmingham, United Kingdom

Return to contents
Preface

Introduced in 2010, the Nephrology SCE (Specialty Certificate Examination)


is compulsory for the Nephrology trainees to gain a CCT (Certificate of
Completion of Training) in the Specialty. Candidates who have sat for the exam
with their written MRCP are familiar with the best of five formats where they
select the best of five plausible and comparable answers. This requires both
knowledge and exam technique that is often best achieved through practice
questions.

Currently, difficulty of exam preparation is the unavailability of reliable practice


questions. This book seeks to fill that void by providing the best of five
questions that resemble the exam format with their explanatory answers. The
question material was written with the 2010 syllabus in mind and based on
contemporary guidelines from KDIGO, KDOQI, ISPD and Renal Association.

Our goal was to create a valuable tool for all trainees revising for the SCE exam,
American Board of Nephrology and Arab Board of Nephrology Exam.

Our authors were supported by consultants in the region and we offer our
thanks to them all.

We would like to extend our gratitude to our parents and families. Without
their support, blessings and encouragement this would not be possible.

We acknowledge the graciousness of our patients who consented the


publishing of their pictures for education purposes. We would also like to credit
Dr. Ramesh Abdullah for providing renal histology slides from his personal
collection.

Finally, much appreciation is given to Professor Philip Kalra for providing his
inspirational commentary for this book.

Ebad & Farhan

Return to contents
Normal Lab ranges

Serum Sodium 133-146 mmol/L


Serum Potassium 3.5-5.3 mmol/L
Serum Urea 2.5-7.8 mmol/L
Serum Creatinine 62-115 umol/L
Estimated Glomerular Filtration rate >90 ml/min/1.73m2
Corrected Calcium 2.20-2.60 mmol/L
Serum Phosphate 0.80-1.50 mmol/L
ALT 7-40 U/L
Alkaline phosphatase 30-130 U/L
Albumin 35-50g/L
Proteins 60-80g/L
Globulins 20-35 g/L
Total Bilirubin 0-20 umol/L
CRP <10 mg/L
pH 7.35-7.45
PO2 12-15KPa
Pco2 4.5-6.0 KPa
Plasma Lactate 0.50-2.20 umol/L
Random capillary blood glucouse 3.0-6.0 mmol/L
Haemoglobin 115-165g/L
RBC 3.80-5.80
HCT 0.37-0.47L/L
MCV 84-105 fL
MCH 27-32 Pg
MCHC 305-350g/L
RDW 11.5-14.5
WBC 4.0-11.0
Neutrophil 1.8-7.5
Lymphocyte 1.0-4
Eosinophil 0.0-0.4
Basophil 0.0-1.0
Platelet 150-450
C3 0.83-1.46g/L
C4 0.15-0.52g/L
MP250 0.0-3.5 IU/mL
PR3 250 0.0-2.0 IU/mL
Free kappa chains 3.3-19.4 mg/L
Free Lambda chains 5.7-26.3 mg/L
Kappa/Lambda ratio 0.3-1.7

Return to contents
Question 1 Q
A 17-year-old boy was referred for investigation of renal impairment and
hypertension. These had been identified during an assessment for possible
diabetes mellitus. He gave a 13-month history of increasing polyuria and
polydipsia. He was taking no regular medications. Slit lamp examination
findings are shown in the picture (1). His blood pressure was 154/96 mmHg.

Investigations:
serum potassium 3.1 mmol/L (3.54.9)
serum bicarbonate 17 mmol/L (2028)
serum creatinine 109 mol/L (60110)
eGFR 80 ml/min
serum phosphate 0.61 mmol/L (0.81.4)
serum chloride 113 mmol/L (95107)
urinalysis trace blood, 2+ protein
1+ glucose
urinary protein: creatinine ratio 57 mg/mmol (<15)

What would be the most appropriate treatment?

A Cysteamine
B Elimination of dietary lactose
C Penicillamine
D Prednisolone
E Pyridoxine

Return to contents
A nswer A - Cysteamine

Options B, C, D and E are incorrect, option A is the correct answer.


Picture shows characteristic corneal crystal of cystinosis.
Cystinosis is a rare autosomal recessive disease, which means that both parents
are carriers of the abnormal gene that leads to this condition. In such couples,
the odds are one in four that their children will have cystinosis.
Autosomal recessive defect in cystinosin in lysosomes leading to lysosomal
cystine accumulation. The transporter for cystine is defective in children with
cystinosis, which causes the cystine to crystallize within tissues. This build
up eventually destroys all the bodys organs including the kidneys, liver,
muscles, white blood cells, eyes and central nervous system.
Most cases present in childhood.
Without specific treatment, these children progress to end-stage renal failure
by an average age of nine years. In the past, this meant death. Today these
patients can receive renal dialysis or transplantation.

Clinical features include multi system involvement


Eyes (corneal crystals)
Renal (Fanconi syndrome of proximal convoluted tubule dysfunction)
Endocrine (hypothyroidism, hypogonadism, Impaired glucose tolerance
& diabetes)
Neuro (dysphagia, myopathy, psychomteric defects)
GI (Hepatomegaly, splenomegaly)

Diagnosis:
The diagnosis of cystinosis is commonly made by examining the eyes for the
presence of corneal crystals, and confirmed by measuring the level of cystine in
blood cells. Individuals with cystinosis exhibit cystine levels up to 100 times the
level found in persons without this disorder.

Treatment:
Symptomatic- Rehydration, replacement of bicarbonate losses with sodium
citrate or bicarbonate
Topical cysteamine eyedrops results in dissolution of corneal crystals.
Drug- Cysteamine (It forms a complex with cystine that can leave lysosomes,
ameliorating disease)
Dialysis or Kidney Transplantation in those who developed ESRD

Pencillamine is used to treat cystinuria, it has no role in cystinosis.

Return to contents
Question 2 Q
A 69-year-old man with stage 4 CKD, diabetes mellitus, hypertension, and
stable coronary artery disease is seen in your clinic for routine follow-up. He
complains of occasional morning nausea, decreased appetite, and nocturnal
leg cramps. His weight is stable. He denies chest pain, shortness of breath,
and fatigue.

Current Medications are aspirin, atorvastatin, lisinopril, and metoprolol.

On physical examination, he appears well. His blood pressure is 135/70 mmHg.


A left radiocephalic fistula is well developed with a palpable thrill.
The remainder of the examination is normal.

Laboratory studies show a white cell count of 7500/l, haemoglobin level of 9.6
g/dl (9.2 g/dl 3 months ago), platelet count of 300,000/l, transferrin saturation
(TSAT) of 34%, and ferritin level of 602 ng/ml. eGFR using the Modification of
Diet in Renal Disease formula is 23 ml/min per 1.73 m2. Serum vitamin B12
and folate levels are normal. Reticulocyte count is 40,000/l (reference range,
23,00090,000/l).

Which one of the following is the most appropriate next step in the
management of this patients anaemia?

A Continue current regimen
B Initiate an erythropoiesis-stimulating agent (ESA)
C Begin intravenous iron
D Measure serum erythropoietin level
E Begin oral iron

Return to question 1 Return to contents


A nswer A - Continue current regimen

Options B, C, D and E are incorrect, option A is the correct answer. The most
appropriate management of this patient is to continue the current regimen with
continued regular monitoring of his clinical status and anaemia. This patient
has stage 4 CKD and associated anaemia. The 2012 Kidney Disease Improving
Global Outcomes (KDIGO) Practice Guideline for Anaemia in Chronic Kidney
Disease recommends that the decision to treat anaemia in such patients
should be an individual one balancing the risk of stroke, hypertension, and
vascular access loss against the benefit of lowering the requirement for blood
transfusions and improving anaemia-related symptoms. He has no symptoms
attributable to his anaemia, and his haemoglobin level has been stable.
There is therefore no indication to treat this patients anaemia.
The iron indices indicate adequate iron stores. The KDIGO guidelines for
anaemia in CKD recommend a trial of intravenous iron or a 1- to 3-month trial
of oral iron if an increase in haemoglobin concentration without starting ESA
treatment is desired and the TSAT is <30% and ferritin is <500 ng/ml. This
patient does not have a compelling indication for treatment of anaemia,
and the iron studies do not indicate absolute or functional iron deficiency.
Therefore, iron therapy is not likely to be of benefit.

Furthermore, intravenous iron should not be used unless they patient cannot
tolerate or did not benefit from oral iron.
If he had an indication to start an ESA, the goal haemoglobin would be
1012 g/dl. (NICE & KDOQI)
KDIGO guidelines recommend that the evaluation of anaemia in CKD patients
include a complete blood count, absolute reticulocyte count, serum ferritin
level, serum TSAT, serum vitamin B12, and folate levels. Routine measurement
of the serum erythropoietin level is not recommended.

Kidney Disease Global Outcomes Anemia Work Group. KDIGO Clinical Practice Guideline for
anaemia in chronic kidney disease. Kidney Int Suppl 2, 279-335, 2012

Return to contents
A
Learning point: if a CKD Patient is anaemic in CKD stage 1, 2 & 3 A then
unlikely to be due solely EPO deficiency.

Exclude causes other than EPO deficiency


1 Check ferritin, vit B12, and folate level
2 CRP to rule out infection
3 Reticulocytre count to rule out bleeding
4 Haemoglobinopathy screen, Myeloma screen
5 Rule out primary bone marrow disorders

Key KDIGO Recommendations Regarding Target Haemoglobin Level in


Patients Receiving Erythropoiesis-Stimulating Agent Therapy

1. For adult CKD nondialysis patients with Hb less than 10 g/dl, it is


suggested that the decision to initiate ESA therapy is individualized
based on the rate of fall of Hb, previous response to iron therapy, risk
of needing transfusion, risks related to ESA therapy, and presence of
symptoms attributable to anaemia.
2. For adult CKD patients on dialysis, it is suggested that ESA therapy
is used to avoid having the Hb concentration fall below 9 g/dl by
starting ESA therapy when the Hb is 9 to 10 g/dl. Individualization of
therapy is reasonable as some patients may have improvements in QoL
at higher Hb concentration, and ESA therapy may be started above 10
g/dl.
3.In general, it is suggested that ESAs are not used to maintain Hb
concentration above 11.5 g/dl in adult patients with CKD.
4.Individualization of therapy will be necessary as some patients
experience improvements in QoL at Hb concentrations above
11.5 g/dl and will be prepared to accept the risks.
5.In all adult patients it is recommended that ESAs not be used
intentionally to increase the Hb above 13 g/dl.

CKD, Chronic kidney disease; ESA, erythropoiesis-stimulating agent; KDIGO, Kidney Disease:
Improving Global Outcomes; QoL, quality of life.

Return to question 1 Return to contents


A nswer

KDOQI :Target Iron indices

Serum ferritin >100ug/L(non haemodialysis)


or >200 ug/L(Haemodialysis) Hypochromic RBC<10%, TSAT>20%
Generally aim 200-500 ug/L, Avoid >800 ug/L
Measure iron stores 1-3 monthly depending on response to
treatment and no earlier than 1 week after IV iron administration.

Dos and donts in Anaemia management

Do use IV iron first whenever possible, esp. if ferritin <100 ug/L


Do start ESA therapy when Hb 9-10 rather than 10-11
Do aim for target Hb 10-12 g/dL
Do not escalate ESA dose in patients responding poorly to treatment.
Do not administer IV iron to patients with active infection
Do not aim Hb>13g/dl

Causes of Poor Response to ESA

Most common
Iron deficiency, infection, under dialysis
Less common
Poor compliance
Blood Loss
Hyperparathyroidism, Vit B12, folate deficiency
Bone marrow disorders, Sickle cell disease, ACEi, ARB,
AntiEPO antibodies causing PRCA (Pure red cell aplasia)

References:
Kidney Disease Global Outcomes Anaemia Work Group. KDIGO Clinical Practice
Guideline for anaemia in chronic kidney disease. Kidney Int Suppl 2, 279-335, 2012

Return to contents
Question 3 Q
Look at the image.

Which one of the following is the most likely complication seen in the above
image during insertion of right tunnelled line for dialysis in this patient?

A Clot in catheter
B Kink in catheter
C Malposition of Catheter
D Pneumothorax
E Perforation of Heart

Return to question 1 Return to contents


A nswer B - Kink in catheter

Options A, C, D and E are incorrect, option B is the correct answer.


Image shows an acute kink in the tunnelled portion of the catheter resulting in
catheter dysfunction.
Tunnelled dialysis catheter is a frequently used vascular access to initiate
haemodialysis treatment for patients with end stage renal disease.
Right internal jugular vein is the preferred site due to its straight continuation
in to the superior vena cava. Improper placement technique can lead to early
dysfunction and inadequate dialysis treatment.
The ideal treatment option, when possible, is to create a new tunnel utilizing
the same venous puncture thus preserving the venous access site.

Complications

Catheters are the major source of dialysis access infections.


Average HD patient has 1-2 times bacteraemia per yr.
Annual catheter infection rate should be <5%.
10% catheter infections are polymicrobial, 70% are with Gram positive
organisms.
Fever in an HD patient with a line = line sepsis until proven otherwise.
Following are metastatic serious complications
Clue - (Persistent Fever or High CRP)
Endocarditis
Osteomyelitis
Epidural abscess
Discitis
Septic shock
Septic arthritis

Return to contents
A
Septic thrombophlebitis
Death
UK Renal association guidelines for Tunnelled HD CVC
(central venous catheters)
Tunnelled cuff catheters are preferred for temporary HD needed for >3
weeks
Preferred site is internal jugular vein
Chest x-ray should be done to check position of catheter and R/O
Pneumothorax prior to use
<20% patients on long term HD should be on tunnelled CVC
All units should use US guidance for insertion, and full barrier precautions
during insertion.
Peripheral and central cultures should be taken before starting antibiotics
Audit data should be collected for bacteraemia rates.
Rules for Treatment of catheter related bacteraemia

Principles Practice
Always document the infection Routine survelliance cultures post therapy
is cured 1 week.
Fever or rigors highly predictive Start antibiotics without waiting for culture
of bacteraemia results
Gram -ve are common up to 40% Empiric coverage includes GPC and GNR
Fungal catheter infection Daily blood cultures and treat for at least 2
weeks from the first -ve culture
Staphlococcus aureus organism Empiric antibiotics should include
often methicillin resistant vancomycin
GPC-gram+ve cocci, GNR-gram-ve rods

Indications for Catheter Removal

1. No response to sensitive antibiotic after 12-24 hrs


2. Tunnel infection
3. Pus at exit site
4. Septic shock
5. Metastatic complications
6. Fungal infection
7. All cases if Pseudomonas or staph aureus infection

Return to question 1 Return to contents


A nswer

Table showing percentage of organisms responsible for haemodialysis central


venous catheters blood stream infections

Organism Percentage reported


Gram Positive Cocci 52-85%
Staphylococcus aureus 22-60%
Staphylococcus epidrmidis 9-13%
Methicillin resistant staphylococcus aureus 6-29%
Enterococcus faecalis 2-18%
Gram negative bacilli 20-28%
Pseudomonas aeruginosa 2-15%
Enterobacter cloacae 9%
Escherichia coli 10%
Klebsiella pneumoniae 6%
Polymicrobial 16-20%
Acid fast organism Rare
Fungi Rarely reported

Algorithm of Treatment of dialysis catheter related Bacteremia

Adapt dose according to Length of Treatment


residual renal function 1. 7 - 10 days for tunnel
Dialysis infections without
High flux bacterecmia if catheter
Haemodiafiltration Empiric treatment removed
Daily dialysis should be started by 2. 3 weeks uncomplicated
Prolonged dialysi the recorded infections 3. 6 weeks if
in the unit bactercmia/fungemia persist
(Monitor Predialysis trough after 72 hours
levels) 4. or infective endocarditis
Preference for Abx 5. 8 weeks if osteomyelitis
which should be
administered with
dialysis

Algorithm of Vancomycin
Cefazohn
Treatment of dialysis If unit with high
Ceflazidlme
catheter related prevalence of MRSA
Use vancomycin for
Daptomycin
Bacteraemia empiric treatment
Teicoplanin

If Methecillin sensitive staph


If MIC vancomycin>2ug/L
aureus, use cefazolin
Or Allergy to vancomycin, then
use Daptomydn (if available)

Return to contents
A
Start antibiotic empirically if fever>38 C, rigors or low BP
Cover for both Gram positive and Gram negative organisms
For Gram positive cover use either vancomycin (10-20 mg/kg iv- usually 1
gm) check trough level at 20 hr and redose if<15 mg/L) or teicoplanin
For Gram negative cover use gentamycin (2mg/kg) iv check trough level at
20 hr and redoes if <2mg/L.
Vancomycin is partially removed by HD, Gentamycin completely

Duration of antibiotics for Catheter related infection Treatment


Organism Duration
Staphlococcus infection 4 weeks
Metastatic infection 6-8 weeks
Gram negative ve infection or Candida 2 weeks

Antibiotic Dosage Comment


Empiric combination
treatment
vancomycin 20 to 25 mg/kg Loading dose
cephazolin 2g For a 2-d interval
Maintenance treatment
vancomycin 500 mg For MRSA, trough target of 15 to
20 g/ml
cephazolin 2 g for 2-d interval, For MSSA
3 g for 3-d interval
Alternative agents
(limited data available)
daptomycin 6 mg/kg per 48 h Do not use in MRSA pneumonia
linezolid 600 mg twice a day Cumulative toxicity after 2 to 3
wk of use

Treatment of Staphylococcus aureus bacteraemia in haemodialysis


(a MRSA, methicillin-resistant S. aureus; MSSA, methicillin-sensitive S. aureus).

Return to question 1 Return to contents


A nswer

S. aureus typically appears as Gram stainpositive cocci in grape-like clusters


The most important characteristic of S. aureus is its marked capacity for swift
tissue invasion, multiplication at the nidus of infection, and subsequent rapid
dissemination throughout the body.

Dialysis patients are particularly vulnerable to infections caused by S. aureus.


S. aureus accounts for >8% of the mortality in the dialysis population and is the
leading cause of mainly Vascular access-siterelated infections.

Daptomycin is approved for the treatment of complicated skin and skin-


structure infections with MRSA in a dosage of 4 mg/kg per d and for MRSA
bacteraemia, with or without right-side endocarditis, in a dosage of 6 mg/kg
per d. In stage 4 or 5 renal failure, a similar dosage every 48hr's
is recommended.

Linezolid is a bacteriostatic oxazolidin that has high oral bioavailability and


inhibits bacterial protein synthesis by binding to the 50S ribosomal subunit
in Gram-positive bacteria and mycobacterial species. The use of linezolid
for a period beyond 2 to 3 wk is associated a dosage- and time-dependent
myelosuppression and risk for lactic acidosis that is caused by a depletion of
several mitochondrial proteins.

Return to contents
Question 4 Q
A 44-year-old male attended accident and emergency with a 2-week history
of skin rash (See image). He had a past medical history of known case of ESRD
secondary to ADPKD received a cadaveric kidney transplant 4 years back.
He was on triple immunosuppression (tacrolimus, Mycophenolate mofetil
and Prednisolone).

eGFR 81umol/L

Look at the image.

Which one of the following is the most likely diagnosis?

A Molluscum contagiosum
B Chicken pox
C Herpes zoster
D Kaposi Sarcoma
E Folliculitis

Return to question 1 Return to contents


A nswer A - Molluscum contagiosum

Options B, C, D and E are incorrect, option A is the correct answer. Patient with
kidney transplant are on immunosuppressive drugs have high risk of infections
as compared to others. Below is the time table of most common infections post
kidney transplant.
Molluscum contagiosum is a pox virus that causes a chronic localised infection
consisting of flesh colour dome shape papules with indentation or umbilication
on the skin of an infected individual. Its incubation period is 2 to 6 weeks. In
immunocompromised patients the lesions can be large (aka giant Molluscum).
In immunocompetent patients it is self-limiting but might not be so in
immunocompromised patients.

Time frame of Post Kidney Transplant infections

Months after Kidney Transplantation

Return to contents
Question 5 Q
In renal transplantation the risk of recurrence in the allograft is lowest for which
of the following renal diseases?

A Oxalosis
B FSGS (Focal Segmental Glomerulosclerosis)
C IgA nephropathy
D Lupus nephritis
E MPGN Type 1 (Membranoproliferative Glomerulonephritis Type 1)

Return to question 1 Return to contents


A nswer D - Lupus nephritis

Options A, B, C and E are incorrect, option D is the correct answer.

Recurrent of Secondary Glomerulonephritis disease post transplantation with


consequent graft loss

Recurrence of Primary Glomerulonephritis with consequent graft loss

ANCA, antineutrophil cytoplasmic antibody; EMC, essential mixed cryoglobulinemia; GN,


glomerulonephritis; HSP, Henoch-Schnlein purpura; HUS, (nondiarrheal associated) hemolytic
uremic syndrome; LCDD, light chain deposition disease; SLE, systemic lupus erythematosus.

Return to contents
Question 6 Q
You are in a Chronic Kidney Disease clinic (pre-dialysis). One of your CKD stage
5 patient is approaching the need for renal replacement. He is contemplating
continuous ambulatory peritoneal dialysis(CAPD) and asks you about medical
contraindications to this modality.

Which of the following is not a relative contraindication to CAPD treatment?

A Diabetes mellitus
B Bilateral inguinal hernia
C Severe chronic obstructive pulmonary disease
D Previous perforated diverticular disease and sigmoid colectomy
E Severe Rheumatoid arthritis

Return to question 1 Return to contents


A nswer A - Diabetes mellitus

Options B, C, D and E are incorrect, option A is the correct answer.


Previous pelvic surgery reduces the likelihood of successful peritoneal dialysis,
adhesions often making catheter placement difficult and reducing the
peritoneal surface available for dialysis.
Inguinal hernia rapidly fills with peritoneal dialysate causing patient discomfort
and inefficient dialysis.
Peritoneal dialysis is a good treatment for patients with diabetes mellitus
although the glucose load in the dialysate may necessitate a change in insulin
dosage.
Splinting of the diaphragm by a large volume of intraperitoneally fluid often
exacerbates chronic obstructive pulmonary disease (COPD), and this should be
considered a relative, but not absolute contraindication.
Effective peritoneal dialysis requires a degree of manual dexterity and would
not be easy for a patient with severe Rheumatoid arthritis but they can do with
help of carers.
Adapted from KDOQI guidelines.
Peritoneal Dialysis Contraindications
Absolute Relative
1. Known peritoneal sclerosis loss of Weight (>100kg)-(inadequate
membrane function Clearance)
2. Mentally or physically incapable Anuria
(but not assisted PD) Intolerance of fluid in abdomen (e.g.
3. Large irreparable hernia or other back pain, respiratory disease)
abdominal wall defects Malnutrition
4. Ileostomy,colostomy, ileal conduit Recurrent diverticulitis/IBD
5. Active diverticular disease Peritoneal leaks
6. Very poor housing Recent-intra-abdominal prosthesis
7. Poor personal hygiene Severe gastro paresis (worsening
Vomiting)
Frailty/Dementia

Haemodialysis Contraindications
Absolute contraindications Relative contraindications
Thrombosed central veins Severe vascular disease
No Vascular access Severe hemodynamic instability
Hypotensive heart failure Long distance from HD unit

Return to contents
Question 7 Q
An 18-year-old woman was referred for investigation of progressive generalised
muscle weakness and lethargy. She was taking no regular medications.
Examination was unremarkable. Her blood pressure was 108/60 mmHg.
Her body mass index was 20.

Investigations:
serum potassium 2.8mmol/L (3.54.9)
serum bicarbonate 33 mmol/L (2028)
serum creatinine 77 mol/L (60110)
eGFR >90 ml/min
serum magnesium 0.61 mmol/L (0.751.05)
serum chloride 85mmol/L (95107)

urinary chloride 61
urinary sodium 57
24-hour urinary calcium 1.6 mmol (2.57.5)

What is the most likely diagnosis?

A Bartters syndrome
B Gitelmans syndrome
C Hypokalaemic periodic paralysis
D Liddles syndrome
E Gordans syndrome

Return to question 1 Return to contents


A nswer B - Gitelmans syndrome

Options A, C, D and E are incorrect, option B is the correct answer.


This patient has Hypokalaemic metabolic alkalosis with normal BP
Low Potassium + High Bicarbonate and Normal Blood Pressure
Consider

Bartters syndrome: mode of presentation & severity varies; autosomal


recessive disorder. At least 6 subtypes now detected. All shared impaired
Na reabsorption in the thick ascending limb of loop of henle giving a clinical
picture of salt wasting and hypokalaemic alkalosis.
On Biopsy Hyperplasia and hypertophy of the juxtaglomerular apparatus is
seen. Its often present in childhood with polyuria (nocturia), Failure to thrive,
muscle weakness; can be associated with nephrocalcinosis; can cause ESRD.

Treatment
Oral potassium supplementation,
Spironolactone and NSAID
Adjuvant use of ACEi has also been successful.

Gitelmans: Autosomal recessive disorder its gene encoding the thiazide


sensitive NACL co transporter in Distal convolulated tubule. Patient may
complain of fatigue, muscle cramps, tetany. Associated with chondrocalcinosis.
Distinguished from Bartters by hypomagnesaemia and hypocalciuria.

Treatment
Oral potassium supplementation, Amiloride 5-40 mg od.
NSAID are not helpful.

Liddle syndrome
Very rare
Mutation in ENaC (epithelial sodium channel expressed on the apical surface of
collecting duct cells).

Autosomal dominant inheritance , young onset Hypertension with +ve family


history salt retention K+ decrease renin and aldosterone

Treatment:
Low salt diet,
Response to amiloride / triamterene (which directly inhibit ENaC).

Return to contents
A
Key difference of common Rare Syndromes

Disease Channel affected Site of nephron defect

Barters syndrome (6 NakCl (loop) Thick ascending limb of


subtypes) loop of henle

Gitelmans syndrome (AR) NaCl (thiazide) Distal convulated tubule

Gordans syndrome (AD) NacCl (thiazide) Distal convulated tubule

Liddles syndrome (AD) ENAC (amiloride) Collecting duct

Syndrome of apparent Aldosterone receptor


Mineralocorticoid excess (spirnolactone)
(AR) defect in 11B-HSD2

Nephron Picture showing


site of defects for rare
syndromes

Return to question 1 Return to contents


A nswer

Rare inherited disorders of potassium

Hypokalaemia + Metabolic alkalosis + Normal Blood Pressure

Bartters Syndrome
Gitlemans Syndrome

Hypokalaemia + Metabolic alkalosis + Hypertension


Liddles Syndrome
Syndrome of apparent Mineralocorticoid Excess
Glucocorticoid remediable Hypertension

Hyperkalaemia + Metabolic acidosis + Normal Blood Pressure

Pseudohypoaldosteronism Type 1

Hyperkalaemia + Metabolic acidosis + Hypertension

Pseudohypoaldosteronism Type 2 (Gordans Syndrome)

Return to contents
Questions 8 & 9 Q
Question 8
In double positive patients (ANCA and anti-GBM antibodies) with standard
treatment, one year Patient survival in patients with Creatinine less than
500 was nearly

A 25%
B 30%
C 50%
D 75%
E 100%

Question 9
Which of the following is the Pathognomonic Eye abnormality
seen in XLinked Alport's Syndrome?

A Posterior Lenticonus
B Anterior Lenticonus
C Peri Macular Flecks
D Corneal Ulcers
E Blindness

Return to question 1 Return to contents


A nswer
8. E - 100%
9. B - Anterior Lenticonus

Answer 8
Options A, B, C and D are incorrect, option E is the correct answer. Patient
survival was 100% at one year when Creatinine was less than 500 in the
presence of both ANCA and anti-GBM antibodies in a retrospective analysis by
Levy et al.

One year outcome in double positive patients


Patient survival (%) Renal survival (%)
N
Creat <500 7 100 71
Creat >500 1 100 0
Dialysis 17 35 0
Total 25 56 40
Levy, Kidney Int 2004
Answer 9
Options A, C, D and E are incorrect. Option B is the correct answer.
The demonstration of lenticonus is diagnostic for Alport's syndrome. Anterior
lenticonus is present in 50% of men, but not women, with X-linked disease,
where it is associated with early-onset renal failure and perimacular retinopathy.
In contrast, lenticonus is common in both men and women with autosomal
recessive inheritance, and therefore, women with Alport's syndrome and
lenticonus are likely to have recessive disease.

Image Showing
Anterior lenticonus
of Alport's syndrome

Return to contents
Question 10 Q
Which of the following immune responses is specific to kidney transplantation?

A Direct allorecognition
B Indirect allorecognition
C Activation of adaptive immune response
D Activation of innate immune response
E None of above

Return to question 1 Return to contents


A nswer A - Direct allorecognition

Options B, C, D and E are incorrect, option A is the correct answer. Direct


allorecognition occurs when passenger dendritic cells from donated organ
travel to lymphoid tissue in the recipient resulting in lymphocyte activation.
This can only take place in the setting of an organ transplant.

Panel Reactive Antibody (PRA) CDC Cross Match (Complement


is an immunological laboratory test routinely dependent cross match)
performed on the blood of people awaiting It seeks to identify clinically significant
organ transplantation. The PRA score is donor specific HLA antibody mediated
expressed as a percentage between 0% responses for a given recipient.
and 99%. It represents the proportion of the The CDC crossmatch. Recipient serum
population to which the person being tested potentially containing donor- specific
will react via pre-existing antibodies. These anti-HLA antibodies is added to
antibodies target the Human Leukocyte Antigen donor T or B lymphocytes, along with
(HLA), a protein found on most cells of the complement
body. Each population will have a different (A). If donor-
demographic of HLA antigens, and so the specific
PRA test will differ from country to country. antibodies
A high PRA usually means that the individual are not
is primed to react immunologically against a present,
large proportion of the population. Individuals no lysis
with a high PRA are often termed "sensitized", occurs and
which indicates that they have been exposed the result
to "foreign" (or "non-self") proteins in the is deemed
past and have developed antibodies to them. negative
These antibodies develop following previous (B). If donor-
transplants, blood transfusions and pregnancy. specific
Transplanting organs into recipients who anti-HLA
are "sensitized" to the organs significantly antibodies
increases the risk of rejection, resulting in bind to the
higher immunosuppressant requirement and lymphocytes
shorter transplant survival. People with high PRA and then
therefore spend longer waiting for an organ activate
to which they have no pre-existing antibodies. complement,
Extensive efforts have been made to identify cell lysis will
treatment regimes to reduce PRA in sensitized occur and the crossmatch result will be
transplant candidates. In certain circumstances, deemed positive (C). The proportion
plasma exchange, intravenous immunoglobulin, of lysed cells is assessed and the
rituximab and other "antibody-directed" crossmatch is graded a being weakly,
immune therapies may be employed, but this is moderately or strongly positive.
an area in which active investigation continues.

Return to contents
Question 11 Q
A 71-year-old woman first noticed bilateral lower extremity oedema in
November of 2014. She reduced her sodium intake and a week later saw her
primary care physician who, in working up the oedema, sent her for a urinalysis
that showed 3+ proteinuria and no haematuria. Previous blood and urine tests,
done in June of 2013 as part of a routine yearly visit, had been unremarkable,
including no detectable proteinuria.
She was referred to a nephrologist in December of 2014 at that visit, she
reported no symptoms other than oedema, which persisted in both lower
extremities to the ankles. Specifically, she denied fevers, anorexia, weight loss
night sweats, rash, joint pain, or change in urine color. Her past medical history
included 50 pack-years of cigarette smoking with ongoing tobacco abuse,
osteoarthritis, and hyperlipidaemia. The only medications that she was taking
at this time were an over the counter fish oil preparation and acetaminophen
as needed for pain. Her family history was negative for renal disease, but her
mother had died of leukemia and her sister had died of uterine cancer.
Her vital signs (including BP) were in normal range, and her physical
examination was notable only for 1-mm pitting edema to the ankles.
Laboratory workup revealed 11g/d proteinuria on 24-
hour collection, Serum Creatinine of 62 umol/l, total
cholesterol of 9.0 mmo/l, and albumin of 26g/L.
The following tests were either negative or in the
normal range: serum protein electrophoresis, urine
protein electrophoresis, antinuclear antibody, C3
and C4, hepatitis B and C, serological tests for A
syphilis, Myeloperoxidase (MPO) and PR3 ANCA.
Renal ultrasound was unremarkable, including no
evidence of renal vein thrombosis, and ultrasound
of the legs showed no deep venous thrombosis.
She was started on lisinopril, atorvastatin, and
furosemide. A Kidney Biopsy was undertaken which
showed: see images B
What is the lesion noted on light, IF, and electron
microscopy?
A Minimal change disease
B Membranoproliferative GN
C Membranous nephropathy (MN)
D Focal segmental glomerulosclerosis C
E Amyloidosis See Kidney Biopsy (A-LM, B-IF, C-EM)

Return to question 1 Return to contents


A nswer C - Membranous Nephropathy (MN)

Options A, B, D and E are incorrect, option C is the correct answer.

The biopsy shows findings of Membranous nephropathy


On light microscopy, the major abnormality is thickening of the glomerular
basement membranes (GBMs) caused by the presence of spike formation
between subepithelial immune complex deposits, which is most easily seen on
the Jones methenamine silver stain (Figure 1A).
IF microscopy shows dominant granular staining for IgG (e.g., 3+) (Figure 1B)
with similar staining for - and -light chains along with granular staining for C3
of lower intensity (e.g. 1+).
On electron microscopy (Figure 1C), subepithelial electron dense deposits and
intervening spikes of GBM material are the hallmark findings of MN.

Common causes of Membranous Nephropathy

Idiopathic-70-75% (Antibodies to Phospholipase receptor APLA2R +ve)

30% associated with Hep B, Hep C, Syphilis


1. Infections

2. Autoimmune SLE, Rheumatoid arthritis,Thyroiditis, Sjogrens


syndrome
3. Malignancy Ca Lung, Prostate

4. Drugs Gold, Penicillamine, NSAID, Captopril

Kidney Biopsy Findings in Membranous Nephropathy

Light microscopy Immunofluorescence Electron


Microscopy

Membranous Thickened capillary Granular capillary Subepithelial


Nephropathy loops and spike and loop staining for IgG electron dense
holes by silver stain and C3 deposits

Return to contents
A
EM - Showing Subepithelial Immune deposits of Membranous Nephropathy

Treatment of Idiopathic Membranous Nephropathy


(Evidence based therapy)

1. Initial conservative treatment: supportive care


2. Treating symptoms (Diuretics for oedema, Dietary salt restriction)
3. Target BP level 125/75 (Acei/ARB) (start with low dose ACEi then increase
ACEi to maximum tolerated dose, if serum K <5 add ARB, if serum K>5
add nondihydropiridine ca antagonist)
4. Add statins in case of hypercholesterolemia
5. Consider anticoagulants (warfarin) in case of hypoalbuminaemia if alb <25
g/l
6. Stop smoking

Return to question 1 Return to contents


A nswer

Possible Treatment choices in patients with idiopathic Membranous


Nephropathy

Treatment Results Notes

Steroids alone No benefit Although ineffective, frequently


used by practitioner

Steroids- Can significantly The results are confirmed by


alkylating increase the probability randomized controlled trials.
agents of complete or partial Risk of side effects (infection,
remission. Protect renal leucopenia). Avoid frequent
function in the long term repetitions (risk of oncogenic or
gonadotoxic effects)

CNI Can significantly reduce Relapse of proteinuria is


(Calcineurin the amount of proteinuria frequent after CNI withdrawal.
inhibitor) and increase the Risk of hypertension,
probability of complete nephrotoxicity. Little information
or partial remission. Little about long-term safety
information about their
effects on renal function
Mycophenolate Ineffective when given Only small-sized studies with
salts alone. Can reduce short-term follow-up are
proteinuria when given available. High relapse rate. No
together with steroidsv information about the long-term
safety and Efficacy
ACTH Can reduce proteinuria Only few small-sized studies
(Adreno- with short-term follow-up
corticotropic are available. A randomized
Hormone) controlled trial is in progress
Rituximab Can reduce proteinuria Large observational studies
available. No head-to- head
comparison with other
treatments

Return to contents
A
Membranous Nephropathy Good & Poor prognostic factors

GOOD PROGNOSIS POOR PROGNOSIS


Female sex Male sex
Proteinuria <4 gm/day >8 gm proteinuria
Complete remission and partial Age >50 yrs
remission
Younger age, Homogenous immune Hypertension, High Serum
deposit then heterogenous Creatinine
Interstitial fibrosis vascular
disease
Troyaonav: KI 2006

KDIGO Guidelines
Treatment should be restricted to high-risk patients
Renal function impairment
Persistent massive proteinuria >8gm/day associated with worse outcome.

In patients with >8 gm proteinuria treatment with above protocol should be


continue for at least 6 months before being considered as treatment failure if
there is no remission

Return to question 1 Return to contents


A nswer

Anti-PLA2R antibodies
Sensitivity 70-78% and specificity 90-99% for primary membranous GN
Not present in patients with non-membranous nephrosis
Correlates with outcome and disease activity; not always with proteinuria; with
recurrence post Transplant
Swedish registry: 9% of patients with positive antibody developed malignancy
vs 37% of those without antibodies (Timmermans 2013 AJKD)
PLA2R-Ab detection in serum and glomerular PLA2R staining are helpful tools
for the diagnosis of MN
PLA2R antibody levels are useful biomarkers in primary MN and allow
individual treatment monitoring

Subtypes of IgG in Primary and Secondary Membranous Nephropathy

Idiopathic Membranous Usually IgG4


Lupus Membranous Strong IgG1, 2,3
Secondary Membranous due to Cancer Mostly IgG1, 2,4 little IgG3

Investigations suggested to detect/exclude an underlying cancer in a patient


with apparently idiopathic (primary) MN and repeatedly negative serologic
tests for anti-PLA2R1 autoantibody and/or absence of PLA2R1 or IgG4 in
glomerular deposits

Cancer Type Young Adult Older Patient


Lung Chest x-ray Computed tomography
Kidney Ultrasonography, malignant cells Ultrasonography, malignant
in the urine cells in the urine
Breast Physical examination Mammography
Stomach Faecal occult blood? Gastroscopy
Colon Faecal occult blood? Colonoscopy
Prostate Rectal digital examination, Ultrasonography, prostate
percentage PSA biopsy
Uterus Gynecologic examination Colposcopy
In young patients, faecal occult blood is usually searched for only in the case of anaemia. MN,
membranous nephropathy; PLA2R1, phospholipase A2 receptor 1; PSA, prostate specific antigen.

Return to contents
Question 12 Q
A 29-year-old man presented with a 7-month history of polyuria, polydipsia
and worsening shortness of breath on exertion for the past 3 weeks.
He had also experienced pain in both feet but could not remember injuring
them recently. On examination blood pressure was 165/83 mmHg,
there were bibasal crepitations on auscultation of the chest and a number
of abdominal telangectasia.

Investigations:
serum creatinine 290 mol/l (60 110)
protein/creatinine ratio 312 mg/mmol (<3.5)

A renal biopsy was performed.

Which of the following is most likely to be found on electron microscopy?

A Tubular reticular inclusion bodies


B Multilamellar myelin bodies in podocytes
C Subendothelial electron dense deposits
D Mesangial electron dense deposits
E Diffuse foot process effacement

Return to question 1 Return to contents


A nswer A - Tubular reticular inclusion bodies

Options B, C, D and E are incorrect, option A is the correct answer.


This patient has Fabrys disease.

Fabrys disease also known as Anderson-Fabys disease is the second most


prevalent lysosomal storage disorder after GauchersFabrys Disease is a rare
X-linked lysosomal storage disease caused by deficiency of a-galactosidase A
(metabolise glycosphingolipids)

Leads to accumulation of globotriaosylceramide & ceramide trihexose


(endothelium).

Fabrys disease is the second most common lysosomal storage disorder after
Gauchers disease

Prevelance is 2-3 Per million populations

Clinical features Multi-organ involvement

Skin Chracterstic angiokeratomas

Eyes Corneal opacities- corneal verticillata

Kidneys Progressive decline in function and proteinuria


is present in 50% of males and 20% of females,
Progressive CKD with Microscopic haematuria,
proteinuria ESRD typically age 30-50.
Heart IHD, LVH, Cardiomyopathy, valvular lesion

Neuro Acropararhesia, CVA, autonomic dysfunction

Lungs SOB, cough, airflow problem

Diagnosis is by measuring the leucocyte alfa- Gal A activity

Kidney biopsy electron microscopy shows characterstic tubuloreticular


inclusion bodies (also known as Zebra Bodies) with an onion skin appearance
(characteristic of liposomal storage disorders)

Return to contents
A
Kidney biopsy electron microscopy shows characteristic tubuloreticular
inclusion bodies (also known as Myelin or Zebra Bodies)

Typical angiokeratoma around periumblical area of Fabrys Disease

Return to question 1 Return to contents


A nswer

Teaching notes - Kidney Biopsy Features of Fabrys disease

Light microscopy Immunofluorescence Electron


Microscopy

Fabrys Focal or segmental Negative Myeloid bodies


disease glomerulosclerosis or zebra bodies
and interstitial in podocytes,
fibrosis and tubules and
tubular atrophy. endothelial cells.

Treatment
Enzyme replacement therapy (ERT) with recombinant alpha-Gal A
ERT should be continued in ESRD patients for its non-renal benefits.

Return to contents
Question 13 Q
A 51-year-old man presented with a palpable purpuric rash (look at the image)
predominantly over his legs and marked dependent oedema.
Further examination revealed 4cm hepatomegly and 1 cm splenomegaly.

Investigations:
serum creatinine 275 mol/L (60110)
urine protein/creatinine ratio 429 mg/mmol (<15)
serum albumin 20 g/dL g/L (3749)
HbA1C 5.7%

ANCA negative
Anti-GBM negative
dsDNA negative
C3 Normal
C4 low
Pre-dialysis screen HIV ab neg, Hep B ab neg, Hep C ab positive

Look at the image.

What is the best management?

A Conservative management
B Chlorambucil and glucocorticoids
C Pegylated interferon
D Pegylated interferon and ribavirin
E Rituximab

Return to question 1 Return to contents


A nswer D - Pegylated interferon and ribavirin

Options A, B, C and E are incorrect, option D is the correct answer. This man
has nephrotic range proteinuria with moderate renal impairment. The rash
and reduced complement levels make membranous nephropathy less likely
as a diagnosis. The presence of Hepatitis C antibody in this setting should
prompt consideration of Type II cryoglobulinaemic vasculitis. A reduction in
proteinuria with the treatment of HCV using interferon alpha without significant
improvement in renal function (milder dysfunction, however, sCr average
1.8mg/dl). In all studies, there are large numbers of relapses when treatment
is withdrawn. Interferon alfa has been shown to be safe in advanced (dialysis
dependent) renal dysfunction Concurrent treatment with ribavirin has been
shown to increase the chances of sustained viral clearance but there have been
many concerns about its use in renal dysfunction due to severe haemolytic
anaemia anaemia as a side effect when levels accumulate.
However there have been two trials suggesting that with dose reduction it
can be a safe treatment. The use of ribavirin was safe in dialysis dependent
patients. One study demonstrated that with close monitoring of Hb and
appropriate iron and epo. Supplementation use of ribavirin was safe down to
an eGFR of 10ml/min at entry.
Ref: Alric et al Am J Kid Dis 2004 Apr;43(4):617-23.
Johnson et al KI 46(1994) 1700-1704
Kokoglu et al J Gastroenterol Hepatol 2006 Mar;21(3):575-80.
Bruchfeld et al NDT (2003) 18: 1573-1580
Rendina et al J Hepatol 2007 May;46(5):768-74

Suggested HCV Treatment in Patients on Haemodialysis

Transplant candidates and patients with long life expectancy: (IFN or PEG-
IFN combined with RBV, for 24 wk in In genotype 2 and 3 or 48 wk in HCV
genotype 1, 4, 5, and 6)
IFN (3 MU) 3 times weekly or PEG- IFN--2a 135 g/wk

RBV initial dose 200 mg daily with further adaptation to plasma concentration
10 to 15 mol/L; with erythropoietin administration.

Patients with short life expectancy: initiation of antiviral therapy should be


based on the degree of the liver damage

Kamar N, Rostaing L, Alric L: Treatment of hepatitis C-virus-related glomerulonephritis. Kidney Int


69 :436 439,2006

Return to contents
Question 14 Q
Which of the following statements is incorrect for treatment in crescentic
glomerulonephritis?

A In MEPEX trial, renal survival was found to be better with plasma


exchange than IV Methylprednisolone.

B In CYCLOPS trial, IV cyclophosphamide was found to be non-inferior to


oral cyclophosphamide in achieving the end point of time to remission.

C In RITUXVAS trial, Rituximab was found to be inferior to


Cyclophosphamide in achieving sustained remission at 12 months.

D In RAVE trial, Rituximab was found to be non-inferior to


Cyclophosphamide in achieving remission without Prednisolone at 6
months.

E In NORAM trial, Methotrexate was non-inferior to Cyclophosphamide in


achieving remission at 6 months.

Return to question 1 Return to contents


A nswer C

Options A, B, D and E are correct, whereas option C is incorrect. Rituximab


was non-inferior to Cyclophosphamide in achieving sustained remission at 12
months in RITUXVAS trial.

NICE Guideline

Rituximab in combination with glucocorticoids for treating anti-neutrophil


cytoplasmic antibody-associated vasculitis

Rituximab taken with glucocorticoids is recommended as a possible treatment


for people with anti-neutrophil cytoplasmic antibody-associated vasculitis (that
is, severely active granulomatosis with polyangiitis [also known as Wegener's
granulomatosis] and microscopic polyangiitis) if:

more treatment with cyclophosphamide would exceed the maximum


amount of cyclophosphamide they can have or

cyclophosphamide is not suitable for them or they cannot take it or they


want to have children and treatment with cyclophosphamide may
affect their fertility or

the disease has stayed active or got worse after a course of


cyclophosphamide lasting 36 months or the person has had cancer
affecting the lining of the bladder and other parts of the urinary
system.

Return to contents
Question 15 Q
A 24-year-old army recruit presents with acute renal failure. Which one of the
following features is not consistent with a diagnosis of rhabdomyolysis?

A Elevated plasma creatinine kinase level


B Anuria
C Elevated plasma creatinine
D Blood and red cell casts in the urine
E Hyperkalaemia.

Return to question 1 Return to contents


A nswer D - blood and red cell casts in the urine

Options A, B, C and E are incorrect. Option D is the correct answer.


Rhabdomyolysis results from muscle injury and the release of myoglobin from
muscle cells. Myoglobin is toxic to the renal tubules and oliguria or even anuria
can occur. Myoglobinuria usually occurs. Muscle cells also release creatinine
kinase and potassium when they are injured. Red cell casts are not a feature of
rhabdomyolysis and would suggest a glomerular disease.

Medications and Toxic Substances That Increase the Risk of Rhabdomyolysis

Direct Myotoxicity Indirect Muscle damage

HMG-CoA reductase inhibitors, especially in Alcohol Central nervous


combination with fibrate-derived lipid-lowering system depressants
agents such as niacin (nicotinic acid; Nicolar) Cocaine Amphetamine
Cyclosporine (Sandimmune) Itraconazole Ecstasy (MDMA) LSD
(Sporanox) Erythromycin Colchicine Zidovudine Neuromuscular blocking
(Retrovir) Corticosteroids agents
HMG-CoA = 3-hydroxy-3-methylglutaryl coenzyme A; LSD = lysergic acid diethylamide; MDMA =
3,4-methylene dioxymethamphetamine.

Complications of Rhabdomyolysis
Early complications
Hyperkalemia
Hypocalcaemia
Hepatic inflammation
Cardiac arrhythmia
Cardiac arrest

Late complications
Acute renal failure
Disseminated intravascular coagulation

Return to contents
Questions 16 & 17 Q
Question 16
A haemodialysis patient is troubled by gout.

Which one of the following treatments is unlikely to prove effective?

A Colchicine
B Allopurinol
C Prednisolone
D Increase Dialysis
E Probenecid

Question 17
Which of the following is not a cause of hypokalemic distal (type 1) renal
tubular acidosis?

A Sjogrens syndrome
B Primary biliary cirrhosis
C Amphotericin B
D Hypogammaglobulinaemia
E Vitamin D intoxication

Return to question 1 Return to contents


A nswer
16. E - Probenecid
17. D - Hypogammaglobulinaemia

Answer 16
Options A, B, C and D are incorrect, option E is the correct answer. Probenecid
is a uricosuric agent, likely to produce little benefit in a patient with severe
renal impairment necessitating haemodialysis. Allopurinol, if tolerated, will
reduce urate production, and increased dialysis will improve urate clearance,
both reducing the serum urate level and thus the predisposition to attacks.
Prednisolone and colchicine both have anti-inflammatory properties, beneficial
in the treatment of acute attacks.

Treatment of acute gout Prevention of gout


NSAIDS Allopurinol 100-900mg daily
Colchicine 500mcg tds Febuxostat 80-120mg daily
Steroids

Answer 17
Options A, B, C and E are incorrect. Option D is the correct answer-
Hypogammaglobulinaemia Hypergammaglobulinaemia causes hypokalaemic
distal renal tubular acidosis hypogammaglobulinaemia is not reported to cause
the distal renal tubular acidosis.
Renal tubular acidosis (RTA)
Type1(distal) Type 2 (proximal) Type 4

Defect Inability to excrete H+ Hco3 lost in Hyporeinemic


ions in distal tubule proximal tubule hypoaldosteronism
Urine pH >5.5 <5.5 <5.5
Serum K Low Low or normal until High
treated

Associated with - + _
Fanconi syndrome
Nephrolithiasis + - -
Treatment Na or K bicarbonate Na or K bicarbonate
or citrate-2mEQ/kg/d or citrate- 5-15
adults mEq/kg/d+/- HCTZ
Causes Sjogren syndrome Multiple myeloma Diabetic or
Rheumatoid arthritis, with cast hypertensive
active cirrhosis, nephropathy, retinopathy,
obstructive uropathy, Acetazolamide, tubulointerstitial
ifosfamide, disease, spironolactone,
toluene toxicity,
tenofovir, heavy ARBs or ACE inhibitors,
amphotericin metals K sparing diuretics

Return to contents
Question 18 Q
A 61-year-old woman on haemodialysis complained of progressive abdominal
pain and bloating for three months. She was otherwise well. Her primary renal
illness was Focal segmental glomerulosclerosis (FSGS), diagnosed 8 years ago.

She was started on peritoneal dialysis immediately and was only changed to
haemodialysis nine months ago following an episode of fungal peritonitis.

On examination she was afebrile with a tight, distended abdomen.

What finding is most likely to be seen on investigation?

A CT evidence of visceral peritoneal calcification


B ECG evidence of bradycardia
C Evidence of small bowel obstruction on abdominal radiograph
D Gram negative bacilli on effluent culture
E Transudative peritoneal effluent

Return to question 1 Return to contents


A nswer
A - CT evidence of visceral
peritoneal calcification

Options B, C, D and E are incorrect, option A is the correct answer. The


patient is likely to have EPS (encapsulating peritoneal sclerosis). There is a
15-20% probability of developing EPS after 8 years on PD. Up to 65% of new
cases occur after stopping peritoneal dialysis. The ascitic effluent is usually
an exudate. There are associations between EPS and duration of dialysis,
membrane type, recurrent peritonitis and some drugs, including beta-blockers.

From UK EPS guidelines


CT scanning is recommended as the diagnostic imaging modality of choice.
However, mild peritoneal membrane changes on CT without encapsulation or
gastrointestinal dysfunction do not make the diagnosis of EPS (GRADE 1B).
a plain abdominal x-ray may be of value in confirming or excluding the
presence of bowel obstruction. If bowel obstruction is present, there will be a
high suspicion of EPS,
in the absence of peritoneal calcification the diagnosis should not be made
on plain film alone.
CT scanning is probably of greatest value - widely available; greatest
reproducibility.

Reporting should be done by individuals with experience of diagnosing EPS


looking for peritoneal calcification, bowel wall thickening, bowel tethering, and
bowel dilatation.

CT Abdomen showing peritoneal calcification in EPS

Return to contents
Question 19 Q
A 55-year-old female with end stage renal disease from cresentric IgA
nephropathy on automated peritoneal dialysis (APD) for 10 years and anuric for
6 yrs. She is doing well overall. Weekly KT/V has always been>1.7/wk. Initial
Hb was 11 g/dl whilst on an Darbepoetin. Recent Hb was 12.6g/dl while not on
oral iron or Darbepoetin. Ferritin is 450 and Transferrin saturation is 30%.

What would be the most appropriate next step?

A Discontinue oral iron


B Imaging of kidney
C Phlebotomy to reduce Hb<12
D No change
E Repeat iron studies

Return to question 1 Return to contents


A nswer B - Imaging of kidney

Options A, C, D and E are incorrect, option B is the correct answer.

This patient has acquired cystic disease. A long term dialysis patients are at risk
of developing Acquired cystic kidney disease.
The cyst may become malignant -Renal cell carcinoma(RCC).These RCC often
produce Erythropoietin about 10% have polycythemia although majority
present with anaemia.

Return to contents
A
CT Scan showing Atrophic Kidneys and Renal cell Carcinoma

Renal cell carcinoma (RCC)


and Peritoneal Dialysis
Atrophic kidneys and RCC

Acquired Cystic disease of dialysis


Severity and incidence increase with longer dialysis duration.
Usually small cysts, but may mimic ADPKD, effects kidneys only and with
no family history of ADPKD
Many regress with transplant.

Return to question 1 Return to contents


A nswer

Differential Diagnosis of Cystic Kidney Disease

Kidney Size/ Cyst Features Extrarenal Features Associated


Shape Neoplasms
ADPKD Enlarged/irregular Common (liver Rare
cysts, heart valve
abnormalities,
IC aneurysms,
diverticula)
ARPKD Enlarged/reniform Congenital hepatic No
fibrosis
Localized Enlarged/irregular No Rare
cystic disease
MSK Normal/reniform No No

TSC Normal or Common (skin Multiple AMLs;


enlarged/reniform lesions, heart, RCC rarely
or irregular brain, retina)
VHL Enlarged if Enlarged if Common, multiple
tumorous/irregular tumorous/irregular (clear cell RCC)
NPHP/MCKD Reduced/reniform In NPHP associated No
with overlapping
syndromes

MCDK Reduced, normal, In multiple Rare


or enlarged/ malformation
irregular syndromes

ACKD Reduced, normal, No Common (papillary


or enlarged/ RCC, ACKD-
irregular associated RCC)

(ACKD, acquired cystic kidney disease; ADPKD, autosomal dominant polycystic kidney disease;
AML, angiomyolipoma; ARPKD, autosomal recessive polycystic kidney disease; ESRD, end-stage
renal disease; IC, intracranial; MCDK, multicystic dysplastic kidney; MCKD, medullary cystic kidney
disease; MSK, medullary sponge kidney; NPHP, nephronophthisis; RCC, renal cell carcinoma; TIN,
tubulointerstitial nephritis; TSC, tuberous sclerosis complex; VHL, von Hippel-Lindau syndrome.)

Return to contents
Question 20 Q
A 72-year-old Caucasian man with a 10-year history of well-controlled type
II diabetes and hypertension presents with nephrotic syndrome (8.4 g/day
proteinuria) and microscopic hematuria. His blood pressure is well controlled
at 128/76 mm Hg, and his BMI is high at 40. He does not have retinopathy.
Serologic testing is negative, but he is found to have a small IgG k monoclonal
spike in the serum and monoclonal k light chains in the urine Serum creatinine
is 305 umol/l with an eGFR of 20-mL/min/1.73 m2.

A kidney biopsy is done. (Look at the image)

Which one of the following is the most likely diagnosis in his scenario?

A Nodular diabetic glomerulosclerosis


B Monoclonal immunoglobulin (k light chain) deposition disease
C Amyloidosis
D Dense deposit disease
E Membranous Nephropathy

Return to question 1 Return to contents


A nswer
B - Monoclonal immunoglobulin
( light chain) deposition disease

Options A, C, D and E are incorrect, option B is the correct answer.


Monoclonal immunoglobulin ( light chain) deposition disease. In monoclonal
immunoglobulin deposition disease, the glomerular changes frequently
resemble diabetic nephropathy with nodular transformation of the mesangium.
This diabetic patient has glomerular nodular mesangial expansion, but
immunofluorescence revealed strong light chain staining throughout the
glomeruli as well as along the tubular basement membranes and vasculature.
Ultrastructural findings showed the deposition of finely granular electron
dense material along the subendothelial aspect of the glomerular basement
membrane (bottom panel). Abundant finely granular electron-dense deposits
were also noted in the mesangium and along the interstitial aspect of the
tubular basement membranes. The glomerular basement membrane thickness
was normal (in fact, segmentally thin), and arterial hyaline deposition (usually
severe in diabetic nephropathy) was mild and focal; therefore, the nodular
glomerular sclerosis was related to light chain deposition disease rather than
diabetic glomerulosclerosis. A Congo Red stain was negative, and fibrillary
material was not seen by electron microscopy. In dense deposit disease,
the electron dense deposits are homogeneous and within the expanded
glomerular basement membrane (not along the subendothelial aspect of it).
Differential diagnosis of Nodular glomerulosclerosis
Diabetic glomerulosclerosis

Chronic membranoproliferative glomerulonephritis (primary or secondary)

Dysproteinemias

amyloidosis

monoclonal Ig deposition disease

Organized glomerular deposition diseases

fibrillary glomerulonephritis

immunotactoid glomerulonephritis

fibronectin glomerulopathy

collagen III glomerulopathy

Chronic hypoxic or ischemic conditions

Takayasu's arteritis with renal artery stenosis

cyanotic congenital heart disease

cystic fibrosis

Idiopathic nodular glomerulosclerosis (often associated with smoking and chronic hypertension)

Return to contents
Question 21 Q
A 64-year-old Caucasian man with a history of hypertension is found to have a
light chain monoclonal spike in both the serum and urine. He reports no history
of diabetes. A 24-hour urine collection reveals 2.8 g proteinuria of which 1.8g
is free light chain. Serum creatinine is 132 umol/L (eGFR, 47 mL/min/1.73
m2 ). Urinalysis reveals glucosuria. A bone marrow biopsy shows few atypical
-restricted plasma cells.

A kidney biopsy is performed.

Which one of the following is the most likely diagnosis in this patient?

A Myeloma Cast Nephropathy


B Fabrys Disease
C Toxic tubulopathy with abnormal mitochondria
D Proximal tubulopathy secondary to intracytoplasmic
monoclonal K crystal formation
E Amyloidosis

Return to question 1 Return to contents


A nswer
D - Proximal tubulopathy secondary to
intracytoplasmic monoclonal crystal formation

Options A, B, C and E are incorrect, option D is the correct answer. Proximal


tubulopathy secondary to intracytoplasmic monoclonal K crystal formation.
Light chain proximal tubulopathy with crystal formation is a rare but not a very
unusual renal complication of myeloma. These crystals are formed by K light
chain. The condition may be missed because these cytoplasmic crystals may
not stand out in routine light microscopy sections, and frequently do not stain
with routine immunofluorescence methods.

Ultrastructural examination of the tubules is diagnostic; the proximal tubular


epithelial cells contain elongated angular, sometimes rhomboid, crystals with
homogeneous structure or fine paracrystalline substructure. The K light chain
restriction in most cases can be proven by immunoelectron microscopy.

Patients with light chain proximal tubulopathy frequently develop Fanconi


syndrome.

The crystals are easy to distinguish from abnormal mitochondria or other types
of tubular epithelial inclusions.

Return to contents
Question 22 Q
A 17-year-old boy is referred to outpatients with nocturnal enuresis. His mother
comments that he is easily tired by sport and is not doing well at school. He
is normotensive and on the 8th centile for height.

Investigations

Sodium 145 mmol/l

Potassium 2.8 mmol/l

Bicarbonate 35 mmol/l

Chloride 80 mmol/l

Urea 5 mmo/l

Glucouse 4.4 mmol/l

24 hr urine K 60 mmol/l
24 hr Urine Na 60 mmol/l

What is the most likely diagnosis in his scenario?

A Laxative abuse
B Diuretic abuse
C Bartter syndrome
D Adrenal tumour
E Villous adenoma

Return to question 1 Return to contents


A nswer C - Bartter's syndrome

Options A, B, D and E are incorrect, option C is the correct answer.

Clue hypokalemic , normotensive with hypochloraemic metabolic alkalosis


with high urine k loss

Bartter's syndrome characterised by severe renal potassium wasting


and inability to concentrate urine. Histologically hyperplasia of JGA
(juxtaglomerular apparatus)

Children present with weakness, polyuria, nocturnal enuresis growth


retardation and low IQ.

Other features include aldosterone and plasma renin levels.

Villous adenoma and laxative abuse is associated with Diarrhoea and low K
loss, adrenal tumour is associated with hypertension.

Teaching - Table showing Difference between Gitelman and Bartter's syndrome

Gitelman Type 3 Bartter


Age at Diagnosis Variable Variable

Symptoms Tetany,
Chondrocalcinosis Variable

K+ Low Low

Mg++ Low Normal

Urine Ca++ Low Normal or High

Nephrocalcinosis No Sometimes

Gene NCCT CLCNKB

Return to contents
A
Algorithm 1

Hypokalemia

Spurious Redistribution

Potassium depletion

Extrarenal etiology Renal K loss


(urine K<20mEq/l) (urine K>20mEq/l)

Metabolic alkalosis

Normal acid-base Metabolic acidosis Metabolic acidosis Variable acid base

lnadequate intake GI tract losses Renal tubular See Magnesium


Anorexia nervosa diarrhoea fistula acidosis algorithm 2 depletion
Tea and toast diet villous adenoma Distal (type I) Acquired
Relative Proxima1 (type II) Hereditary renal
Increase in cell mass Acetazolamide wasting
Copious perspiration DKA Antibiotics
Laxative abuse Uratorosig Penicillins
moldoscopy Aminoglycoside
Leukemia

Teaching- Hypokalaemia differential diagnosis


Algorithm 1 & 2

Return to question 1 Return to contents


A nswer

Algorithm 2

Metabolic alkalosis

Low urine chloride High urine chloride


(urine cl<10mEq/l) (urine cl<10mEq/l)

Normotensive
Normotensive
Vomiting
diuretics
gastric drainage Hypertensive Bartters syndrome
diuretic use
severe k depletion
post-hypercapnea
cl-closing diarrhea Gitelman's syndrome

High aldosterone
and normal renin
Cushings syndrome
High aldosterone

Low renin High renin


primary hyperaldoseronism renovascular hypertension
adenoma malignant hypertension
hyperplasia renin secreting tumor
Low aldosterone and low renin
glycyrrhizic acid
carbenoxolone
exogenous minerallocoticoids
Liddle syndrome
Apparent minerallocorticoid
excess

Return to contents
Question 23 Q
A 68-year-old Caucasian man with a 14-year history of type II diabetes,
hypertension, and chronic obstructive pulmonary disease has a light
chain monoclonal spike in the serum that was interpreted as monoclonal
gammopathy of undetermined significance. Over the last year, his serum
creatinine increased from 153 umol/l (eGFR, 50 mL/min/1.73 m2) to 800 umol/l
(eGFR, 7 mL/min/1.73 m2).

The urinary sediment is bland, and a kidney biopsy is performed.

What is the most likely diagnosis in this patient?

A Granulomatous interstitial nephritis with giant cells


B Diabetic nephropathy with acute tubular necrosis
C Myeloma cast nephropathy
D Myoglobinuric acute kidney injury
E Acute tubular necrosis

Return to question 1 Return to contents


A nswer C - Myeloma cast nephropathy

Options A, B, D and E are incorrect, option C is the correct answer. Myeloma


cast nephropathy. In myeloma cast nephropathy, the monoclonal (in this case,
light chain) protein-containing casts are glassy, refractile, and surrounded by
inflammatory cells (histiocytes and giant cells). Acute tubular injury and some
degree of interstitial nephritis are almost always present. In granulomatous
interstitial nephritis, the giant cells are not within the tubules, but in the
interstitium around granulomas. In myoglobinuric acute kidney injury, the
casts frequently have a globular bright eosinophilic, sometimes pigmented,
appearance without giant cell formation.
Changes of diabetic nephropathy were not evident in this biopsy.
The commonest renal lesion in severe AKI in multiple myeloma
is cast nephropathy.
Rapid identification of immunoglobulin FLC (free light chain) is a
critical component of diagnosis of cast nephropathy secondary to
multiple myeloma.
Myeloma Cast Nephropathy:

Seen in about half of myeloma patients with renal disease


Caused by precipitation of light chains in renal tubules
Predisposing factors dehydration, infection, NSAIDS, radiocontrast material

Clinical Features
Diagnostic criteria for Multiple Myeloma
Serum and/or urine monoclonal protein and/or clonal bone marrow plasma
cells, Presence of plasma cell dyscrasia-related lytic bone lesions, anaemia,
Hypercalcaemia, or renal failure.

Return to contents 70
A
TYPICAL
Progressive renal insuffiency over 1-3 months
Bland urine sediment
SFLC>1500mg/L (Serum Free light chains)
Dipstick negative for albumin,but positive on
heat/sulfosalicylic acid
(High UP/Creat, but low MALB/Creat)
Consider biopsy if above not present

Other features
Hypercalcaemia
Hyperphosphatemia and anaemia out of proportion to renal
failure
Low or positive serum anion gap

Renal manifestation in patients with Multiple Myeloma

Typical clinical manifestation/histological feature


Commonest Cast nephropathy Hyaline fractured casts, degenerated tubular cells
Fibrils; Congo red staining positive, apple-green
AL amyloidosis
birefringence under polarized light
LCDD Deposition of light chains

Heavy-chain deposition
Deposition of heavy chains
disease
Tubular disturbance
Fanconi syndrome Hyperaminoaciduria, glucosuria

Renal insufficiency caused by


Hypercalcaemia high plasma calcium
Hyperviscosity high plasma monoclonal IgM
Hyperuricaemia hyperuricaemia (high tumour load)

MPGN. Cryoglobulins

Plasma cell infiltrates Direct infiltrates

Rhabdomyolysis Light-chain deposition in the muscle

Pyelonephritis/sepsis Immunodeficiency with frequent infections

Return to question 1 71 Return to contents


A nswer

Use of immunoglobulin light chain assay in diagnosis of paraprotein related


Kidney disease

Acute Kidney injury

SPE-serum protein electrophoresis, FLC-Free light chain

Return to contents
Question 24 Q
Current UK guidelines for combined liver kidney transplant do not include:

A Genetic liver kidney syndromes; Oxalosis, Glycogen storage disease type 1

B Chronic liver disease meeting at least one of the three current criteria for
transplant selection + end stage renal disease on long term dialysis program

C Chronic liver disease meeting at least one of the three current criteria for
transplant selection + Hepatorenal syndrome with serum creat >200 and
dialysis >8 weeks

D Chronic liver disease meeting at least one of the three current criteria for
transplant selection + MDRD stage 3b or GFR <30 ml/min or renal biopsy
showing >30% fibrosis and/or glomerulosclerosis

E Mitochondrial disorders

Return to question 1 Return to contents


A nswer E - Mitochondrial disorders

No indication for combined liver kidney transplants in mitochondrial disorders.


Options A, B, C, D are indications for combine kidney and liver transplantation.

Indications for simultaneous liver and kidney transplantation

Primary hyperoxaluria
Hereditary amyloidosis
HUS secondary to hereditary complement mutations
ADPKD:ESRD with large , symptomatic polycystic liver disease
Hepatorenal syndrome with irreversible renal failure

Return to contents
Question 25 Q
A 65-year-old man started Haemodialysis with a 14-week-old left
Brachiocephalic AVF which is now successfully needled. It has a strong thrill
and is well matured.
The patient developed pins and needles in the fistula hand, acro-cyanosis and
dull aching pain whilst on dialysis.
On examination his hand was cold to touch with absent radial pulse and
reduced grip. There were signs of nail dystrophy, shiny skin and areas of patchy
redness.

Look at the image

What is the most likely diagnosis in his clinical scenario?

A Carpal Tunnell syndrome


B Dialysis related Amyloidosis
C Steal syndrome
D Cellulitis
E Atheroembolism

Return to question 1 Return to contents


A nswer C - Steal syndrome

Options A, B, D and E are incorrect, option C is the correct answer. This Patient
has Steal syndrome. Dialysis Ischaemic Steal syndrome is an uncommon but
serious condition of arterial insufficiency distal to a permanent haemodialysis
fistula. When severe it can lead to digital gangrene.
This has to be differentiated from Ischaemic Monomelic Neuropathy which
occurs immediately after the access surgery due to damage to the nerves
of the hand.

Treatment- Access requires revision or banding to increase the resistance


moving the arterial inflow more proximally or sometimes closure.

Stages of Steal Syndrome

Normal Arteriovenous Fistula

Return to contents
Question 26 Q
A 77-year-old man with lower extremity edema for many months now presents
with nephrotic syndrome and 10 g of proteinuria. He has no other past medical
history. Complements are normal and other serologies including

Antinuclear antibody and antineutrophil cytoplasmic antibody are negative.


A fat pad biopsy done to rule out amyloidosis is inconclusive.

A kidney biopsy revealed amorphous eosinophilic material infiltrating the


mesangium and peripheral capillary walls on light microscopy, lambda light
chain staining on immunofluorescence, and randomly arranged fibrillary
deposits that are 10 nm in average diameter on electron microscopy (EM).
Congo Red staining is positive.

What is the most likely diagnosis on kidney biopsy in this patient?

A Immunotactoid Glomerulonephritis
B AL Amyloidosis
C Lupus Nephritis
D Cryoglobulinemic glomerulonephritis
E Light chain deposition disease

Return to question 1 Return to contents


A nswer B - AL Amyloidosis

Options A, C, D and E are incorrect, option B is the correct answer. Amyloidosis


is Congo Red positive with an often nodular appearance on light microscopy.
When due to monoclonal light chain (AL type), the immunofluorescence
shows staining for one but not the other light chain. The electron microscopy
(EM) shows randomly arranged fibrils 8-10 nm in diameter.Monoclonal
immunoglobulin disease (light-chain deposition disease, heavy chain
deposition disease, light and heavy chain deposition disease)is Congo Red
negative and monotypic light and/or heavy chains on immunofluorescence.
EM shows amorphous deposits that are usually finely granular. Immunotactoid
glomerulopathy (immunotactoid glomerulonephritis) is Congo Red negative,
with variable lesions on light microscopy such as mesangial proliferative
changes or a membranoproliferative pattern. The immunofluorescence shows
mostly monoclonal immunoglobulin G (IgG) or IgM, however, an M spike or
cryoglobulins cannot be demonstrated in the circulation. EM shows 30-90 nm
hollow microtubules arranged as parallel stacks. The process is often associated
with lymphoproliferative disorders. Fibrillary glomerular disease (fibrillary
glomerulonephritis) is Congo Red negative, with variable light microscopy
findings, similar to immunotactoid glomerulonephritis. Immunofluorescence
shows IgG and C3 with equivalent kappa and lambda light chains. The EM
shows non-branching fibrils, randomly arranged in an amorphous matrix. The
fibrils are larger than amyloid and measure around 20nm. Cryoglobulinemic
glomerulonephritis is Congo Red negative, with a diffuse proliferative,
membranoprolierative, or mesangioproliferative pattern. Pseudothrombi can
be seen in the glomerular capillaries. Immunofluorescence findings depend on
the type: monotypic.

Kidney biopsy Light microscopy Immunofluorescence Electron Microscopy


finding

Amyloidosis Amorphous deposits in Depends on sub type. Fibrillary deposits


glomeruli, vessels and Amyloid Al stains in mesangium and
interstitium which are for either kappa or subepithelial regions.
silver stain negative, lambda light chains. Fibrils measure 8-10 mm in
PAS stain weak, Congo Other types of amyloid thickness.
red positive . are generally negative.
Kidney biopsy features
Fibrillary Amorphous glomerular Bright stain for IgG. Fibrils measuring more than
glomerulopathy mesangial deposits. May be monoclonal. 15 mm in thickness.
Immunotactoid Amorphous glomerular Bright stain for IgG. Fibrils measuring more than
glomerulopathy mesangial deposits. May be monoclonal. 30mm with a microtubular
(Variant of structure and is associated
fibrillary with monoclonal
glomerulopathy) gammopathies.

Return to contents
Questions 27 & 28 Q
Question 27
Which one of these antibiotics least likely to achieve therapeutic concentration
in cystic fluid when treating UTI in polycystic disease?

A Ciprofloxacin
B Trimethoprim
C Chloramphenicol
D Cephalosporins
E Co-trimoxazole

Question 28
A 55-year-old male who has end stage renal disease secondary to hypertension
is stable on peritoneal dialysis for the last 2 years. He is on Automated
peritoneal dialysis (APD) with 2 litre Icodextrin as his last fill. He has remained
stable but for the last two weeks, he has become more volume overloaded.
There is no change in his dialysis prescription or ultra-filtration.
No history of infection or change in medication in the recent past.

What is the most probable reason for the volume overload?

A Change in peritoneal membrane character


B He has changed the icodextran to night
C He has reduced icodextran volume to 1.5 liter
D His dwell time has increased
E Reduced urine output.

Return to question 1 Return to contents


A nswer
27. D - Cephalosporins
28. E - Reduced in urine output

Answer 27
Options A, B, C and E are incorrect, option D is the correct answer.
Cephalosporins, Penicillins and aminoglycosides do not penetrate adequately
into cysts. (Muther J Urology 124:596,1980) (Elzinger. KI 32:884, 1987)
Answer 28
Options A, B, C and D are incorrect, option E is the correct answer. Patient
has dropped down urine output. Urine output is contributory factor
to maintain Kt/V. and euvolaemia. It is common finding with patient
on peritoneal dialysis.
The overall prognosis and wellbeing of the PD patient is closely linked to
the degree of residual kidney function at the start and maintained during
PD therapy.
Well marked CANSUSA study showed for every 5ml/min of residual kidney
glomerular filtration rate there was 12% decrease in the relative risk of death
and for every 250 ml of urine the relative risk was reduced by 36%.

Clinical consequences of loss of residual kidney function


1. Reduced survival
2. Erythropoietin resistance
3. Worsened bloodpressure control
4. Increase risk of volume expansion with edema
formation
5. Reduced middle molecule clearence
6. Worsened cardiac hypertrophy
7. Low serum albumin level
8. Elevated calcium phosphorus product

Strategies to prevent Residual kidney function
1. Avoid prolonged use of aminoglycosides
2. Use of Loop diuretics-Furosemide 250 mg od increases urine output
but not GFR
3. Use of RAS (Renin angiotensin system) inhibitors
4. Avoidance of intravascular volume depletion
5. Avoidance of use of iodinated contrast agents-Make sure patient is
well hydrated and consider using acetylcysteine beforehand
(no good evidence for benefit)
6. Avoidance of use of NSAID

Return to contents
Question 29 Q
A 60-year-old male is on haemodialysis for the last 3 years. He suffers from
repeated episodes of intra dialytic hypotension. His average weight gain is
one kilogram over his dry weight between dialysis. He is not hypertensive.
Examination did not reveal any signs of volume overload.

Laboratory data showed

Serum calcium 2.6 mmol/L


Serum sodium 144 mmol/L
Serum potassium 4.5 mmol/L
Serum phosphorus 1.5mmol/L
Serum albumin 35 g/L.

What is the best intervention to prevent intra-dialytic hypotension?

A Low calcium bath


B Intravenous albumin during dialysis
C Reduce dialysate temperature
D Ultrafiltration profillng
E Low Dialysate sodium bath

Return to question 1 Return to contents


A nswer C - Reduce dialysate temperature

Options A, B, D and E are incorrect, option C is the correct answer. This patient
has intradialytic hypotension(IDH), In this scenario with the above findings,
reducing dialysate temperature during dialysis may help in stabilising blood
pressure. Low dialysate temp (36.5) will cause vasoconstriction so raising the BP
without tachycardia or increased myocardial contractility.
Cooled dialysate reduces IDH (intradialysis hypotension). Low sodium and
calcium bath will further lower down blood pressure during dialysis. Intravenous
albumin will not add anything, as patient is having normal albumin level.
IDH is defined as fall in SBP >20 mmHg, associated with symptoms or fall
to SBP<100 mmHg.
Symptoms associated with IDH
Cramps, abdominal pain or nausea (reduced gut perfusion)
Dizziness (reduced cereberal perfusion)
Chest pain or arrhythmias

IDH always related to rate of ultrafiltration, It is more common if UF


rate>0.6 ml/min/kg and less if UF rate <0.3 ml/min/kg.

IDH more common in


1. Elderly
2. Diabetics (autonomic neuropathy)
3. Cardiomyopathies
4. Occult sepsis

Management of Interdialytic Hypotension
Modified from European best practice guidelines
First line
Careful clinical reassessment of dry weight
Dietary sodium restriction reinforcement
Refrain from food intake during dialysis
Use of dialysate temp of 36.5 C - this leads to cutaneous
vasoconstriction, which helps maintain BP during dialysis.
Check dosing and timing of antihypertensive agents

Second line
Perform cardiac evaluation- Echocardiography
Gradual reduction of dialysate temperature from 36.5 to 35 C
Switch to haemodiafiltration
Use dialysate calcium concentration of 1.50 mmol/L
Prolong dialysis time or increase dialysis frequency

Third line
Consider midodrine po 2.5-10 mg before dialysis
Consider L carnitine(IV 20mg/kg/treatment) supplementation after
dialysis
Consider PD

Return to contents
Question 30 Q
A 50-year-old male with end stage renal disease secondary to diabetic
nephropathy is on CAPD for the last 1 year. For the last month or so, he
is developing fluid retention in his extremities. His CAPD regime has not
changed and his ultra- filtration is going down. He is on 2 litre dwell volume
with 1.25% dextrose base solution. On examination he is oedematous while
his D/P ratio is 0.9.

What is the best measure to improve ultrafiltration?

A Change to haemodialysis
B Increase the dialysate concentration to 4.25%
C Change to automated peritoneal dialysis
D Continue the same regimen
E Encourage the patient to decrease fluid intake.

Return to question 1 Return to contents


A nswer
C - Change to automated
peritoneal dialysis

Options A, B, D and E are incorrect, option C is the correct answer. Patient is


rapid transporter with D/P is 0.9. He will get benefit to change his
regimen to APD. Icodextrin can be added as a long dwell to improve the
ultrafiltration further.

The peritoneal equilibration test (PET)

PET Test interpretation

D/P creat 0.81-1.03 High(fast) Transporter


D/P creat 0.65-0.81 High average Transporter
D/P creat 0.50-0.65 Low average Transporter
D/P creat 0.34-0.50 Low(slow)Transporter

D/P=Dialysate/Plasma

Return to contents
A
High transport implies a structural or functional alteration of the peritoneum

A larger effective peritoneal surface area


A higher intrinsic membrane permeability (for the rapid equilibration of small
solutes including creatinine and urea).

High transporters are prone to lose the osmotic gradient required for sustained
ultrafiltration because of rapid absorption of glucose from the dialysate.

Subsequent decrease in ultrafiltration capacity


Tendency to have greater systemic exposure to glucose than low
transporters do.

High transporters tends to have problems achieving ultrafiltration goal but are
efficient with clearance

Low transporters tend to achieve ultrafiltration goal but have difficulty with
clearance targets.

High transporters were thought to do best on regimens that frequent involve


short duration dwells (APD) maximizing ultrafiltration.

Low transporters needed longer dwell times (CAPD) to maximize clearance

Memberane transport status should be measured within 6 months of starting PD


Ideally measurements should be repeated annually
Measurement should be repeated in anuric patients if declining Ccrea or
Ultrafiltration
All patients should have a minimal fluid removal >750 ml/day(combination of urine
output and UF)
If Ccrea <50 liters/week/1.73 m2, dialysis prescription should be increased, if not
possible then transfer patient on HD

Return to question 1 Return to contents


A nswer

These are the typical PD regimens used to achieve adequate solute


clearance according to patient size and membrane characteristics in anuric
patients.
The total volume of dialysate fluid required increases with body size
(2.5 to 3 L1 exchange).
APD using shorter overnight dwells is favored over CAPD, as solute
transport increases.
Both CAPD and APD may have to be augmented by use of an
additional exchange (for increased solute clearances or UF
respectively).
The use of icodextrin solution for the long exchange will enhance both
solute clearance and ultrafiltration.

Return to contents
Questions 31 & 32 Q
Question 31
A 60-year-old male, ex-smoker, suffered from right sided hemiparesis four
weeks ago. He has a family history of type 2 Diabetes mellitus. Lab tests
revealed hyperlipidemia, HbA1c of 5.5 and normal renal profile.
He was started on simvastatin 10 mg daily by his general practitioner.

Recent clinical trials demonstrated that statins are associated with increase
development of one the following?

A Hypertension
B Hyperlipidemia
C Diabetes mellitus
D Cancer
E Cerebrovascular accident

Question 32
Incidence of anti-GBM disease in patients with Alports undergoing kidney
transplant is ?

A 5%
B 10%
C 25%
D 50%
E 75%

Return to question 1 Return to contents


A nswer
31. C - Diabetes mellitus
32. A - 5%

Answer 31
Options A, B, D and E are incorrect, option C is the correct answer. Statins are
associated with increased risk of developing Diabetes mellitus, while initially it
was labelled that statins are carcinogenic but later studies proved that statins
do not increase risk of cancer.

Answer 32
Options B, C, D and E are incorrect, option A is the correct answer. Less than
5% of patients with Alports syndrome develop de novo anti-GBM disease
following renal transplantation.
More common in males. Antibodies are directed against alpha-5(IV) and
alpha-3(IV) chains on the basement membrane.
Plasma exchange and Cyclophoshamide(which is used in primary anti-GBM) is
of limited value. High rate of recurrence on re transplantation.

Return to contents
Questions 33 & 34 Q
Question 33
The CYCLOPS trial for induction of remission of ANCA associated vasculitis
showed following results

A I.V. cyclophosphamide has comparable same remission and relapse rate with
oral cyclophosphamide but reduces cumulative cyclophosphamide dose.
B I.V. cyclophosphamide has comparable same remission and relapse rate with
oral cyclophosphamide but increases cumulative cyclophosphamide dose.
C I.V. cyclophosphamide does not has comparable same remission and relapse
rate with oral cyclophosphamide but reduces cumulative cyclophosphamide
dose.
D I.V. cyclophosphamide has same remission but more relapse rate with oral
cyclophosphamide but decreases cumulative cyclophosphamide dose.
E None of above.

Question 34
Which of the following disease is not associated with linear deposits of IgG on
the Glomerular basement membrane?

A Diabetes mellitus
B Light chain disease
C SLE
D Membranous nephropathy
E Cadaver kidneys.

Return to question 1 Return to contents


A nswer
33 - A
34 - D

Answer 33
Options B, C, D and E are incorrect, option A is the correct answer. I.V.
cyclophosphamide has comparable same remission and relapse rate with oral
cyclophosphamide but reduces cumulative cyclophosphamide dose.

CYCLOPS study was comparison with intravenous and oral cyclophophamide


as induction therapy. Intravenous cyclophophamide therapy showed the same
remission and relapse rate compared to oral cyclophosphamide therapy but
overall cumulative dose was reduced. The length of follow up was too short in
this study
Ann Intern Med. 2009 May 19;150(10):670-80.

Long term outcomes were ascertained from the patients previously recruited
in the CYCLOPS study which showed pulsed cyclophosphamide is associated
with a higher relapse risk than daily oral cyclophosphamide. However, this is
not associated with increased mortality or long-term morbidity
Ann Rheum Dis. 2012 Jun;71(6):955-60. doi: 10.1136/annrheumdis-2011-200477. Epub 2011 Nov 29

Answer 34
Options A, B, C and E are incorrect, option D is the correct answer.
Membranous nephropathy.

Membranous nephropathy binds to Glomerluar basement, but IgG binding to


GBM is granular while the rest above mentioned diseases have linear binding
to GBM.

Causes of Linear Binding to Glomerluar Basement Membrane

1. Goodpasture's syndrome
2. Alport's syndrome after renal transplantation
3. Fibrillary glomerulonephritis
4. Diabetic Nephropathy
5. LCDD (Light chain deposition disease)

Return to contents
Questions 35 & 36 Q
Question 35
All of the following factors favours the aggressive treatment of Goodpastures
syndrome except?

A Presence of pulmonary haemorrhage


B Absence of oliguria
C Creatinine less than 500 umol/L at presentation
D No other co-morbid condition
E Creatinine more than 500umol/L at presentation.

Question 36
A 32 year old female was admitted to hospital due to community acquired
pneumonia. During her stay, she was found to have haematuria, proteinuria
and mild renal impairment. Her hepatitis serology came back as negative,
complements and ASO titres were normal, Anti GBM, ANCA, ANA & anti-ds
DNA antibodies also came back as negative. She went on to have a kidney
biopsy which showed IgA Nephropathy with Crescents in 10% of glomeruli.
Bloods were sent off including blood cultures, the results of which
were pending.

What will be the first line of treatment in this patient ?

A Antibiotics
B Mycophenolate mofetil
C Cyclophosphamide
D Plasmaphresis
E Corticosteroids

Return to question 1 Return to contents


A nswer
35. E - Creatinine more than
500umol/L at presentation.
36. A - Antibiotics

Answer 35
Options A, B, C and D are incorrect, option E is the correct answer.
Creatinine more than 500umol/L at presentation.

Creatinine more than 500umol/L on presentation is a bad prognostic sign.


Other bad prognostic signs are oliguria at presentation, severe damage
on kidney biopsy, unusually high risk from immunosuppression and other
associated co-morbid conditions present at the time of diagnosis do not favour
aggressive treatment.

Transplantation is not recommended until anti GBM antibodies have


been absent for 6 months. If present then further treatment with
Rituximab is contemplated.

Answer 36
Options B, C, D and E are incorrect, option A is the correct answer, Antibiotics.

Patient has mild renal impairment and kidney biopsy showed crescents in 10%
of glomeruli. The patient has active infection and the priority here is to treat the
infection first.

Return to contents
Question 37 Q
One of the trials showed intensive blood pressure control improves renal and
patient survival. The effect is independent of baseline renal profile,
but there was increased mortality after reducing blood pressure to below the
following target?

A 120 mmHg systolic


B 110 mmHg systolic
C 100 mmHg systolic
D 130 mmHg systolic
E 140 mmHg systolic

Return to question 1 Return to contents


A nswer A - 120 mmHg systolic

Options B, C, D and E are incorrect, option A is the correct answer. Irbesartan


diabetic nephropathy trial, IDNT analysis showed that lowering the blood
pressure below 120 mmHg systolic is associated with increased mortality.

Albumin/Creatinine Ratio interpretation


Reference range On spot urine 24 hr urine collection

Normal <30mg/gm <30mg/24 hr


<17mg/gm (men)
<25mg/gm (women)
Microalbuminuria 30-300 mg/gm (either sex)
<17-300mg/gm (men) 30-300 mg/24 hr
25-300mg/gm (women)

Macroalbuminuria >300 mg/gm >300 mg/24 hr

Albumin/Creatinine ratio is used to measure albuminuria as a sign of kidney


disease and in diabetes and hypertension, which may lead to kidney disease
or cardiovascular disease.

Return to contents
Question 38 Q
38 year old male comes to you for consultation two days after an upper
respiratory tract infection with haematuria. He was investigated further
and found to have a urine protein to creatinine ratio of 550. All serological
markers were negative. Complements level were normal. A Kidney biopsy was
suboptimal and tissue was sent, only for immunofluorescence.

What is the most likely diagnosis?

A Focal segmental glomerulosclerosis (FSGS)


B Membranoproliferative glomerulonephritis
C Membranous nephropathy
D IgA Nephropathy
E Postinfectious glomerulonephritis

Return to question 1 Return to contents


A nswer D - IgA Nephropathy

Options A, B, C and E are incorrect, option D is the correct answer. Patient


has IgA nephropathy. With a history of upper respiratory tract infection,
the differential diagnosis here is IgA Nephropathy or Poststreptococcal
glomerulonephritis (PSGN). He developed haematuria very soon upper
respiratory tract infection which clearly points towards IgAN. In post infectious
glomerulonephritis, it takes one to three weeks to develop. Haematuria (urine
looks smoky or coca cola coloured) if it is pharyngitis and three to six weeks if it
is skin infection.

IF in IgAN shows large globular mesangial IgA deposits which are


characteristic. The capillary walls are not outlined.
IF in PSGN the deposits are distributed in a diffuse granular pattern within
the mesangium and the capillary wall.

Kidney biopsy IgA Nephropathy

Light microscopy Immunofluorescence Electron


Microscopy
IgA Normal glomeruli, Mesangial IgA Mesangial
nephropathy mesangial deposits. electron dense
hypercellularity, deposits.
proliferative
glomeruli, crescentic
glomeruli.

Return to contents
A
The Oxford classification of lgA nephropathy:

Mesanglal hypercellularity - in > or <50% of glomeruli MO or M1

Endocapillary hypercellularity - present/absent EO or E1

Segmental sclerosis/adhesions - present/absent SO or S1

Tubular atrophy/interstitial fibrosis - 0-25%, 26-50%, >50% TO or T1 or T2

KDIGO Guidelines- Treatment of IgA Nephropathy

IgAN- Proteinuria is IgAN -Proteinuria is more Crescentic IgAN


more than 1 g/d than 1 g/d despite 3-6 Steroids and
long-term ACE-1 or months ARB/ACE and BP 125/75
cyclophosphamide
and GFR more than 50ml/min
ARB 6-months course of analogous to the
Blood pressure corticosteroid therapy on treatment of ANCA
target- 125/75 top of ACE/ARB vasculitis

Poor Prognosis Factors for IgA Nephropathy


1. Impaired Renal Function
2. Proteinuria>3gm/day
3. Difficult to control Hypertension
4. Severe (>25%) tubulointerstitial fibrosis and glomerulosclerosis on kidney biopsy
5. Rapidly Progressive Crescentic IgA Nephropathy

Return to question 1 Return to contents


A nswer

Differential Diagnosis of Mesangial Proliferation

Disease Differentiating Features


1. IgA nephropathy Mesangial IgA, IgM, IgG, C3, kappa, lambda
2. Lupus nephritis (class II) Full-house IF, TRI by EM
3. Incidental or Mesangial C3, subepithelial humplike
incompletely resolved PIGN deposits
4. IgM nephropathy Mesangial IgM
5. C1q nephropathy Mesangial C1q, IgG, IgM, C3; FSGS variant (?)

IF - immunofluorescence, TRI -Tubuloreticular inclusion

Return to contents
Question 39 Q
All of the following are Indications of kidney biopsy in diabetic patients with
proteinuria except?

A No retinopathy
B New onset nephrotic range proteinuria
C Macroscopic hematuria
D Red cell cast
E. Type 1 diabetes mellitus more than10 years

Return to question 1 Return to contents


A nswer
E - Type 1 diabetes mellitus
more than10 years

Options A, B, C and D are incorrect, option E is the correct answer. Type 1


diabetes mellitus > 10 years is not an indication for biopsy. Patients with long
standing type 1 diabetes mellitus for more than 10 years develop proteinuria
due to diabetic nephropathy. Urological causes should be ruled out in
macroscopic haematuria; Red cell cast is of glomerular origin and is present in
glomerulonephritis. Patients with nephropathy and type 1 DM almost always
have retinopathy or neuropathy. The converse is not true.

Indications for kidney biopsy in Diabetic nephropathy


1. Absence of diabetic retinopathy
a. Particularly in T1DM(retinopathy is observed in 85-99% of
patients with established nephropathy
b. T2DM patients have diabetic nephropathy without retinopathy
more commonly
2. Presence of dysmorphic RBC, red cell cast or macroscopic
haematuria
3. Rapid onset, rapidly increasing massive proteinuria
4. Rapid worsening GFR despite good BP and glycaemic control
5. Symptoms or signs suggestive of other multisystem disorders

Classification of Diabetic Nephropathy

Return to contents
Questions 40 & 41 Q
Question 40
Clinical trials prove the following effect of statins on cardiovascular outcome in
end stage renal disease patients?

A Statins improve cardiovascular outcome


B Statins have no effect on cardiovascular outcome
C Statins increase cardiovascular events
D Statins increase cereberovascular events
E Non of above

Question 41
Which of the following glomerular disease is associated with higher percentage
of crescents formation on kidney biopsy?

A Anti GBM antibody mediated GN


B ANCA associated Glomerulonephritis
C Lupus Nephritis stage IV
D Lupus Nephritis stage 5(Membranous)
E Acute post-infectious GN

Return to question 1 Return to contents


A nswer
40. B - Statins have no effect on
cardiovascular outcome
41. A - AntiGBM

Answer 40
Options A, C, D and E are incorrect, option B is the correct answer.
Statins have no effect on cardiovascular outcome. In CKD patients, who are not
on dialysis, treatment with statins improve cardiovascular outcome.
In ESRD and dialysis dependent patients statins showed no beneficial effect in
terms of cardiovascular outcome.
Statins do not prevent the loss of renal function.
2013 KDIGO guideline suggests continuation of statins which was initiated
prior to dialysis.

Answer 41
Options B, C, D and E are incorrect, option A is the correct answer. AntiGBM
antibody mediated Glomerulonephritis.
All of the options in question are associated with crescents formation but
Anti GBM antibody mediated GN is associated with about 95% of crescents
formation.

Avg
with> Glomeruli
% with 50% with
Disease Crescents Crescent with Crescents

Anti-GBM antibody-mediated GN 95 81 77
ANCA-associated GN 90 48 48
Immune complex-mediated GN
Lupus GN (class Ill and rv) 40 11 31
HSP 53 5 24
lgA nephropathy 27 5 24
Acute postinfectious GN 25 4 17
Fibrillary GN 20 7 29
Type I membranoproliferative 20 3 21
GN
Membranous lupus GN {class V) 12 1 17
Membranous GN {non-lupus) 5 0 17

Table showing different glomerular disease with percentage of crescents formation


Return to contents
Questions 42 & 43 Q
Question 42
A 65-year-old man with chronic renal insufficiency presents with weakness,
paresthesias, and progressively worsening shortness of breath. He has been
experiencing these symptoms for 4 days. Laboratory findings show potassium
level of 7.2; an electrocardiogram reveals peaked T waves and widening of the
QRS complex.

Which one of the following is not indicated in the initial treatment of this
patient?

A Intravenous calcium
B Intravenous glucose and insulin
C Dialysis
D Sodium polystyrene sulfonate (Kayexalate, Kionex)
E Beta blockers

Question 43
22-year-old woman presents to the emergency department with nausea,
vomiting, and abdominal pain of 4 days' duration. Her fluid balance profile is as
follows: Na+, 145; K+, 5.0; Cl-, 105; HCO3-, 15; urea, 37mmol/L; Cr, 175umol/l
glucose, 780; Urine analysis 4+ ketones.

What is the best initial treatment of this patient's acid-base disorder?

A Free water
B Normal saline
C Normal saline, sodium bicarbonate, and insulin
D Half normal saline and insulin
E Normal saline and insulin

Return to question 1 Return to contents


A nswer
42. E - Beta-blockers
43. E - Normal saline and insulin.

Answer 42
Options A, B, C and D are incorrect, option E is the correct answer.
Beta-blockers.
The initial manifestations of hyperkalemia are usually neuromuscular in
origin and are non-specific. Diagnosis is based on serum potassium level;
emergent treatment is based on whether cardiac arrhythmias are present or
electrocardiographic changes are occurring. Treatment involves the use of
intravenous calcium to reduce the excitability of cardiac cell membrane and
use of intravenous glucose and insulin to facilitate transport of potassium into
the intracellular space. Sodium polystyrene sulfonate is used to increase the
excretion of potassium in the colon. In refractory cases, dialysis may be initiated
to rapidly remove serum potassium. In addition, intranasal beta agonists may
be used to transiently reduce serum potassium levels in the acute setting. Beta-
blockers can increase potassium levels.

Answer 43
Options A, B, C and D are incorrect, option E is the correct answer.
Normal saline and insulin.
To understand the diagnosis and treatment of diabetic ketoacidosis Metabolic
acidosis can be classified into two types: that associated with an elevation
in the anion gap, and that in which the anion gap is normal. A calculation
of the anion gap in this patient reveals a gap of 25. Among the causes of
acidosis associated with an elevated anion gap are alcoholic ketoacidosis,
lactic acidosis, starvation, ingestion of alcohols, ingestion of salicylates, and
diabetic ketoacidosis. In patients with diabetic ketoacidosis (such as this
patient), optimal initial treatment includes fluid replacement with normal saline
to promote ketonuria and insulin to facilitate glucose transport. Bicarbonate
therapy is not usually indicated unless the acidosis is severe or severe
hyperkalemia is present.

Return to contents
Question 44 Q
A 58-year-old woman with end-stage kidney disease due to diabetic kidney
disease is evaluated 4 weeks following pre-emptive living donor kidney
transplantation. The serum creatinine level fell from 670 umol/L prior to
transplantation to 98 umol/L at discharge. One week ago, a double-J ureteral
stent placed at the time of surgery was found coiled in the bladder and was
removed cystoscopically.

On physical examination, vital signs are normal. There is mild suprapubic


tenderness, and the remainder of the examination is normal.
The serum creatinine level is increased at 175umol/L and the tacrolimus level is
7 ng/dl.

Urinalysis reveals no proteinuria, pyruria, or blood. Ultrasound of the


transplanted kidney shows a 8 x 8-cmfluid collection in the pelvis, with mild
caliectasis of the renal allograft.

The fluid collection is drained. Laboratory studies show the following:


Color: Straw yellow
Culture: No growth
Lactate dehydrogenase: 150 IU/L
Triglycerides: 168 mg/dl
Creatinine: 950umol/L

Which one of the following is the most likely diagnosis in this patient?

A Urine leak
B Liquefied hematoma
C Seroma
D Sterile abscess
E Lymphocele

Return to question 1 Return to contents


A nswer A - Urine leak

Options B, C, D and E are incorrect, option A is the correct answer. The fluid
characteristics are most consistent with a urine leak. Urinary leak or urinoma
is usually due to ureteral or bladder injury resulting in collection of the urine
outside the bladder. The frequency of this complication is approximately
0.8%9.3%, usually occurring in the first month following transplantation.
The most frequent causes include trauma to the ureter during dissection or
ischemia of the distal ureter resulting in necrosis or acute rejection. Urine
leaks can also occur at the ureteral-vesical anastomosis, particularly in patients
with diabetes mellitus that may have a friable and thin bladder mucosa. The
dislodgement of the ureteral stent in this case could have contributed to the
development the leak.
The increased level of creatinine in the fluid aspirate relative to the serum
creatinine is consistent with a urine leak. Bladder decompression with a
catheter is a critical initial step in the management of suspected leaks. The
catheter reduces intravesical pressure and in some cases may resolve the
leak completely. The diagnosis is confirmed by demonstration of contrast
on cystogram, antegrade nephrostogram, or radioactive tracer on nuclear
medicine scanning outside the collecting system and bladder. Definitive
treatment depends on the etiology and may often require surgical intervention.
Lymphoceles also cause pelvic fluid collections but show creatinine values
equal to that of the serum, as well as increased triglyceride values.
The creatinine level in seromas, liquefied hematomas, and sterile abscesses
is also equivalent to that of the serum, inconsistent with the findings in this
patient.
Surgical complications of kidney transplantation
Early
Bleeding
Wound infection
Graft thrombosis
Urinary leak
Lymphocoele
Late
Ureteric stenosis
Renal artery stenosis

Return to contents
Question 45 Q
A 68-year-old man with chronic renal failure secondary to type 2 diabetes
mellitus presents with hematemesis. Initial laboratory values indicate a
hematocrit of 23%, platelet count of 267,000/mm3, Blood urea 65, and
creatinine of 995 umol/l. A decision is made to transfuse the patient with
2 units of packed red blood cells and to arrange for upper endoscopy. In
addition, preparations are made for temporary access to initiate haemodialysis.

What other therapy would be most likely to minimize bleeding in this patient?

A Platelet transfusion
B IV Desmopressin
C IM Vitamin K
D IV Protamine
E IV Octreotide

Return to question 1 Return to contents


A nswer B - Desmopressin I.V.

Options A, C, D and E are incorrect, option B is the correct answer.


IV Desmopressin
To be able to recognize the bleeding diathesis of ESRD and understand its
management .This patient presents with acute bleeding and compromised
renal function. The renal dysfunction has a negative impact on platelet function.
Desmopressin appears to help with platelet function. As there is generally
an adequate platelet count, platelet transfusion does not have much impact.
Vitamin K is used to help reverse problems with the extrinsic clotting cascade.
Protamine is used to help reverse the effects of heparin, and octreotide is used
for variceal bleeding. Estrogen is also used occasionally for uremic bleeding.

Return to contents
Question 46 Q
A 54-year-old man with stage 3 CKD due to IgA nephropathy presents for
further evaluation of a 2-week history of persistent gross hematuria. He denies
flank pain, fever, or dysuria. He was diagnosed with IgA nephropathy at age
34. At that time, he presented with normal kidney function, dysmorphic
hematuria, and 1.6 g of urinary protein excretion per day. Kidney biopsy
showed IgA nephropathy with focal proliferative glomerulonephritis. He was
treated with enalapril, and the proteinuria declined and has remained stable at
approximately 0.4 g/day.
On physical examination, the blood pressure is 180/95 mmHg, increased
from 130/80 mmHg 4 months ago. A bruit is heard over the left kidney
and is present during both systole and diastole. The remainder of the
examination is normal.
Laboratory studies show a stable serum creatinine level of 102 umol/L,
haemoglobin level of 12.2 g/dl, and platelet count of 350,000/l. The urine
albumin:creatinine ratio is 53 mg/g.
Urinalysis shows trace protein and 2+ blood. Microscopic examination of the
urinary sediment shows too numerous to count red cells per high power field,
most of which have normal morphology.
A kidney ultrasound with colour Doppler is shown.
Remaining material is same with no change.
A kidney ultrasound with colour Doppler is shown.

Which one of the following is the most likely diagnosis in this patient?
A Renal cell carcinoma
B Renal infarction
C Arteriovenous fistula (AVF)
D Renal artery stenosis
E Simple renal cyst

Return to question 1 Return to contents


A nswer C - Arteriovenous fistula (AVF)

Options A, B, D and E are incorrect, option C is the correct answer. The most
likely diagnosis is an AVF that most likely developed as a consequence of the
previous kidney biopsy. Doppler ultrasound shows a blue-red flow pattern within
the hypoechoic area in the lower pole of the left kidney characteristic of AVF.
The frequency of AVF formation following kidney biopsy ranges from 0.4% to
18% but may be more common with renal transplant biopsies. Most AVFs are
asymptomatic, do not require intervention, and resolve spontaneously.
In some cases, however, they may become aneurysmal and present years later
with hematuria and hypertension as noted in this patient. High-output cardiac
failure, rupture, infection, mass effect, and impaired kidney function have also
been reported. An abdominal or flank bruit is an important diagnostic clue.
Computed tomography (CT) angiography and magnetic resonance angiography
serve as complimentary diagnostic studies. Digital subtraction angiography is
reserved for percutaneous intervention or for definitive anatomic evaluation prior
to surgery. Transcatheter arterial embolization or surgery is usually indicated for
symptomatic or enlarging lesions, particularly when symptoms are more severe.
The diagnosis in this case was confirmed angiography (see figure). This patient
was successfully treated with embolization.

Simple kidney cysts are thin walled, hypoechoic with


through transmission, and do not have color flow as
seen in this case. Renal cell carcinomas are typically solid
lesions and may also be seen as complex cystic lesions
on ultrasound. CT scanning with radiocontrast using cuts
<5 mm is recommended in patients with well preserved
kidney function to further evaluate complex cysts seen on
ultrasound.
Renal infarction can present with hematuria and
hypertension but would not present with a focal area of
increased flow within the renal parenchyma. Flank pain
is often a presenting complaint. Peripheral wedge-shaped hypoechoic lesions
with decreased rather than increased flow on Doppler ultrasound would be
more typical of kidney infarction. Renal artery stenosis may also present with
hypertension or an abdominal bruit, but not with hematuria unless there was
concurrent dissection and renal infarction. The turbulent flow shown on the
Doppler ultrasound in this case is not in the renal artery, but rather within the
renal parenchyma.

Return to contents
Questions 47 & 48 Q
Question 47
There is no benefit in treating asymptomatic bacteriuria except in?

A Diabetes
B Elderly
C Neurogenic bladder
D Pregnancy
E Catheterised patients

Question 48
KDIGO recommendations for management of AKI do not include

A Not using diuretics to prevent AKI


B Using diuretics to manage fluid overload in AKI
C Not using low dose dopamine to prevent or treat AKI
D Using Fenoldopam to prevent or treat AKI
E Not using atrial natriuretic peptide to prevent or treat AKI

Return to question 1 Return to contents


A nswer
47. D - Pregnancy
48. D - Using Fenoldopam

Answer 47
Options A, B, C and E are incorrect, option D is the correct answer.
Pregnancy
There is proven benefit in treating asymptomatic bacteriuria in pregnant
patients. There is 8% chance of asymptomatic UTI to become symptomatic
within 7 days.
No benefit of treatment shown in Diabetes, Elderly, Neurogenic bladder
and in catheterised patients.
In CKD patients alteration of dose is required to avoid toxicity
NICE Guideline suggest 5-10 days treatment for uncomplicated Lower UTI in
women with CKD.
(Smaill, Cochrane database 2007)

Answer 48
Options A, B, C and E are incorrect, option D is the correct answer.
Using Fenoldopam to prevent or treat AKI

KDIGO recommend not using fenoldopam to prevent or treat AKI


(Kidney International 2012;Vol 2:1-141)

Return to contents
Question 49 Q
A 50-year-old man with end-stage kidney disease secondary to diabetic
kidney disease maintained on peritoneal dialysis is seen in follow-up 5 days
after presenting with a third episode of peritonitis over the last 3 months. Two
months ago, he was treated for culture-negative peritonitis with intraperitoneal
cefazolin. One month ago, he was treated with intraperitoneal cefazolin for
oxacillin-sensitive Staphlococcal epidermidis peritonitis. On both occasions, the
patient admitted to touch contamination. Peritoneal fluid analysis 5 days ago
showed 804 white cells/mm3, with 70% neutrophils.
Gram stain was negative. The patient was started on empiric intraperitoneal
vancomycin and ceftazidime.
He currently complains of abdominal pain and persistently cloudy peritoneal
fluid. On physical examination, he appears ill. The blood pressure is 114/80
mmHg, pulse is 99 beats/min, and temperature is 38.3o C (100.9oF).
The abdomen shows diffuse tenderness with guarding, rebound tenderness,
and decreased bowel sounds. The exit site of the peritoneal dialysis catheter
shows no erythema or exudate. There is no induration or tenderness along the
catheter tunnel.
Peritoneal fluid analysis shows 6600 white cells/mm3, of which 80% are
neutrophils. Peritoneal dialysis fluid cultures show no growth at 5 days.

Which one of the following is the most appropriate next step in management
of this patient ?

A Remove peritoneal dialysis catheter


B Add intravenous aminoglycoside
C Add intraperitoneal amphotericin
D No change in therapy
E Ultrasound of the peritoneal catheter tunnel

Return to question 1 Return to contents


A nswer A - Remove peritoneal dialysis catheter

Options B, C, D and E are incorrect, option A is the correct answer. The patient
has refractory peritonitis, defined as failure of the effluent to clear after 5 days
of appropriate antibiotics, and removal of the dialysis catheter is indicated.
Causes of refractory peritonitis include antibiotic-resistant pathogens,
fungal peritonitis, tunnel infection, or atypical pathogens. Recent treatment
for two episodes of bacterial peritonitis significantly increases the risk for
fungal peritonitis, and the worsening clinical picture despite broad-spectrum
antibiotics should raise suspicion for this diagnosis.
Fungal peritonitis is often difficult to diagnose. Gram stain of peritoneal fluid
is positive in only 30% of cases, and culture may take several days to weeks to
show growth.
The International Society for Peritoneal Dialysis guideline on peritoneal
dialysisrelated infections recommends catheter removal in cases of refractory
peritonitis. In cases of fungal peritonitis, immediate catheter removal is
recommended after fungi are identified by microscopy or culture.
Empiric addition of an aminoglycoside antibiotic would be appropriate for
documented Pseudomonas infection, but this patient has negative cultures and
refractory peritonitis necessitating catheter removal.
Addition of antifungal therapy pending culture results, in addition to
catheter removal, is appropriate for this patient because there is high clinical
suspicion for fungal peritonitis. Therapy with oral fluconazole, or alternatively
with intravenous amphotericin B or an intravenous echinocandin such as
caspofungin (when there is significant prior exposure to azole antifungals),
is indicated. Use of intraperitoneal amphotericin is discouraged because
of associated chemical peritonitis, pain, peritoneal membrane injury, and
treatment failure.
It would not be appropriate to continue current therapy because the patient
meets criteria for refractory peritonitis. Moreover, given the high suspicion for
fungal peritonitis, this strategy places the patient at increased risk for significant
morbidity and mortality.
An ultrasound of the catheter can help assess whether a tunnel infection
is present. However, there are clear indications for catheter removal in this
patient, and results of this study would not change management.

References
Li PK, Szeto CC, Piraino B, Bernardini J, Figueiredo AE, Gupta A, Johnson DW, Kuijper EJ, Lye WC,
Salzer W, Schaefer F, Struijk DG; International Society for Peritoneal Dialysis: Peritoneal dialysis-
related infections recommendations: 2010 update. Perit Dial Int 30:393423, 2010

Return to contents
Question 50 Q
A 40-year-old male with alcoholic liver disease and portal hypertension was
admitted with confusion. He was taking spironolactone, thiamine, Vitamin B12
and propranolol. He was treated initially with intravenous fluids and lactulose.

Investigation on admission
Urea 4 mmol/L 2.5-7.0
Creatinine 80 umol/L 60-110

Investigations 3 days later


Urea 8.2 mmol/L 2.5-7.0
Creatinine 153 umol/L 60-110

Which of the following investigations favours a diagnosis of acute tubular


necrosis over hepatorenal syndrome (HRS)?

A High urine osmolarity


B Low urinary sodium
C Urinary protein of 300 mg/24 hours
D Normal renal ultrasound
E Granular cast and renal epithelial cells seen on urine microscopy

Return to question 1 Return to contents


A nswer E - Granular cast and renal epithelial cells
seen on urine microscopy

Options A, B, C and D are incorrect, option E is the correct answer. Granular


cast can be found on both acute tubular necrosis and HRS but renal epithelial
cells are more suggestive of ATN.
Renal ultrasound is useful for excluding an obstructed urinary system and
parenchymal disease but could be seen in both ATN and HRS.
Low urinary sodium concentration and high urinary osmolarity are consisted
with HRS as renal tubular integrity is well maintained in contrast to ATN.
This patient is remained on diuretics therefore urinary electrolytes are
uninterpretable. Diuretics must be stopped for atleast 2 days before the
diagnosis of HRS is made.
Hypovolaemia or septic shock is often precede ATN, where as HRS occurs with
severe alcoholic hepatitis or in a patient with advanced cirrhosis following a
septic insult like SBP (subacute bacterial peritonitis).

Main causes of renal dysfunction in Cirrhosis of Liver

Hepatorenal syndrome
Type 1
Type 2
Nephrotoxic drugs
NSAID
Aminoglycosides
Contrast agents
Hypovolaemia induced
Gastrointestinal bleeding
Diuretics
Diarrhoea
Parenchymal renal disease
Acute tubular necrosis
Glomerulonephritis (Hepatitis B, Hepatitis C)
IgA Nephropathy

Return to contents
Question 51 Q
A 51-year-old man presents with marked peripheral oedema .
His UPCR (Urine protein Creatinine ration) 900
Serum Albumin 27 g/L
eGFR of 85 ml/min.

Renal biopsy is done

Look at the image

Which one of the following is least likely to be associated with this disease?

A Gold therapy
B Graft versus host disease
C Heavy metal ingestion
D Hodgkins disease
E Diclofenac use

Return to question 1 Return to contents


A nswer A - Gold therapy

Options B, C, D and E are incorrect, option A is the correct answer. Electron


microscopy shows diffuse effacement of foot processes consistent with
diagnosis of Minimal Change Disease.

Gold therapy is associated with membranous Glomerulonephritis and not


Minimal change disease.

Kidney biopsy Minimal Change Disease

Light microscopy Immunofluorescence Electron


Microscopy
Minimal Normal Negative Diffuse epithelial
Change Disease foot process
effacement.

Causes of Minimal Change Disease

Idiopathic 87%

Secondary 13%
Drugs NSAID
Alpha Interferon
Lithium
Gold (Usually Membranous)
Allergy 1. Pollens
2. House dust
3. Immunisations
4. Insect stings
Malignancy 1. Hodgkins
2. Mycosis fungoides
3. CLL (but usually MPGN)

Return to contents 118


A
Differential Diagnosis of Minimal Change Disease
Disease Differentiating Features
IgM nephropathy Mesangial IgM
IgA nephropathy Mesangial IgA, C3, kappa, lambda

C1q nephropathy Mesangial C1q; less intense IgG, IgM, C3

Unsampled FSGS Focal interstitial fibrosis and tubular atrophy


Segmental IgM or C3

KDIGO Guideline for Treatment of Idiopathic Minimal change disease in Adults

Drug and dosing scheme

Initial treatment
Prednisone
Daily single dose of 1 mg/kg (maximum 80 mg) or alternate-day single
dose of 2 mg/kg (maximum 120 mg) until complete remission (minimum
4 weeks to a maximum of 16 weeks)after complete remission, tapered
slowly over 6 months.
FR (frequent relapsing) Remission in >90% of patients or SD (steroid
dependent) MCD (minimal change disease)
1. Cyclophosphamide (oral) single course 22.5 mg/kg/d as tolerated for 8
weeks.
2. Relapsed despite cyclophosphamide, or patients of childbearing age a
Cyclosporine starting dose 35 mg/kg/d (in two equally divided doses)
b. Tacrolimus 0.050.1 mg/kg/d (in two equally divided doses)
Following 3 months of stable remission, tapered to reach the minimum
dosage that maintains remission, for 12 years
3. Intolerant to corticosteroids, cyclophosphamide, and/or CNIs
a. Mycophenolate mofetil 5001000 mg twice daily for 12 years

Return to question 1 119 Return to contents


A nswer

Indications for second line drugs in treatment of Adult MCD


(Cyclophosphamide, Cyclosporine, Tacrolimus, Mycophenolate mofetil)

Use in <10% of patients who failed to achieve remission with prednisolone


alone. Needs to individualise and involve patients in decision

Frequency of relapses
Steroid denpendence
Contraindications for use of steroids (Diabetes mellitus, obesity,
Psychiatric conditions, severe osteoporosis)
Fertility concerns

3rd Line treatment not included in KDIGO guidelines

Rituximab- No RCT (Randomised controlled trials),case reports


encouraging
Levamisole- high relapse rate after stopping(as in children)
May be worth a try when all else has failed.

Return to contents
Question 52 Q
Which one of the following statements is incorrect with regards to duration of
treatment in Uncomplicated UTI?

A In uncomplicated lower UTI, 3 day treatment is as effective as 5 or


10 days treatment
B Single dose treatment is less effective in uncomplicated lower UTI
C Treatment duration can vary from 4-20 days in uncomplicated
pyelonephritis
D In chronic kidney disease, alteration of dose is needed to avoid toxicity
E There is a less than 1% chance of symptomatic UTI within 7 days of having
asymptomatic bacteriuria

Return to question 1 Return to contents


A nswer E

Options A, B, C and D are incorrect, option E is the correct answer.


There is a less than 1% chance of symptomatic UTI within 7 days of having
asymptomatic bacteriuria.

There is an 8% chance of symptomatic UTI within 7 days of having


asymptomatic bacteriuria (Smaill, Cochrane database).

Treatment of Uncomplicated UTI


Duration 3 days therapy is as effective as 5-10 days treatment
First Line Antibiotics

1. Trimethoprim 200 mg bd or cotrimoxazole 960 mg bd


(Nice Guidelines)
Longer course 7-10 days therapy
1. Nitrofurantoin 100 mg bd (not in renal impairment)
Second line drugs

1. Fluoroquinolones such as ciprofloxacin 500 mg bd or levofloxacin


250 mg bd
2. Fosfomycin 3 g single dose

(Avoid ampicillin not effective in eradicating vaginal and periuretheral colonization)

Encourage Fluid intake>2 L/day



Microbiology of UTI (Farrell. J Infect. 46: 94, 2003)

Organisms Community Hospital acquired Pyelonephritis


acquired
E Coli 77% 56% 69%
Proteus 4% 6% 3%
Klebsiella sp 4% 7% 9%
Enterococcus 4% 9% 6%
Pseudomonas 2% 4% -
S Saprophyticus 4% - -
Staph aureus - 2.5% -

Return to contents
Questions 53 & 54 Q
Question 53
KDIGO guideline suggestions for management of AKI do not include?

A Using protocol based management of haemodynamic and oxygenation


parameters to prevent development or worsening of AKI in high-risk patients
B Insulin therapy targeting plasma glucose 6.1-8.3 mmol/l in critically ill
C Achieving a total energy intake of 20-30 kcal/kg/d in patients with AKI
D To avoid protein restriction with the aim of preventing or delaying RRT
E Providing nutrition preferentially via parenteral route

Question 54
A 65-year-old male presented with nephrotic syndrome receives steroid
therapy without benefit. His investigations show an albumin of 20 g/L , Total
cholesterol of 13 mmol/l, dipstick urinalysis reveals ++++ protein and a renal
biopsy shows Idiopathic focal segmental glomerulosclerosis.

Which one of the following is most likely to preserve renal function in this
patient?

A Ramipril
B Dietary salt restriction
C Mycophenolate mofetil
D Low dietary protein intake
E Atorvastatin

Return to question 1 Return to contents


A nswer
53. E - providing nutrition
preferentially via parenteral route
54. A - Ramipril

Answer 53
Options A, B, C and D are incorrect, option E is the correct answer. KDIGO
suggest providing nutrition preferentially via enteral route
(Kidney International 2012; Vol 2:1-141)
Answer 54
Options B, C, D and E are incorrect, option A is the correct answer.
Approximately 50% of subjects with FSGS do not respond to steroid therapy
but ACE inhibitors with a target BP <130/80 is a recognized strategy to
slow the progression of renal disease. This patient is clearly at high risk of
cardiovascular disease with a very high cholesterol .
KDIGO Guidelines for Treatment of Idiopathic FSGS

FSGS with non-nephrotic proteinuria


ACE/ARB ONLY

FSGS with nephrotic proteinuria


prednisone-at a daily single dose of 1 mg/kg (maximum 80mg) or
alternate-day dose of 2 mg/kg given for minimum of 4 weeks;
Continue high-dose corticosteroids up to a maximum of 16 weeks, as
weeks, as tolerated, or until complete re mission
Corticosteroids should be tapered slowly over a period of 6 months after
achieving complete remission

FSGS with nephrotic proteinuria


CNIs be considered as first-line therapy for patients with relative
contraindications
Intolerance to high-dose corticosteroids

Steroid-resistant FSGS
cydosporine at 3-5 mgl/kg/d in divided doses to be given for at least
4-6 months
If there is a partial or complete remission. continue cyclosporine
treatment for at least 12 months. followed by a slow taper.
Those who do not tolerate cydosporine. be treated with a combination
of mycophenolate mofetil and highdose dexamethasone

Return to contents
Questions 55 & 56 Q
Question 55
A 77-year-old woman presents with sudden onset atrial fibrillation (ventricular
rate of 170/minute). Her serum creatinine concentration was 258umol with
eGFR 24 ml/min.

What is the main factor that determines the choice of loading dose of digoxin
in this patient?

A Lipid solubility
B Plasma half-life
C Renal excretion
D Volume of distribution
E Absorption

Question 56
A 42-year-old man comes to follow up clinic for a kidney biopsy result
His Blood pressure was 138/82

His recent 24 hr urine protein is 1.3g/day and serum creatinine was 80umol/l.

Kidney biopsy revealed IgA Nephropathy with no background kidney damage.

What is the target Blood pressure in this patient to slow down the progression
of his kidney disease?

A 125/75
B 125/80
C 130/80
D 135/80
E 140/80

Return to question 1 Return to contents


A nswer
55. C - Renal excretion
56. A - 125/75

Answer 55
Options A, B, D and E are incorrect, option C is the correct answer.
Digoxin is cleared by the kidneys so the maintenance dose would require
adjustment in renal failure.
The pharmacokinetics of digoxin is complex and best explained by a two
Compartment model. The loading dose is mainly dependent on the Volume of
Distribution of a drug but this patient has moderate renal failure. The loading
dose is calculated (using various models) by taking into account age, Creatinine
clearance, body surface area etc. Volume of distribution becomes important
particularly when body weight is 40kg or less. On balance it is the renal failure
that is the most important factor in this patient in determining the loading dose.

Answer 56
Options B, C, D and E are incorrect, option A is the correct answer. Total
number of glomeruli seen on the biopsy is not an independent predictor of
clinical outcome.

KDIGO Guidelines Suggest

Target Blood pressure if proteinuria<1g/24hr 130/80


Target Blood pressure if Proteinuria>1g/24 hrs125/75
Achieve with salt restriction and RAS (Renin angiotensin system) blockade

Hypertension and antihypertensive agents in non Diabetic


Chronic Kidney Disease
Additionol Agonts to Roduco
Preferred CVD Risk and Reach Target
Clinical Assessment Target Blood Agents for CKD Blood Pressure

Blood pressure >- 130/80 mm Hg and spot urine <130/80 A ACE inhibitor A Diuretic preferred, then beta A
total protein-to-creatinine ratio >- :200 mg/g mm Hg or ARB blocker or calcium-channel blocker

Blood pressure ->130/80 mm Hg and spot urine <130/80 B None preferred Diurelic preferred, then ACE A
total protein-to-crealinine ratio <200 mglg mm Hg inhibitor, ARB, beta-blocker or
calcium-channel blocker
Blood pressure <130/80 mm Hg and spot urine ACE inhibitor C Diuretic preferred, then beta- A
total protein-to-creatinine ratio <- 200 mglg or ARB blocker or calcium-channel blocker

Blood pressure <130/00 mm Hg and spot urine None preferred


total protein-to-creatinine ratio <200 mg/g

Return to contents
Question 57 Q
Which one of the following statements is true regarding use of corticosteroid
for IgA nephropathy as per KDIGO guidelines?

A Patient should receive a 6 month course of corticosteroid with persistent


proteinuria>1g/24hr, despite 12 month of optimized supportive care
(including Acei or ARB and blood pressure control)and GFR<25ml/min

B Patient should receive a 9 month course of corticosteroid with persistent


proteinuria>1g/24hr, despite 12 month of optimized supportive care
(including Acei or ARB and blood pressure control)and GFR<35ml/min

C Patient should receive a 8 month course of corticosteroid with persistent


proteinuria>1g/24hr, despite 12 month of optimized supportive care
(including Acei or ARB and blood pressure control)and GFR<40ml/min

D Patient should receive a 3 month course of corticosteroid with persistent


proteinuria>1g/24hr, despite 12 month of optimized supportive care
(including Acei or ARB and blood pressure control)and GFR<50ml/min

E Patient should receive a 6 month course of corticosteroid with persistent


proteinuria>1g/24hr, despite 3-6 month of optimized supportive care
(including Acei or ARB and blood pressure control)and GFR>50ml/min

Return to question 1 Return to contents


A nswer E - 6 month course of corticosteroid

Options A, B, C and D are incorrect, option E is the correct answer. Patient


should receive a 6 month course of corticosteroid with persistent
proteinuria>1g/24hr, despite 3-6 month of optimized supportive care

KDIGO guideline suggest

Patient should receive a 6 month course of corticosteroid with persistent


proteinuria>1g/24hr, despite 3-6 month of optimized supportive care
(including Acei or ARB and blood pressure control) and GFR>50ml/min.

KDIGO Guidelines for IgA Nephropathy

IgAN- Proteinuria is IgAN -Proteinuria is more Crescentic IgAN


more than 1 g/d than 1 g/d despite 3-6 Steroids and
long-tern ACE-1 or months ARB/ACE and BP 125/75
cyclophosphamide
and GFR more than 50ml/min
ARB 6-months course of analogous to the
Blood pressure corticosteroid therapy on treatment of ANCA
target- 125/75 top of ACE/ARB vasculitis

Return to contents
Question 58 Q
Which one of the following blood work up is not recommended for CKD stage
5D care in a Haemodialysis unit?

A For serum calcium and phosphorus, every 13 months


B PTH every 36 months.
C For alkaline phosphatase activity, every 3 months
D 25(OH) D (calcidiol) levels might be measured if needed.
E FGF23, pro collagen type I, C-terminal pro peptide-yearly

Return to question 1 Return to contents


A nswer E - FGF23, pro collagen type I, C - terminal
pro peptide-yearly

Options A, B, C and D are incorrect, option E is the correct answer.


KDIGO Guidelines

Serum calcium should be maintained between 2.2 and 2.5 mmol/L, with
avoidance of hypercalcaemic episodes.
Serum phosphate in patients with CKD stage 3b-5 should be maintained
between 0.9 and 1.5 mmol/L.
Serum phosphate in dialysis patients (stage 5D) between 1.1 and 1.7 mmol/L
CKD stages 3b-5 not on dialysis therapy in whom serum PTH -higher than the
upper reference limit for the assay, despite correction of modifiable factors
Stage 5D- target range for parathyroid hormone measured using an intact PTH
assay should be between 2 and 9 times the upper limit of normal for the assay
PTH range corresponds to approximately 130-600pg/mL

Bone-specific alkaline phosphatase
can be used to evaluate bone
markedly high or low values predict underlying bone turnover

Table showing suggested frequency of biochemical testing in CKD-MBD

CKD Calcium Phosphorus PTH Alkaline Calcidol


Stage Phosphatase 25(OH)D

3b Every 6-12 Every 6-12 Baseline Every 6-12 Baseline


Progressive Months Months months

4 Every 3-6 Every 3-6 Every Every 3-6 Baseline


months months 6-12 months
months
5 Every 1-3 Every 1-3 Every 3-6 Every 1-3 Baseline
months months months months

5D Every 1-3 Every 1-3 Every 3-6 Every 1-3 Baseline


months months months months

Return to contents
Question 59 Q
A 67-year-old man was referred to the nephrologists after he was noted to be
oliguric following an elective coronary angiography. He had a past medical
history of hypertension, for which he was taking ramipril. On examination he
was euvolemic and had mottled skin changes on his legs.

Labs shows
serum sodium 134 mmol/L (137144)
serum potassium 5.5 mmol/L (3.54.9)
serum urea 33.3 mmol/L (2.57.0)
serum creatinine 356 mol/L (60110)
creatine kinase 460 U/L (24170)

haemoglobin 11.5 (11.516.5)


white cell count 12.6 109/L (411)
neutrophil count 9.3 109/L (1.57.0)
lymphocyte count 2.5 109/L (1.54.0)
eosinophil count 1.3 109/L (0.040.40)

Kidney biopsy is done. Look at the Image

Which is the most likely diagnosis in this patient clinical scenario?

A Myeloma cast Nephropathy


B Acute Phosphate Crystals deposition
C Acute tubular necrosis
D Cholesterol emboli
E Acute interstitial nephritis

Return to question 1 Return to contents


A nswer D - Cholesterol emboli

Options A, B, C and E are incorrect, option D is the correct answer. Kidney


Biopsy showing characteristic cholesterol cleft of cholesterol emboli.
Cholesterol embolization suggested with:
Presence of embolic lesions (e.g. toes)
Livedo reticularis
Eosinophilia
AKI delayed days to weeks after procedure (over 3- 8
weeks)
AKI not resolving
Hypocomplementemia. (LOW C3 & C4)

Predisposing factors for atheroembolic Renal Disease


Atherosclerosis
1. systemic hypertension
2. cigarette smoking
3. hypercholesterolemia
4. diabetes mellitus
5. age >55 yr
6. white race

Trauma to atheromatous plaque


7. invasive vascular procedure involving the aorta proximal to origin of
renal arteries
8. thrombolytic therapy
9. blunt trauma

Prevention of healing of the eroded plaque


Anticoagulant therapy
Thrombolytics

Treatment
No specific
Control of CV risk factors (BP and cholesterol)
Overall renal prognosis is poor
Differential Diagnosis- Contrast induced Nephropathy & Ischaemic AKI
1. In contrast nephropathy, the renal failure occurs within 48 to 72 hr after
the dye infusion and generally resolves within 4 to 7 days.
2. Ischemic acute renal failure is diagnosed by its immediate onset, its
association with hypotension, and the lack of systemic manifestations such
as skin rash, eosinophilia, and hypocomplementemia.

Return to contents
Question 60 Q
Compared to general population which one of the following malignancy occurs
after kidney transplantation with the highest relative risk compared to general
population?

A Nonmelanoma skin cancer


B Lymphoma
C Kaposi sarcoma
D Lung cancer
E Cervical cancer

Return to question 1 Return to contents


A nswer C - Kaposi sarcoma

Options A, B, D and E are incorrect, option C is the correct answer. There


is now consistent evidence that shows renal transplant recipients are
approximately three times more likely to develop cancers than the general
population. This excess risk is type specific and greatest for Kaposi's sarcoma
(200 times increased risk) and nonmelanocytic and melanocytic skin cancer
(nine to 20 times increased risk) as compared to general population.
Kasiske BL, Snyder JJ, Gilbertson DT, Wang C: Cancer after kidney transplantation in the United
States. Am J Transplant4 :905 913,2004

Cancers that are increased in Kidney Transplant Patients compared to


general population

Much higher Higher Not different


Skin-HPV (Human papilloma virus) uterine, cervix Breast
Kaposi- HHV6 (Human herpes virus) Oesophagus Testicular
Vulvovaginal Liver Ovarian

Lymphoma-EBV (Ebstein bar virus Lung


Kidney Colon

Clinical Presentation of Post Transplant Lymphoproliferative


Disorder associated with Epstein-Barr Virus

Unexplained Fever
Mononucleosis like syndrome
( Fever, malaise, pharyngitis,tonsillitis)
Gastrointestinal bleeding,obstruction, perforation
Abdominal mass lesion
Infiltrative disease of allograft
Hepatocellular or pancreatic dysfunction
Central nervous system disease

Return to contents
Question 61 Q
72-year-old female was referred with diagnosis of 6 gram proteinuria/day
Patient seen on Wednesday, admitted on Friday for kidney biopsy but she
developed pulmonary embolism on Saturday morning.
Investigations showed antiPLA2R negative but positive lupus serology.
Clinically she has no features of SLE.

Kidney biopsy done after 4 months of anticoagulation confirms membranous


Nephropathy (Class V Lupus)

Which is the best treatment option for her in long-term maintenance phase?

A Rituximab
B Corticosteroid alone
C Mycophenolate mofetil (MMF)+Low dose Prednisolone
D RAAS(Renin angiotensin aldosterone blockade) Blockade only
E Calcineurin inhibitor (CNI) with or without corticosteroid

Return to question 1 Return to contents


A nswer
C - Mycophenolate mofetil
(MMF)+Low dose Prednisolone

Options A, B, D and E are incorrect, option C is the correct answer.

International Society of Nephrology/Renal Pathology Society 2004


Classification of Lupus Nephritis
Designation Description Characteristic Clinical
Feature
Class I: minimal mesangial No LM abnormalities; isolated mesangial IC deposits on IF Normal urine or
lupus nephritis and/or EM microscopic hematuria
Class II: mesangial Mesangial hypercellularity or matrix expansion with mesangial Microscopic hematuria and/
proliferative lupus nephritis IC deposits on IF and/or EM or low-grade proteinuria
Class III: focal lupus <50% of glomeruli on LM display segmental (<50% Nephritic urine sediment
nephritis* of glomerular tuft) or global (>50% of glomerular tuft) and subnephrotic
endocapillary and/or extracapillary proliferation or sclerosis; proteinuria
mesangial and focal subendothelial IC deposits on IF and EM

Class IV: diffuse lupus 50% of glomeruli on LM display endocapillary and/or Nephritic and nephrotic
nephritis* extracapillary proliferation or sclerosis; class IV-S denotes syndromes, hypertension,
50% of affected glomeruli have segmental lesions; class reduced kidney function
IV-G denotes 50% of affected glomeruli have global lesions;
mesangial and diffuse subendothelial IC deposits on IF and EM
Class V: membranous Diffuse thickening of the glomerular capillary walls on LM Nephrotic syndrome
lupus nephritis with subepithelial IC deposits on IF and EM, with or without
mesangial IC deposits

Class VI: advanced >90% of glomeruli on LM are globally sclerosed with no Markedly reduced kidney
sclerosing lupus nephritis residual activity function, hypertension

Treatment of Lupus Nephritis, Stratified by ISN classification and Phase of Treatment


Class Induction Phase Maintenance Phase
Class Conservative, nonimmunomodulatory therapy Conservative, nonimmunomodulatory therapy (e.g., RAAS
I* (e.g., RAAS blockade) blockade)
Class Mesangial hypercellularity or matrix expansion Microscopic hematuria and/
II with mesangial IC deposits on IF and/or EM or low-grade proteinuria
Class Pulse IV steroids followed by tapering doses of Lowest tolerable amount of oral steroids
III* oral steroids
Class And IV cyclophosphamide 0.75-1.0 g/m2 IV And MMF 2000 mg/day for 6 mo, then 1500 mg/day for 3-6
IV monthly for 6 doses or IV cyclophosphamide 500 mo, then 1000 mg/day afterward assuming stable disease or
mg IV every 2 wk for 6 doses or MMF 2000-3000 Azathioprine 2.0 mg/kg/day for 6 mo, then 1.5 mg/kg/day for
mg/day for 6 mo 3-6 mo, then 1.0 mg/kg/day afterward assuming stable disease
Class Pulse IV steroids followed by tapering doses of NLowest tolerable amount of oral steroids and MMF 2000 mg/
V oral steroids and IV cyclophosphamide 0.75-1.0 g/ day for 6 mo, then 1500 mg/day for 3-6 mo, then 1000 mg/
m2 IV monthly for 6 doses or Cyclosporine (dose day afterward assuming stable disease or Azathioprine 2.0 mg/
adjusted to goal trough level 125-200 mcg/L) or kg/day for 6 mo, then 1.5 mg/kg/day for 3-6 mo, then 1.0
Tacrolimus (dose adjusted to goal trough level mg/kg/day afterward assuming stable disease
5-10 mcg/L) or MMF 2000-3000 mg/day for 6 mo
Class Conservative, nonimmunomodulatory therapy Not applicable
VI (e.g., RAAS blockade) with preparation for kidney
replacement therapy

Return to contents 136


Question 62 Q
Each nephron is about 3-4 cm, and the length of collecting duct is 2cm.
Which one of the segment of nephron is impermeable to water?

A Collecting duct
B Ascending limb
C Proximal tubule
D Distal tubule
E Descending limb

Return to question 1 Return to contents


A nswer B - Ascending limb

Options A, C ,D and E are incorrect, option B is the correct answer.


The ascending limb of the loop of Henle is a direct continuation from the
descending limb of loop of Henle, and one of the structures in the nephron of
the kidney. The ascending limb has a thin and a thick segment. The ascending
limb drains urine into the distal convoluted tubule.

The thin ascending limb is found in the medulla of the kidney, and the thick
ascending limb can be divided into a part that is in the renal medulla and a
part that is in the renal cortex.

Ascending limb is impermeable to water and ions, except sodium and chloride
which cross by diffusion.

Organization of the nephron

Structures are noted as follows:


1, glomerulus
2, proximal convoluted tubule
3, proximal straight tubule
4, descending thin limb
5, ascending thin limb
6, TAL
7, macula densa
8, distal convoluted tubule
9, connecting tubule
10, cortical collecting duct
11, outer medullary collecting duct
12, inner medullary collecting duct

Return to contents
Question 63 Q
Which one of the following statement is incorrect about collapsing variant of
Focal segmental glomerulosclerosis?

A Associated with HIV and parvovirus infection and pamidronate and IFN
treatment
B May be idiopathic
C Much more common in AfroCaribbeans
D Typically present as mild nephrotic syndrome
E Biopsy Changes may look similar to crescents

Return to question 1 Return to contents


A nswer
D - Typically present as mild
nephrotic syndrome

Options A, B, C, E are correct but not option D.


Collapsing FSGS is typically associated with severe nephrotic syndrome

Light microscopy Immunofluorescence Electron


Microscopy

Collapsing FSGS. Collapsed Negative or non- Negative or


(Collapsing glomerular capillary specific C3 and IgM non-specific
glomerulopathy). loops surrounded in glomeruli. deposits.
by hypertrophied
podocytes.

Collapsing variant FSGS


Causes
Infection-HIV(HIVAN Nephropathy),Parovirus
Drugs-Pamidronate, alpha interferon, Sirolimus
and Heroin

Electron microscopy
Endothelial
tubuloreticular
inclusions in a
specimen with
collapsing FSGS
increase the likelihood
of HIV associated
FSGS

Light Microscopy-Collapsing FSGS

Return to contents
Question 64 Q
A 59-year-old man with stage 3 CKD receiving therapy with sunitinib for stage
IV renal cell carcinoma is seen for new onset hypertension. He denies shortness
of breath, leg edema, headache, or focal neurologic symptoms.
On physical examination, he is alert and oriented. The blood pressure is 160/94
mmHg, which is increased from 130/80 mmHg 1 month ago. Fundoscopic
examination shows no papilledema. There is trace pretibial edema bilaterally.
There are no neurologic deficits.

Laboratory studies show a serum creatinine level of 160 umol/L (stable from
previous measurements), Modification of Diet in Renal Disease eGFR of 54
ml/min per 1.73 m2, white cell count of 10,000/l, haemoglobin level of 11
g/dl, and platelet count of 300,000/l. Peripheral blood smear shows no
schistocytes.

The urine protein:creatinine ratio is 1200 mg/g, increased from 200 mg/g one
month ago.

The urinalysis shows 1+ protein and no blood. Microscopic examination of the


urine sediment shows no casts or cells.

Which one of the following is the most appropriate next step in the
management of this patient?

A Abdominal magnetic resonance venography
B Begin a non-dihydropyridine calcium antagonist
C Discontinue sunitinib
D Renal artery duplex doppler ultrasonography
E Start an angiotensin converting enzyme inhibitor

Return to question 1 Return to contents


A nswer
E - Start an angiotensin converting
enzyme inhibitor

Options A, B, C and D are incorrect, option E is the correct answer.


Start an angiotensin converting enzyme inhibitor

Sunitinib (marketed as Sutent by Pfizer, and previously known as SU11248) is an


oral, small-molecule, multi-targeted receptor tyrosine kinase (RTK) inhibitor that
was approved by the FDA for the treatment of renal cell carcinoma (RCC) and
imatinib-resistant gastrointestinal stromal tumor.
Hypertension is a common complication of antiangiogenic therapy, and onset
of hypertension in the absence of significant end-organ injury generally does
not require discontinuation of antiangiogenic therapy.
The National Cancer Institute Angiogenesis Task Force recommends treatment
to a target blood pressure of <140/90 mmHg or treatment to maintain
the diastolic blood pressure <20 mmHg above baseline for patients with
low normal blood pressure. The preferred class of medications for treating
hypertension in this setting has yet to be established.
ACE inhibitors or angiotensin receptor blockers, particularly in patients with
proteinuria, or dihydropyridine calcium antagonists are reasonable choices.
Nondihydropyridine calcium antagonists are generally avoided because they
inhibit CYP3A4 and may lead to decreased metabolism of sunitinib.
Evidence suggests that onset of hypertension is a biomarker for improved
tumor response.

Indications to discontinue antiangiogenic therapy include reversible posterior


leukencephalopathy syndrome and onset of thrombotic microangiopathy,
which are not present in this patient.

Arterial and venous thromboembolism has been reported in association with


antiangiogenic therapy, most commonly with bevacizumab and aflibercept.
Arterial and venous thrombosis of the renal vasculature could cause
hypertension, but the absence of flank pain, gross hematuria, and other urinary
sediment abnormalities make these diagnoses less likely. Therefore, imaging
with abdominal magnetic resonance venography or renal artery duplex doppler
ultrasonography would not be indicated.

Return to contents
Question 65 Q
A 51-year-old man known case of chronic kidney disease stage 4 admitted with
right lower abdominal pain

Urine dipstick 1+blood,1+protein, Leucocyte ve

Serum creatinine 356 umol/L (baseline is 340)

CT abdomen is requested

Look at the image above.

What is the most likely cause of his loin pain?

A Cyst infection
B Cyst torsion
C Cyst rupture
D Cyst haemorrhage
E UTI

Return to question 1 Return to contents


A nswer D - Cyst haemorrhage

Options A, B, C and E are incorrect, option D is the correct answer.


Patient has CKD secondary to ADPKD, picture shows multiple cysts in liver.

Star pointed towards increase density of cyst which indicate cyst haemorrhage,
arrow indicates cyst compressing inferior vena cava (IVC).

ADPKD is the 4rth most common cause of ESRD in UK. 10% of prevalent ESRD
in UK. 100% suffers renal cyst, 80% suffers liver cysts.

Liver cysts develop later than renal cysts

Liver function usually normal (alkphos and GGT can increase to 5x normal)

20% prevalence at age 30

70-80% prevalence at age 70 then plateau

Women more prone to severe liver cysts than men (prevalence equivalent)
Oestrogen therapy risk factor
Number of pregnancies a risk factor
Severity of renal disease a risk factor

No data to suggest a genetic cause for massive PLD (polycystic liver disease)

Familial aggregation of massive PLD is rare

Return to contents
Question 66 Q
Which one of the following is a relatively more accurate parameter to monitor
renal function in patients with diabetes who have co existent liver cirrhosis?

A Urinary C peptide
B Albumin Creatinine ratio
C Serum Creatinine
D Serum cystatin C
E Serum chromogranin B

Return to question 1 Return to contents


A nswer D - Serum Cystatin C

Options A, B, C and E are incorrect, option D is the correct answer. Cystatin


C measurement may be helpful in clinical circumstances where Creatinine
measurement may not be appropriate such as
Patient with liver cirrhosis
Morbid obesity
Malnourished patient with small muscle mass

Cystatin C (Serum)

Reference range- Male: 0.56-0.98mg/L, Female: 0.52-0.90mg/L

Cystatin C is a 120 aminoacid, basic protein secreted by all nucleated cells


and removed by glomerular filtration. It can be used to estimate GFR with
less effect of age, race and muscle mass than Creatinine but it is affected by
body composition, cancer and various conditions.

Increased:
Acute or chronic renal failure, HIV, increased BMI, high C-reactive
protein, hyperthyroidism, corticosteroid use
Decreased:
Atherosclerotic vascular disease, cyclosporine treatment

Return to contents
Question 67 Q
A 50-year-old kidney transplant patient admitted with a acute rise in serum
Creatinine.

Graft (Transplant kidney) biopsy done

Look at the image

Which one of the following is the most likely diagnosis on Graft Biopsy?

A Severe tubulitis.
B BK virus inclusions.
C Cytomegalovirus inclusions.
D Arteritis
E Oxalate crystal deposits

Return to question 1 Return to contents


A nswer A Severe tubulitis

Options B, C ,D and E are incorrect, option A is the correct answer.


Severe tubulitis indicative of severe acute rejection
The Banff Classification
No rejection < 10% interstitial inflammation Borderline Rejection, any
inflammation with mild tubulitis.
Acute Rejection
Grade I: Tubulointerstitial Rejection = Moderate interstitial
inflammation and moderate tubulitis
Grade II: Vascular rejection = Endotheliitis
Grade III: Severe Vascular Rejection = Transmural inflammation
and fibrinoid necrosis

The Banff 97 Working Classification of Renal Allograft Pathology, Kidney International, Vol 55, 1999,
pp 713-723

Kidney Biopsy Features

Light microscopy Immunofluorescence Electron


Microscopy

Allograft Moderate interstitial Peritubular capillary Not applicable


rejection cellular inflammation and moderate staining for C4d is
tubulitis negative

Allograft Peritubular capillary Peritubular capillary Peritubular


rejection margination of inflammatory staining for C4d is capillary
antibody cells, thrombotic positive. lamellations.
mediated microangiopathy, glomerular
and arteriolar thrombi.

Return to contents
Question 68 Q
A 42-year-old man known case of ESRD secondary to IgA Nephropathy
received a live related kidney transplant from his brother 4 years ago;
attended an accident and emergency with a 2 day history of skin rash.
His immunosuppressive medications are Tacrolimus, Prednisolone and
Mycophenolate mofetil.
His eGFR is 70 ml/min stable

See the image

Which one of the following is the most likely aetiology of skin rash in
this patient?

A Chicken pox
B HHV6 infection
C HHV8
D Molluscum contagiosum
E Fungal infection

Return to question 1 Return to contents


A nswer C - HHV8

Options A, B, D and E are incorrect, option C is the correct answer. Above


Image showing Kaposi sarcoma caused by HHV8 (Human Herpes Virus).
The erythematous to violaceous cutaneous lesions seen in Kaposi sarcoma
have several morphologies: macular, patch, plaque, nodular, and exophytic.

Classic Kaposi sarcoma tends to be indolent, presenting with erythematous or


violaceous patches on the lower extremities (see image))

Common Neoplastic disorders Post kidney Transplant

1. Lymphoma (Posttransplant Lymphoproliferative Disease)


associated with Epstein-Barr virus
2. Non melanomatous Skin Cancer
Increased incidence with time post transplantation
Relatively high ratio of squamous to basal cell carcinoma
3. Other Solid Tumours

Teaching point in management

Diagnosis is by skin biopsy


1. Kaposi Sarcoma is one of the post transplant cancer in which switching
immunosuppression to SRL (sirolimus) is clearly beneficial

2. Switch CNI (Tacrolimus or cyclosporine to Sirolimus and decrease


Azathioprine dose).

3. Sirolimus inhibits the progression of dermal Kaposi sarcoma by


decreasing VEGF signalling

Return to contents
Question 69 Q
A 70-year-old retired politician was reviewed in the diabetes clinic on a
routine follow up visit. He was diagnosed as having type 2 diabetes about
8 years earlier and was taking metformin and gliclazide tablets. He was
known patient with hypertension, currently on ramipril and amlodipine
tablets. On examination he had a BMI of 31 kg/m2, with evidence of central
adiposity. There was evidence of bilateral background diabetic retinopathy on
ophthalmological examination.

Investigations

Serum urea Serum urea 9.6mmol/L (2.5-7.8 mmol/L)


Serum Creatinine 150 umol/L (60-115)
eGFR 42ml/min
HbA1c 49mmol/mol (<46)
Urine albumin Creatinine ratio >300 mg/mmol

Which one of the following is highly suggestive of an alternative aetiology for


the renal impairment rather than development of diabetic nephropathy?

A Systemic hypertension
B Absence of RBC cast in urine
C Duration of Type 2 DM >5 years
D The presence of diabetic retinopathy
E Rapid decline in eGFR

Return to question 1 Return to contents


A nswer E - Rapid decline in eGFR

Options A, B, C and D are incorrect, option E is the correct answer. The


presence of the following clinical and biochemical characteristics should
prompt a search for an alternative aetiology for renal impairment in patients
with diabetes.

Duration type 2 DM<5 years or Type 1 DM<10 years


Absence of diabetic retinopathy
Presence of active sediment in urine
Rapid decline in eGFR
Refractory hypertension
>30% decline in eGFR within 2-3 months of initiation of ace inhibitors
or ARB therapy
Significant haematuria
Presence of systemic illness

Ref: KDOQI clinical practice guidline for Diabetes and CKD 2012

Return to contents
Question 70 Q
A 69-year-old woman with hypertension and stage 4 chronic kidney disease
(eGFR 24 ml/min per 1.73 m2) attended a clinic for her routine review.

Investigations:
serum cholesterol 6.9 mmol/L (<5.2)
serum LDL cholesterol 3.9 mmol/L (<3.36)

Which one of the following is most likely to be true in patients with CKD?

A Hyperlipidaemia is uncommon in late stages of CKD


B Lipid lowering with cholestyramine has been demonstrated to reduce the
rate of renal disease progression
C Statin therapy has been demonstrated to be effective at reducing risk for
death in ESRD
D Treatment with simvastatin reduces lipid levels as well as C-reactive protein
in CKD
E Treatment with atorvastatin reduces the need for ACEI or angiotensin II
receptor blockers in 40% of patients with stage 4 CKD

Return to question 1 Return to contents


A nswer D - Treatment with simvastatin

Options A, B, C and E are incorrect, option D is the correct answer.


Treatment with simvastatin reduces lipid levels as well as C-reactive protein
in CKD.

Inflammation is common in CKD, and is predictive of cardiovascular risk in


this population. C-reactive protein (CRP) has emerged as a valuable indicator
of inflammation and predictor of cardiovascular risk. Although elevated CRP
concentrations probably reflect inflammation that occurs in association with
atherosclerotic disease, it has been hypothesized that CRP may actually play
more of a causative role, mediating processes involved in plaque progression.
Statin therapy is highly effective for reducing cardiovascular events among
patients without kidney disease, but there is less evidence in CKD. In particular,
the recent 4D study did not find benefit for stating in patients with ESRD.
Although there has been interest in lipid lowering therapy as a method for
preserving renal function, there is at yet little evidence to support its efficacy.
Hyperlipidaemia does occur commonly in later stages of CKD, although the
phenotype may differ from other more common forms of hyperlipidemia.
Treatment with atorvastatin has not been found to reduce the need for ACEIs
or ARBs in CKD.

Panichi V, Paoletti S, Mantuano E, Manca-Rizza G, Filippi C, Santi S, Taccola D,


Barsotti G: In vivo and in vitro effects of simvastatin on inflammatory markers in
pre-dialysis patients. Nephrol Dial Transplant 21(2):337-44, 2006

Return to contents
Question 71 Q
A 26-year-old male patient with end stage renal disease on peritoneal dialysis
from previous 5 years admitted with recently decrease history of appetite
along with flatulence. He lost 2 kg in weight over the last two months.
Peritoneal nurse noticed that from the last week, his Ultrafiltration is usually
<400 ml/day.

CT scan of abdomen was done

Look at the image

What is the most likely diagnosis?

A PD Peritonitis
B Paralytic ileus
C Encapsulating peritoneal sclerosis
D Bowel obstruction
E None of above

Return to question 1 Return to contents


A nswer C - Encapsulating peritoneal sclerosis

Options A, B, D and E are incorrect, option C is the correct answer. Patient


developed encapsulating peritoneal sclerosis. It is associated with duration
of Peritoneal dialysis, at 5 years incidence is 2-3% and at 10 years incidence
increases to 6-20%.

CT Abdomen-Encapsulating Peritoneal Sclerosis

Return to contents
Questions 72 & 73 Q
Question 72
Gitelmans syndrome transporter protein defect is located on which one of the
segment of Nephron?

A Proximal Tubule
B Thick Ascending Limb
C Thin Ascending Limb
D Distal Convoluted Tubule
E Collecting Duct

Question 73
Which one of the following chronic metal exposure is associated with an
increase risk of developing renal cell carcinoma?

A Lead
B Arsenic
C Nickel
D Cadmium
E None of above

Return to question 1 Return to contents


A nswer
72. D - Distal Convoluted Tubule
73. D - Cadmium

Answer 72
Options A, C, D and E are incorrect. Option B is the correct answer. Gitelmans
syndrome transporter defect is located at distal convoluted tubule.
Nephron defects in Transport Proteins resulting in Renal Disease
Cystinuria
Proximal Tubule
Proximal renal tubular acidosis
Bartter syndrome Thick ascending limb
Gitelmans syndrome Distal convoluted tubule
Liddles syndrome
Distal renal tubular acidosis Collecting duct
Nephrogenic Diabetes insipidus

Answer 73
Options A, B, C & E are incorrect. The correct answer is D - Cadmium.
Cadmium is highly oncogenic and is associated with lung, prostate and renal
cell carcinoma

Return to contents
Questions 74 & 75 Q
Question 74
All of the following are poor prognostic factors for lupus nephritis except?

A Male gender
B Female gender
C Younger age <24 years
D Low socioeconomic status
E Black race

Question 75
Which one of the following drug is least likely to be associated with thrombotic
microangiopathy ?

A Ticlodipine
B Quinine
C Calcineurin inhibitors
D Oral contraceptives
E Aspirin

Return to question 1 Return to contents


A nswer
74. B - Female gender
75. E - Aspirin

Answer 74
Options A, C, D and E are incorrect. Option B is the correct answer. All
above mentioned factors are related to bad prognosis except the female
gender. Other laboratory and clinical factors associated with poor prognosis
include higher serum creatinine at presentation, higher baseline proteinuria,
hypertension, severe anaemia, thrombocytopenia, and hypocomplementemia
with elevated anti-double stranded DNA levels, delay in starting therapy,
response to treatment, and nephritic relapse.

Answer 75
Options A, B, C and D are incorrect. Option E is the correct answer. Thrombotic
microangiopathy is associated with many drugs including anticancer drugs.
Mitomycin C, Tamoxifen, Bleomycin, Cisplatin, Gemcitabine. Other drugs,
OKT3, Interferon alpha, Penicillin, Rifampin, Metronidazole.Aspirin use is not
associated with thrombotic microangiopathy

Return to contents
Questions 76, 77 & 78 Q
Question 76
A 26-year-old female recently diagnosed to have Systemic lupus erythmatosis
fulfilling the American college Rheumatology (ACR )Criteria for SLE, found to
have active urinary sediment.
Urine PCR 225 (<15)
Kidney biopsy was performed found Class IV diffuse proliferative SLE.
She was offered induction therapy with cyclophosphamide & steroids but
refused due to fertility problems in future.

Which one of the following statement is correct in this patient management?


A Cyclophosphamide is only option for induction
B Mycophenolate mofetil is not another reasonable option
C Mycophenolate mofetil is inferior in achieving remission than
cyclophosphamide
D Mycophenolate mofetil is superior in achieving in remission than
cyclophosphamide
E No difference in cyclophosphamide and mycophenolate mofetil of
achieving remission.

Question 77
Which one of the following is approved drug therapy for atypical Haemolytic
uraemic syndrome?
A Basiliximab
B Infliximab
C Etanercept
D Eculizumab.
E Bortezomib

Question 78
Disease activity in lupus nephritis is monitored by all of the following
parameters except?
A Blood pressure
B GFR (Glomerular filteration rate)
C Proteinuria
D Urinary sediment
E Elevated anti-ds DNA level.

Return to question 1 Return to contents


A nswer
76. E
77. D - Eculizumab
78. E - Elevated anti-ds DNA level

Answer 76
Options A, B, C and D are incorrect, option E is the correct answer. No
difference in cyclophosphamide and mycophenolate mofetil of achieving
remission.

Many trials including ALMS proved that mycophenolate mofetil compared


with cyclophosphamide has achieved same remission and relapse rates with
steroids.

Answer 77
Options A, B, C and E are incorrect, option D is the correct answer. Eculizumab
approved for the treatment of atypical haemolytic uraemic syndrome. It is
humanized monoclonal antitbody against complement C5 and blocks the
membrane attacking complex C5b-9. Basiliximab is IL2 blocker and used as
induction therapy in kidney transplant recipients. Infliximab and etanercep used
in autoimmune diseases. Bortezomib is treatment for multiple myeloma.

Answer 78
Options A, B, C and D are incorrect, option E is the correct answer. Disease
activity in lupus nephritis is monitored by blood pressue, GFR, Proteinuria,
Urinary sediment. An elevated anti-ds DNA level usually correlates with the
relapse. Many patients with elevated anti-ds DNA level for years without any
relapse. It is not wise to treat only serologic activity in absence clinical active
disease.

Return to contents
Question 79 Q
What are the possible treatments for cell mediated rejection in a Kidney
transplant patient ?

A Plasmaphersis
B Corticosteroids, Rituximab, Plasmaphersis
C Corticosteroids, IVIG (Immunoglobulin), Thymoglobulin
D Corticosteroids, Thymoglobulin and increase signal 1 Blockers
E Corticosteroids, Bortezomib, Thymoglobulin

Return to question 1 Return to contents


A nswer
C - Corticosteroids, IVIG,
Thymoglobulin

Options A,B,D and E are incorrect, Option C is correct answer.

Treatment of Cell mediated rejection


Corticosteroids
Anti lymphocyte antibodies
Thymoglobulin,OKT3
Increase maintenance drugs
1. Signal1 and signal 3 blockers

Phases of immunosuppression
Tac=tacrolimus; CsA=cyclosporine; Ster=corticosteroids; Sir=sirolimus;
MMF=mycophenolate mofetil; AZA=azathioprine
T-cell activation and proliferation
require at least three signals
mediated by the interaction with
alloantigens. Signal 1 is the first
signal in the activation of the
IL-2 transcription pathway by
a specific antigen and involves
the T cell receptor complex
(C3+CD4) recognizing the MHC
+ peptide presented to it by an
antigen presenting cell (APC).
Costimulation (signal 2) is non-
specific and is mediated by
interaction between the B7 ligand
on the APC and the CD28 ligand
on the T cell. This signal is required
for full T-cell activation. These two
signals activate the intracellular
pathway for IL-2 and other growth
factor expression.
Signal 3 is induced by IL-2 and
other growth factors and leads
to cell cycle progression and proliferation. They are activated via the mTOR
(mammalian target of rapamycin). The fourth "signal" is programmed cell death
-- a natural consequence of T-cell activation.

Return to contents
Questions 80 & 81 Q
Question 80
According to NICE guidelines, which one of the following is the correct
indication to stop metformin therapy in the presence of renal impairment?

A eGFR <75 ml/min/1.73 m2


B eGFR <60 ml/min/1.73 m2
C eGFR <45ml/m/1.73 m2
D eGFR <30 ml/min /1.73 m2
E eGFR <15ml/min/1.73 m2

Question 81
Which one of following is a non-modifiable risk factor for NODAT ?
(New onset diabetes after transplant)

A HCV Infection
B CMV Infection
C Pre-transplant IGT
D Acute rejection History
E Proteinuria

Return to question 1 Return to contents


A nswer
80. C - eGFR <45 ml/min/1.73 m2
81. D - Acute rejection history

Answer 80
Options A, B, D and E are incorrect, option C is the correct answer.
According to NICE guidelines the metformin therapy should be stopped if
serum Creatinine >150 umol/L or eGFR<45ml/min/1.73 m2

Ref: Management of type 2 Diabetes- NICE Guidelines 2009

Answer 81
Options A, B, C and E are incorrect, option D is the correct answer. Non-
modifiable risk factor for NODAT is acute rejection history others all are
modifiable risk factors.

Slide showing Risk Factors for NODAT

Risk Factors for NODAT

Non-modifiable Potentially modifiable Modifiable

Immunosuppression
Black Hispanic Corticosteroid
Age >40 yrs Tacrolimus
Male Gender HCV infection
Cyclosporine
ADPKD CMV infection
Sirolimus
HLA mismatches Pre - TxIGT
Acute rejection Obesity/metabolic
FH Diabetes syndrome

Hypomagnesemia

Return to contents
Questions 82 & 83 Q
Question 82
Which one of the following tools accurately measure GFR (Glomerular filtration
rate) in Pregnancy?

A MDRD
B CKD-EPI
C Cystatin C
D Cockcroft and Gault
E None of above

Question 83
Which one of the following is the least likely cause of uraemic pruritis?

A Adequate dialysis
B Hyperparathyroidism
C Allergic manifestation to heparin or acetate
D Xerosis
E Derangements in immune system

Return to question 1 Return to contents


A nswer
82. E - None of above
83. A - Adequate dialysis

Answer 82
Options A, B, C and D are incorrect, option E is the correct answer.
In pregnancy the best tool is
Pre-pregnancy eGFR OR 24 hour creatinine clearance or TRACK
INDIVIDUAL CREATININE. So, option E is the correct answer

Table showing different modality of GFR estimation and there use in pregnancy

Measurement Accuracy Use Reference


MDRD Underestimates compared No Smith et al BJOG 2009
with inulin clearance
CKD-EPI Underestimates compared No Aper et al Am J Perinatol 2011
with Cr clearance
Cockcroft Underestimates compared No Conrad Renal Disease and
Gault with inulin and Cr clearance Pregnancy 2008

Cystatin Increases in second trimester No Bramham et al Obs Med 2009

Answer 83
Options B,C,D and E are incorrect Option A is the correct answer. Inadequate
dialysis often causes pruritis
Causes of uraemic pruritis
Hypercalcaemia
Hyperphosphataemia
Severe Hyperparathyroidism
Inadequate removal of middle Molecular weight Uraemic toxins
Treatment
First line - Moisturising creams and lotions
Other treatment include
Antihistamines (hydroxyzine)
UV B Phototherapy- need to be given 2-3 times per week
Topical capsaicin cream
Gabapentin- especially in peripheral diabetic neuropathy (300 mg
after each session of dialysis) confirmed in RCT.
Topical tacrolimus ointment
If pruritis occurs only during HD
Change formulation of heparin
Change type of dialysis membrane
Increase rinsing volume
If all modalities failed - Kidney transplant

Return to contents
Questions 84 & 85 Q
Question 84
Which one of the following is the most common sleep disorder in dialysis
patients?

A Insomnia
B Restless leg syndrome
C Nightmares
D Daytime sleepiness
E Sleep apnoea

Question 85
Which one of the following laboratory parameter is associated with an increase
probability of death in End stage renal disease populations?

A Increase PTH
B Low haemoglobin
C High Serum Calcium
D Low Serum Albumin
E None of above

Return to question 1 Return to contents


A nswer
84. A - Insomnia
85. D - Low serum Albumin

Answer 84
Options B, C, D and E are incorrect. Option A is the correct answer.
Insomnia is the most common sleep disorder seen in 70% of Dialysis patients.
Mostly seen in morning shift.
Merlino,Nephrol.Dial.Transpl.(2006) 21:184

Risk factors For Insomnia in Dialysis


Age
Excessive alcohol intake
Cigarette smoking
Polyneuropathy
Medications use to cause insomnia in HD patients
Phenytoin and steroids

Treatment
Behavioural treatment
Medications- benzodiazepines(temazepam or triazolam) and non
benzodiazepines (zolpidem) can be use
Antihistamines and tricyclic antidepressants has no role

Answer 85
Options A, B, C and E are incorrect. Option D is the correct answer.
Low serum albumin strongly associated with increase risk of death in ESRD.

Owen, WF Jr, Lew, NL, Liu, Y, et al, N Engl J Med 1993; 329:1001.

Return to contents
Question 86 Q
A 38-year-old female who got cadaveric kidney transplant one week ago
found to have worse graft function. Her baseline serum creatinine is 700
umol/L started to drop initially but became static around 420 umol/L from
previous three days. She developed right sided weakness which recovered
over 24 hours.

Laboratory data showed that her Haemoglobin dropped 2 gram (9 gram/dl


from baseline 11g/dl) and platelets were persistently low, recent one 82,000
micro/L. Blood film revealed moderate schistocytes.

Biochemistry: Serum creatinine 350 umol/L, electrolytes were normal, Serum


LDH 600 IU/L while coagulation profile were normal.

Bland urinary sediment. FK506 level was within target range.

Which one of the following is the most likely diagnosis in her clinical scenario?

A Chronic rejection
B Thrombotic thrombocytopenic purpura
C Calcineurin inhibitors toxicity
D Infection
E Renal vein thrombosis.

Return to question 1 Return to contents


A nswer
B - Thrombotic thrombocytopenic
purpura

Options A, B, C and E are incorrect. Option D is the correct answer.

Patient fulfilled criteria for Thrombotic microangiopathy (TMA)


It is a fibrin deposition in microvasculature, manifest by
Microangiopathic haemolytic anaemia(MAHA)
Thrombocytopenia <150,000 or <25% from baseline
Organ dysfunction >1 of , Neurological, Renal, Gastrointestinal
Normal PT/APTT
(This differentiate this from DIC (Disseminated intravascular coagulation)- in
which coagulation is disturb)

TMA= Microangiopathy+Low platelets + organ dysfunction + Normal


coagulation

She had CNS manifestation, renal impairment, thrombocytopenia and most


likely secondary to Calcineurin inhibitors.
Acute rejection was another possibility but picture favored more TTP

Difference between HUS (Haemolytic uraemic syndrome &TTP


(Thrombotic thrombocytopenic purpura)

HUS TTP

Clinical Thrombocytopenia Thrombocytopenia


features MAHA MAHA
Renal involvement more Mild renal involvement
common CNS involvement common
30% have CNS involvement
and fever
Age Child Adult

Mechanism STEC-HUS:Shiga toxin ADAMTS 13 abnormality


aHUS- alternative complement level<10%
disorder

Return to contents
A
Secondary causes of TMA

Pregnancy-(Preclampsia, HELP)
Malignant HTN
Systemic sclerosis
Infection(HIV)
Autoimmune -SLE
Disseminated Malignancy- gastric ca, colon ca
Stem cell Transplant
Antiphospholipid syndrome
Pregnancy - (Preclampsia, HELP)
Drugs -
Calcineurin inhibitors- Cyclosporine, Tacrolimus
Quinine
Antiplatlet Ticlopidine
Chemotherapy- Mitomycin, VEGF inhibitors
Gemcitabine

Treatment of Primary TTP

First line

Daily Plasma exchange( 1-1.5 Litre plasma volume with Fresh frozen
Plasma (FFP) or cryo poor plasma
Plasma exchange continues until platelet count>150 and haemolysis
markers(eg LDH normalize)
20% of patients shows transient or minimal response to initial plasma
exchange

Kidney International (2009)75 (suppl 112), S55-S58

Return to question 1 Return to contents


A nswer

Role of Adjuvant therapy in TTP

All patients must get if ADAMTS13


Corticosteroids
deficiency (<10%)

Use in Complicated/severe initial


Rituximab
episode and relapse

Cyclosprine Similar indication as rituximab

Cyclophosphamide & vincristine Refractory to above

Splenectomy Only Last resort

Return to contents
Question 87 Q
Which one of the following statements is correct about
Myeloma Cast Nephropathy?

A Myeloma cast Formed mainly in the Proximal tubules


B Myeloma cast mainly composed of Kappa light chains
C Myeloma cast formed mainly in distal tubules and collecting duct
D Majority of free light chains are extracellulary distributed
E Plasmapheresis is use as first line of treatment for myeloma
cast nephropathy

Return to question 1 Return to contents


A nswer
C - Myeloma cast formed mainly in
distal tubules and collecting duct

Options A, B, D and E are incorrect. Option C is the correct answer

Myeloma cast is typical hyaline, fractured casts (strong stain with Eosin, weak
with PAS), are seen in the distal tubuli and the collecting ducts (rarely in the
proximal tubuli) together with degenerative changes and atrophy of tubular
cells . They are surrounded by multinucleated cells of macrophage origin.

In the tubulointerstitium infiltrating inflammatory cells are seen, accompanied


by fibrosis and tubular atrophy. Rhomboid or needle-shaped crystals are found
in the casts, which can be suspected in light microscopy and seen in electron
microscopy.

Immunohistologically -light chains are detected frequently, but -light


chains rarely. The glomeruli are usually normal with the exception of a slightly
increased matrix.

Myeloma cast is the most common cause of renal insufficiency. Severe AKI
in Myeloma is a medical emergency, rapid confirmation of diagnosis, urgent
referral to haematological service and immediate commencement of disease
specific treatment is very important.

Return to contents
Question 88 Q
A 22-year-old university student of Malaysian origin, admitted with generalised
lethargy, vomiting, and diarrhoea. She has had an Indian takeaway the night
before and has started with gastrointestinal symptoms this morning. She is fit
and well and on no regular medications apart from Chinese slimming herbal
tea that she bought over the internet 3 months ago, which she religiously
drinks 3 times a day. Her blood tests show K 3.4, Urea 15mmol/L, Creatinine
345 umol/L, Urine dip shows 1+ protein. No previous blood tests are available
to compare.

Which one of the following is the most appropriate step in her further
management?

A Stool Culture and sensitivity(C&S)


B Oral fluids, Stool C&S
C IV Fluids, Stool C&S, kidney ultrasound(USS)
D IV Fluids, Stool C&S, USS, stop herbal tea, plan biopsy if renal function does
not improves.
E IV Fluids, Stool C&S, USS, stop herbal tea, empirical steroids

Return to question 1 Return to contents


A nswer D - IV Fluids, Stool C&S, USS,

Options A, B, C and E are incorrect. Option D is the correct answer.

Aristolochic acid nephropathy needs to be ruled out.

Option D is the most sensible plan of action.

The primary constituent of Aristolochia is aristolochic acid. Several case reports


have demonstrated the nephrotoxic properties of aristolochic acid, leading to
what is called Chinese herb nephropathy.

Clinical feature - Acute Kidney Injury, Nonnephrotic range proteinuria and mild
hypertension but most common renal histology lesion is Chronic hypocellular
tubulointerstitial fibrosis therefore on kidney ultrasound is small shrunken
kidneys.There is high incidence of uroepithelial cancers and risk persist lifelong.

Return to contents
Questions 89 & 90 Q
Question 89
A 33-year-old lady presenting with right flank pain, bilateral leg swelling for 2
weeks which is rapidly worsening, urine dip shows 4+ blood and 4+ protein,
other investigations include Alb 17 g/L, Creat 200 umol/L, Urine PCR 1200.
Her ultrasound shows atrophic left kidney and normal sized right kidney, it was
difficult to do duplex.

Which one of the following is the most appropriate investigation in her clinical
scenario?

A Renal biopsy
B CT KUB
C CT Venography
D MR Venography
E Urinary catheter

Question 90
Which one of the following Heavy metal causes Nephrotic syndrome?

A Lead
B Cadmium
C Nickel
D Mercury
E Arsenic

Return to question 1 Return to contents


A nswer
89. D - MR Venography
90. D - Mercury

Answer 89
Options A, B, D and E are incorrect. Option C is the correct answer.
MR Venography (to minimise risk of contrast induced nephrotoxicity) needs to
be done urgently to rule out renal vein thrombosis(in the context of nephrotic
syndrome- thrombogenic state) in the single functioning right kidney.

Answer 90
Options A, B, C and E are incorrect. Option D is the correct answer. Mercury
is the only heavy metal which cause nephrotic syndrome Lead, Arsenic and
Cadmium cause nonnephrotic range proteinuria.

Return to contents
Questions 91 & 92 Q
Question 91
A 82-year old Asian male with a complicated past medical history , including
gastrointestinal (GI) bleeding from esophageal varices, colon cancer and
chronic kidney disease stage 4, is hospitalized with a deep vein venous
thrombosis (DVT). He is obese (BMI- 41) and had a hemicolectomy 2 weeks
ago.

Lovenox (enoxaparin) is started, warfarin (Coumadin) use is contraindicated due


to history of gastrointestinal (GI) bleeding.

He will need anticoagulation with (enoxaparin) for 6 months.

Labs

eGFR 28ml/min- stable

Which of the following parameter would you check to know enoxaparin dose is
therapeutic?

A Factor Xa
B PTT
C PT
D anti-Xa level
E None of the above

Question 92
Which of the following metal exposure from industrial pollution of food
supply has shown an increase in the incidence of HTN, CKD and Kidney stone
disease?

A Cadmium
B Arsenic
C Lead
D Zinc
E Nickel

Return to question 1 Return to contents


A nswer
91. D - Anti Xa level
92. A - Cadmium

Answer 91
Options A, B, C and E are false.
Correct answer is you have to check anti-Xa level. This is NOT factor Xa level,
which most likely will be normal with Lovenox (enoxaparin) therapy.
PTT is not useful in monitoring therapy with LMWH. PTT should be used to
monitor treatment with UFH, i.e. regular IV heparin.
What is anti-Xa level? Is it some "special" factor anti-Xa?
No, there is no factor anti-Xa. There is a factor Xa, just like the other 12 factors
in the anticoagulation cascade (there is no factor VI).
Anti-Xa level is determined by a simple mixing study. Patient plasma is added
to a known amount of factor Xa with antithrombin. LMWH in the patient plasma
will bind to antithrombin and inhibit factor Xa. The amount of residual factor Xa
is inversely proportional to the amount of LMWH the plasma.
Low anti-Xa value indicates a high level of anticoagulation. High anti-Xa level
indicates a low level of anticoagulation. For example, anti-Xa level of 0.5
indicates adequate anticoagulation, a level of 3 is not sufficient.
When is anti-Xa level monitoring indicated?

Occasions in which periodic monitoring of LMWH might be considered include:


- renal failure
- obesity or underweight patients
- prolonged use
- patients at high risk for bleeding or thrombosis

Oral rivaroxaban is non-inferior to standard therapy for symptomatic pulmonary


embolism (PE) and DVT (NEJM, 2012).

Answer 92
Options B, C, D and E are incorrect, option A is the correct answer Number of
population based studies from Sri Lanka, Japan, South Korea, Thailand and
Pakistan has shown increase incidence of HTN, CKD and kidney stones after
exposure to cadmium

Return to contents
Question 93 Q
A 72-year-old man with stage II nonsmall cell lung cancer is found to have
an asymptomatic rise in creatinine coincident with the use of cisplatin. He
denies shortness of breath, leg edema, hematuria, or foamy urine. On physical
examination, he is alert and oriented. The blood pressure is 110/64 mmHg.
There is trace pretibial edema bilaterally.

Laboratory show a serum creatinine level of 170umol/L (increased from


previous measurements), sodium level of 134 mEq/L, potassium level of
3.4 mEq/L, magnesium level of 0.57 mmol/L, white cell count of 3000/l,
haemoglobin level of 9 g/dl, and platelet count of 150,000/l.
Urine output is 1200 ml over the last 24 hours.
Urine protein:creatinine ratio is 700 mg/g, increased from 200 mg/g 1 month ago.
The urine sediment shows no RBC casts or cells.

Which one of the following is the most appropriate intervention to limit further
cisplatin-induced kidney injury?

A Intravenous magnesium
B Oral N-acetylcysteine
C Isotonic saline infusion
D Isotonic saline plus mannitol
E Isotonic saline plus furosemide

Return to question 1 Return to contents


A

nswer C - Isotonic saline infusion

Options A, B, D and E are incorrect. Option C is the correct answer. Infusion


of isotonic saline prior to chemotherapy is the most effective means to
minimize further cisplatin-induced nephrotoxicity. The alkylating agent
cisplatin is nephrotoxic and can cause injury by several mechanisms including
direct proximal tubular toxicity, renal vasoconstriction, and induction of an
inflammatory response. Cisplatin is filtered and secreted by organic cation
transporters in the proximal tubules, and increased concentrations in the
proximal tubule result in tubular injury in the S3 segment of the proximal
tubule. The risk of cisplatin nephrotoxicity rises as the cumulative dose
increases. Carboplatin has somewhat less nephrotoxicity and is sometimes
used in place of cisplatin.

Toxicity can manifest as a rise in serum creatinine from renal impairment, a


Fanconi-like syndrome, or thrombotic microangiopathy (particularly when
given with other agents such as bleomycin or gemcitabine). As with other
nephrotoxic agents such as intravenous contrast, furosemide and mannitol
have not been found to be beneficial, although furosemide may be required in
patients with symptomatic hypervolemia.

N-acetylcysteine has not been shown to be beneficial in the prevention of


cisplatin-induced AKI.

Return to contents
Question 94 Q
A 25-year-old Afro-Carribean male presenting with 20 kg weight gain,
generalized swelling, BP 170/100, Creatinine 200 umol/L, Urine PCR 2000
mg/mmol. His immunology including ANA, ANCA, Anti-GBM and Serum
electrophoresis are unremarkable.

Virology is negative for HIV, Hepatitis B and Hepatitis C. He is treated with IV


diuretics, ACE inhibitor and after controlling his BP a renal biopsy is performed.

The histological features on the biopsy report show collapse and sclerosis of
the entire glomerular tuft, marked hypertrophy and hyperplasia of podocytes,
severe tubular injury and wrinkling and retraction of the glomerular basement
membrane.

Which one of the following is the most likely diagnosis in his clinical scenario?

A Global glomerulosclerosis
B Focal segmental glomerulosclerosis tip variant
C Collapsing glomerulopathy
D Focal segmental glomerulosclerosis cellular variant
E Focal segmental glomerulosclerosis perihilar variant

Return to question 1 Return to contents


A nswer C - Collapsing glomerulopathy

Options A, B, D and E are incorrect. Option C is the correct answer.


Histological variants of Focal segmental glomerulosclerosis
1. Not otherwise specified (NOS)
2. Perihilar variant
3. Cellular variant
4. Tip variant
5. Collapsing variant

Image showing different variants of FSGS

Collapsing variant
Associated with HIV, Parvovirus,
alpha Interferon and Pamidronate
treatment
Much more common in Afro-
carribeans
Typical presentation is severe
nephrotic syndrome with rapidly
declining renal function

Tip variant
Its significance is controversial
Has a good prognosis with response to steroids
Perihilar variant
Associated with hyperfiltration injury of secondary FSGS
It is the appearance characteristically seen in obesity related FSGS
Causes of Secondary FSGS Familial Form for Exams
(detected in <3% adults with FSGS)
-actinin 4
NPHSI = nephrin
NPHS2 = podocin
WT-I
TRPC6
SCARB2 (LIMP2)
INF2
CD2-associated proteins
Mitochondrial cytopathies

Return to contents
Question 95 Q
A 24-year-old woman is seen in consultation for polyuria after sustaining
traumatic brain injury in a motor vehicle accident. The 24-hour urine volume
was 3.5 L over the last day. She has been receiving intravenous 0.45% saline
at a rate of 80 ml/h and has not been able to eat or drink. She was diagnosed
with bipolar disorder 1 year ago and has been maintained on lithium since
that time.
On physical examination, the blood pressure is 130/80 mmHg. The heart and
lungs are normal, and there is no peripheral edema.
Laboratory studies show a serum sodium level of 146 mEq/L and osmolality of
299 mOsm/kg. Urine osmolality is 200 mOsm/kg.
A 24-hour urine shows a total daily solute excretion of 700 mosmol/day.
Under close observation, intravenous fluids are discontinued at 8:00 am,
and the patients nil by mouth status is maintained. Four micrograms of
intravenous desmopressin is administered at 10:00 am.

Results of urine osmolality are shown below.

Time 8:00 AM 8:30 AM 9:00 AM


Urine osmolality (mOsm/kg) 200 275 350

9:30 AM 10:00 AM 10:30 AM 12:00 PM


352 353 475 585

Which one of the following is the most likely diagnosis?

A Salt-wasting nephropathy
B Complete central diabetes insipidus
C Partial nephrogenic diabetes insipidus
D Partial central diabetes insipidus
E Osmotic diuresis

Return to question 1 Return to contents


A nswer D - Partial central diabetes insipidus

Options A, B, C and E are incorrect. Option D is the correct answer. Epithelial


sodium channel (ENaC) inhibition in the collecting duct.
Blockade of the ENaC by trimethoprim is the most likely predominant cause
of this patients hyperkalaemia. The patient has significantly decreased
GFR and was given high-dose oral TMP-SMX to treat cellulitis. This patient
developed hyperkalaemia and concurrent normal anion gap metabolic
acidosis, termed hyperkalaemic distal or type IV renal tubular acidosis.
The fact that the dose of TMP-SMX was not appropriately reduced by 50%
based on the low GFR likely increased this patients risk for hyperkalaemia.
Other drugs with similar effects on the ENaC include amiloride, triamterene,
and pentamidine. Decreased aldosterone production (e.g., with heparinoids,
advanced diabetic kidney disease, HIV infection, nonsterioidal anti-
inflammatory drugs, calcineurin inhibitors, angiotensin blockade) or
decreased aldosterone action (e.g., with spironolactone) can produce similar
biochemical abnormalities.
This patient also has polycythemia vera and an elevated platelet count, which
can also contribute to hyperkalaemia, especially in clotted (serum) samples
where platelets release their intracellular contents (pseudohyperkalaemia). In
this case, however, both serum and plasma (whole blood) levels were relatively
similar, making this mechanism a minor contributor at most.
Hyporeninemic hypoaldosteronism can complicate CKD, particularly in patients
with diabetic nephropathy. However, this patient does not have diabetes
mellitus and did not have hyperkalaemia or metabolic acidosis 1 month ago,
decreasing the likelihood that low serum aldosterone played a significant role
in this patients hyperkalaemia.
Trimethoprim can cause a rise in the serum creatinine level, typically resulting in
up to a 40umol/L increase in serum creatinine, by inhibiting creatinine secretion
via the organic cation transporter in the proximal tubules. This action, however,
does not cause a corresponding decline in GFR. There is nothing in the urine
sediment to support a diagnosis of TMP-SMX induced interstitial nephritis
leading to AKI and resultant decreased potassium excretion.
Volume depletion could have promoted the development of AKI, and indeed,
the BUN/creatinine ratio is slightly >20:1. This could contribute to decreased
urinary K loss as a result of decreased flow through the cortical collecting duct
lumen. However, both vomiting and diarrhea are associated with potassium
losses in the urine and from the gut, respectively,and so extreme hyperkalaemia
would be unlikely from these causes alone.
References: Velazquez H, Perazella MA, Wright FS, Ellison DH: Renal mechanisms of trimethoprim-
induced hyperkalaemia. Ann Intern Med 119: 296301, 1993

Return to contents
Question 96 Q
A 67-year-old man with stage G4 A1 CKD and a history of polycythemia vera
was recently started on trimethoprim-sulfamethoxazole (TMP-SMX) for cellulitis
of the left leg. He presents to the clinic with nausea, vomiting, diarrhoea, and
anorexia. He has no history of diabetes mellitus. Medications include low-dose
aspirin, folic acid, hydroxyurea, and TMP- SMX 160/800 mg twice daily.

On physical examination, he is afebrile; his blood pressure is 130/70 mm Hg,


and pulse is 70 beats/min. The muscle mass is diffusely reduced, and the
remainder of the examination is unremarkable.

Electrocardiogram shows mildly peaked T waves in leads II, V2, and V3.

Laboratory studies reveal a sodium level of 134 mEq/L, potassium (serum) level
of 9.1 mEq/L (1 month ago, it was 4.1 mEq/L), potassium (plasma) level of 8.2
mEq/L, chloride level of 110 mEq/L, total CO2 of 15 mEq/L (1 month ago, it
was 24 mEq/L), Blood urea 15 mmol/L, creatinine level of 220 umol/L(1 month
ago, it was 260 umo/L), calcium level of 21.mmol/L, and a platelet count of
707,000/l.

Urinalysis shows specific gravity of 1.012, pH 5.0, no protein, no blood, and no


leukocyte esterase.
Microscopic examination of the urine sediment shows no cells or casts.

Which one of the following factors contributed the most to the pathogenesis of
the hyperkalemia in this patient?

A Epithelial sodium channel (ENaC) inhibition in the collecting duct


B Potassium release from platelets (pseudohyperkalaemia)
C TMP-SMXinduced AKI
D Low serum aldosterone
E Volume depletion

Return to question 1 Return to contents


A nswer A - Epithelial sodium channel

Options A, B, C and E are incorrect. Option D is the correct answer. Epithelial


sodium channel (ENaC) inhibition in the collecting duct.
Blockade of the ENaC by trimethoprim is the most likely predominant
cause of this patients hyperkalemia. The patient has significantly decreased
GFR and was given high-dose oral TMP-SMX to treat cellulitis. This patient
developed hyperkalemia and concurrent normal anion gap metabolic
acidosis, termed hyperkalemic distal or type IV renal tubular acidosis.
The fact that the dose of TMP-SMX was not appropriately reduced by 50%
based on the low GFR likely increased this patients risk for hyperkalemia.
Other drugs with similar effects on the ENaC include amiloride, triamterene,
and pentamidine. Decreased aldosterone production (e.g., with heparinoids,
advanced diabetic kidney disease, HIV infection, nonsterioidal anti-
inflammatory drugs, calcineurin inhibitors, angiotensin blockade) or
decreased aldosterone action (e.g., with spironolactone) can produce similar
biochemical abnormalities.
This patient also has polycythemia vera and an elevated platelet count, which
can also contribute to hyperkalaemia, especially in clotted (serum) samples
where platelets release their intracellular contents (pseudohyperkalaemia). In
this case, however, both serum and plasma (whole blood) levels were relatively
similar, making this mechanism a minor contributor at most.
Hyporeninemic hypoaldosteronism can complicate CKD, particularly in patients
with diabetic nephropathy. However, this patient does not have diabetes
mellitus and did not have hyperkalaemia or metabolic acidosis 1 month ago,
decreasing the likelihood that low serum aldosterone played a significant role
in this patients hyperkalaemia.
Trimethoprim can cause a rise in the serum creatinine level, typically resulting in
up to a 40umol/L increase in serum creatinine, by inhibiting creatinine secretion
via the organic cation transporter in the proximal tubules. This action, however,
does not cause a corresponding decline in GFR. There is nothing in the urine
sediment to support a diagnosis of TMP-SMX induced interstitial nephritis
leading to AKI and resultant decreased potassium excretion.
Volume depletion could have promoted the development of AKI, and indeed,
the BUN/creatinine ratio is slightly >20:1. This could contribute to decreased
urinary K loss as a result of decreased flow through the cortical collecting duct
lumen. However, both vomiting and diarrhea are associated with potassium
losses in the urine and from the gut, respectively,and so extreme hyperkalaemia
would be unlikely from these causes alone.References: Velazquez H, Perazella MA,
Wright FS, Ellison DH: Renal mechanisms of trimethoprim-induced hyperkalaemia. Ann Intern Med
119: 296301, 1993

Return to contents
Question 97 Q
Hypomagnesaemia from urinary magnesium loss is seen with all of the
following medications except ?

A Cisplatin
B Carboplatin
C Cetuximab
D Omeprazole
E Diclofenac sodium

Return to question 1 Return to contents


A nswer D - Omeprazole

Option D is the correct answer. Options A, B, C and E are incorrect.

Hypomagnesaemia can occur as a result of gastrointestinal (GI) as well as


urinary losses. Proton pump inhibitors are responsible for GI loss of magnesium
as they inhibit the TRPM 6/TRPM 7 (Transient Receptor Potential Melastatin
channels) located on the intestinal epithelial cells. Urinary magnesium levels
are appropriately low in patients with hypomagnesaemia due to PPI. Cisplatin,
carboplatin, and cetuximab are all responsible for urinary magnesium losses.

Return to contents
Question 98 Q
Which one of the following statements is incorrect regarding collapsing
glomerulopathy?

A Blacks with risk polymorphisms in the APOL1 gene are at increased


susceptibility to develop collapsing glomerulopathy
B Collapsing FSGS is most often seen in association with HIV infection
C Presence of tubuloreticular structures in glomerular endothelial cells on
electron microscopy help to differentiate HIV from non-HIV infection
D A greater incidence of proliferative microcystic transformation is seen with
HIV nephropathy
E Overall prognosis after treatment with glucocorticoids is good

Return to question 1 Return to contents


A nswer
E - Overall prognosis after treatment
with glucocorticoids is good

The correct statement is overall prognosis of collapsing FSGS is poor.


Options A, B, C and D are correct.E is the incorrect answer

FSGS (Focal segmental glomerulosclrosis) Recurrence Post transplant

Rapid onset
Median 10- 14 days post transplant but can be within 24 hrs.
More likely to have had acute Kidney Injury and develop rejection.
Biopsy changes take weeks to develop except Podocyte Foot process
effacement.
Management
No good data
Plasma exchange

Return to contents
Questions 99 & 100 Q
Question 99
A 60-year-old male, previously fit and well, has admitted with backache,
tiredness and a Creatinine of 1500 umol/L. He had a normal renal function 2
years ago.
He is started on haemodialysis. He has raised kappa light chains and abnormal
serum electrophoresis.

Which one of the following management offers him the best chance of renal
recovery?
A Bone marrow biopsy, Dexamethasone and Bortezomib
B Renal biopsy, Dexamethasone
C Renal biopsy, skeletal survey, dexamethasone
D Renal biopsy, skeletal survey, dexamethasone, plasma exchange
E Dexamethasone, Bortezomib and plasma exchange

Question 100
A 27-year-old woman presents to the clinic for management of kidney pain.
A detailed family history reveals that her paternal grandfather has autosomal
dominant polycystic kidney disease (ADPKD), hypertension, and a serum
creatinine level of 223umol/L (eGFR, 33 ml/min per 1.73 m2) at the age of 70.
Her father has ADPKD, hypertension, and a serum creatinine level of 90umol/L
at the age of 51. She has never been diagnosed with high blood pressure.
Current medications include a multivitamin and a daily oral contraceptive. On
physical examination, her blood pressure is 115/70 mmHg and her height is
1.7m. The remainder of the examination is unremarkable.
Laboratory data are notable for no protein or blood on urinalysis.
Serum creatinine level is 90umol/L.
Volumetric magnetic resonance imaging (MRI) is performed. The right kidney
volume is 148 ml and the left is 350 ml.

Which one of the following statements best describes the risk of progressive
loss of kidney function in this patient?
A The risk is increased because she has kidney pain
B The risk will increase after her first term pregnancy
C The risk is increased due to the use of estrogen-containing oral
Contraceptives
D The risk is high based on her family history
E The risk of progressive loss of kidney function is low over the next 8 years.

Return to question 1 Return to contents


A
99. A - Bone marrow biopsy, Dexamethasone
and Bortezomib
nswer 100. E - The risk of progressive loss of
kidney function is low

Answer 99
Options B, C, D and E are incorrect, option A is the correct answer. Bone
marrow biopsy confirming increased plasma cell clones, Bortezomib and
Dexamethasone is the best plan out of all of them. The others are either high
risk, unnecessary or do not influence renal recovery.

Management of Multiple Myeloma


Supportive
Optimize urine output, correct Hypercalcaemia, correct acidosis, avoid
diuretics and nephrotoxic drugs
Disease specific
Chemotherapy starts as soon as possible with Dexamethasone and
novel agents. There is increasing experience of Bortezomib in renal
failure
Plasma exchange
Removal of Free light chains by plasma exchange have no role
because of the extracellular distribution immunoglobulin light chains.
85% FLC are extracellular and only 15% are intravascular

There is role of some high cut off dialysis means protein permeable dialysis
remove far more light chain then plasma exchange.

Answer 100
Options A, B, C and D are incorrect, option E is the correct answer. Volumetric
MRI is not currently recommended as part of the management for patients
with ADPKD, but this information can be helpful to identify individuals at high
risk for progression. In addition, it is likely to be used in future clinical studies
to assess response to therapeutic intervention. This patient has a total kidney
volume (TKV) of 498 ml, and a height-adjusted TKV (htTKV) of 293 ml/m. This
is well below the threshold of an htTKV of 600 ml/m associated with loss of
kidney function in 8 years in the Consortium for Radiologic Imaging Studies of
Polycystic Kidney Disease study.
In addition, no one in her family has developed ESRD before the age of
70, arguing in favor of a PKD2 mutation, which is also associated with later
progression of CKD. Her risk for progression of CKD will increase after three
term pregnancies. Pain is not associated with an increased risk of loss of kidney
function. Estrogen exposure may potentially increase her risk of liver cysts but
should not affect her kidney disease.

Return to contents
Question 101 Q
A 37-year-old man recently experienced bilateral parotid swelling and weight
loss. His serum Creatinine rose from 133umol/L (eGFR, 73 mL/min) to 385
umol/L (eGFR, 23 mL/min) within three months. He has been taking NSAID.
He was found to have bilateral hilar lymphadenopathy. Serum calcium level
was slightly elevated. Urine protein/creatinine ratio was 0.3. Urinalysis showed
only 1-2 RBCs/HPF and 1+ protein. Other serologies, including ANA, ANCA,
hepatitis B and C, are negative. Serum complement levels are normal.

Kidney biopsy is done. Look at the image.

Which one of the following is the most likely diagnosis in his clinical scenario?

A Interstitial nephritis secondary to NSAID use


B Granulomatous vasculitis
C Lymphoma involving the salivary glands and kidneys
D Granulomatous interstitial nephritis secondary to sarcoidosis
E Acute tubular necrosis

Return to question 1 Return to contents


A nswer
D - This patient was diagnosed
with sarcoidosis

This patient was diagnosed with sarcoidosis. He also underwent a lymph node
biopsy, which showed nonnecrotizing granulomatous lymphadenitis.

Above kidney biopsy image showing a Granulomatous interstitial nephritis that


is characteristically seen in sarcoidosis. Granulomatous interstitial nephritis
can be seen in a variety of other conditions,including drug-induced interstitial
nephritis, rare infections (such as tuberculosis or brucellosis), granulomatous
vasculitis, and occasionally without an underlying etiology (idiopathic
granulomatous interstitial nephritis). In this patient, there was no evidence
of vasculitis or infection. Lymphomas do not cause granulomatous interstitial
nephritis and steroid treatment alone would not help. NSAID-induced
granulomatous interstitial nephritis is a theoretical possibility, but much less
likely than sarcoidosis given the clinical history
Options A, B, C and E are incorrect.

Differential diagnosis of Granulomatous lesions in Renal Sarcoidosis

Drug Induced 55-70%


Antibiotics
NSAID

Sarcoidosis 5-10%

Granulomatosis with polyangitis (wegeners) 5%

Idiopathic 15-20%

Tuberculosis, Brucellosis, Vasculitis, SLE (all very rare)

Return to contents
Question 102 Q
An 89-year-old man returns for follow-up with his adult daughter. He has a
history of advanced CKD attributed to long-standing hypertension. Over the
last year, his eGFR has declined from 14 to 9 ml/min per 1.73 m2 earlier this
week. His other medical problems include chronic obstructive pulmonary
disease and benign prostatic hypertrophy. At his previous visits, he has been
unwilling to discuss renal replacement therapy or participate in preparations for
this eventuality.
After reviewing the recent laboratory results, the patient and his daughter
both ask whether there are alternatives to dialysis. He notes that he has lived
an active life and values his freedom and independence over quantity of life
and is interested in more conservative therapy. Upset by this conversation, his
daughter asks about the prognosis for conservative, nondialytic therapy.

Which one of the following responses best describes the outcome of active
medical care without dialysis?

A Shorter survival but fewer hospitalizations and preserved quality of life


B Equivalent outcomes and survival
C Inferior physical health
D Greater likelihood of death in hospital
E Improvement in quality of life

Return to question 1 Return to contents


A nswer A - Shorter survival but fewer hospitalizations

Options B, C, D and E are incorrect. Option A is the correct answer.


Shorter survival but fewer hospitalizations and preserved quality of life.

Conservative management of end-stage kidney disease, variably termed


conservative kidney management (CKM) or maximum conservative
management (MCM), nonaggressive kidney care, or active medical care
without dialysis, has received increasing attention as an alternative treatment
strategy for individuals with advanced CKD who are disinclined toward renal
replacement therapy (RRT).
Several studies have been conducted to characterize the expected outcomes
of such conservative therapy compared with RRT. Carson et al. found that
individuals who chose MCM had a 2-year shorter median survival than those
who opted for RRT. However, they were more likely to die at home or in
hospice and had lower rates of hospitalization than those who chose RRT. More
recently, Da Silva-Gane et al. found that patients who opted for CKM had a
13-month shorter median survival than those who opted for RRT. However,
they found that individuals who chose CKM maintained their quality of life, as
measured by stable Satisfaction with Life Scale (SWLS) score, compared with
those who chose RRT and whos SWLS score declined.
Some studies have suggested similar survival with conservative therapy versus
RRT but the majority found shorter survival. Conservative nondialytic therapy
has not been associated with improvement in quality of life.

Return to contents
Question 103 Q
Look at the images.
Image 1. (Nodular glomerulosclerosis)
Image 2. Periodic acidSchiff (PAS stain)
Image 3. (Silver stain positive for kappa light chains)
Image 4. (Silver stain negative for lambda light chains)

1 2

3 4

Which one of the following statements is incorrect regarding this condition?

A There is deposition of light chain in basement membranes and


glomerular mesangium.
B Glomerular changes are typically indistinguishable from nodular
diabetic glomerulosclerosis.
C There may be diffuse widening of mesangium or crescents.
D Tubular basement membranes are thickened by PAS +ve material
E Clinical presentation is typically with heavy proteinuria and normal
renal function.

Return to question 1 Return to contents


A nswer
E - Clinical presentation is typically with
heavy proteinuria

Options A, B, C and D are incorrect. Option E is the correct answer.

This is a light chain deposition disease. Clinical presentation is typically with


heavy proteinuria and impaired renal function.

Monoclonal immunoglobulin deposition disease (MIDD) is a rare systemic


disease that is characterized by severe renal failure as a result of the deposition
of a monotypic LC and/or HC of Ig. Glomerular lesions are so similar to
diabetic nephropathy that MIDD may serve as a model for the understanding
of this common disease. MIDD indeed is the only sclerotic glomerular disease
in which the offending molecule is defined perfectly.

Return to contents
A
Comparison of clinical manifestations, renal lesions, and hematologic features
in patients with Monoclonal immunoglobulin deposition disease (MIDD).

Characteristics LCDD HCDD

Male/female ratio 1.7 0.8

Age (yr) 57 (28 to 94) 57 (26to79)

Hypertension (%) 53 90

Renal failure (serum creatinine 130 mol/L; %) 93 83

Nephrotic syndromeb (%) 36 46

Hematuria (%) 45 89

Nodular glomerulosclerosis (%) 31 to 100 96

Multiple myeloma (%) 53 24

M component (blood or urine) (%) 88 58c

LCCD-light chain deposition disease, HCDD-Heavy chain deposition disease

Features of Light chain deposition disease on kidney biopsy


Deposition of light chain in basement membranes and glomerular
mesangium
Glomerular changes are typically indistinguishable from nodular
diabetic glomerulosclerosis
May be diffuse widening of mesangium or crescents
Tubular basement membranes thickened by PAS +ve material

Return to question 1 Return to contents


A nswer

Nephrotic syndrome Inflammatory


Cardiomyopathy syndrome
Neuropathy Familial history
Hepatomegaly M-Component
(serum/urine)

Biopsy of a superficial
organ/Kidney biopsyo

Congo red stain

Positive Negative

Amyloidosis L(H) CDD Other


HCDD IC like GN*
Immunotoctoid GN*
Cryoglobulinemia
Immunotyping

# Suggestive of AA- and hereditary amyloidosis, respectively


All renal biopsies should be routinely stained for and light chains by
immunofluorescence

* No extrarenal manifestation

Algorithm of diagnostic procedures in AL-amyloidosis and monoclonal Ig


deposition disease.

( Immune complexlike and immunotactoid glomerulonephritis also involve


monotypic deposits, but renal tubules are unaffected. Cryoglobulinemia
kidney usually is identified easily from the pattern of renal lesions. GN,
glomerulonephritis; HCDD, heavy-chain deposition disease; L(H)CDD, light-
and heavy-chain deposition disease; IC, immune complexes; M-Component,
monoclonal component).

Return to contents
Questions 104 & 105 Q
Question 104
Median survival from dialysis onset in AL (Primary) Amyloidosis is?
A 12 Months
B 18 Months
C 24 Months
D 36 Months
E 44 Month

Question 105
A 48-year-old man who had undergone cadaveric kidney transplantation 5
months ago for end-stage kidney disease due to idiopathic FSGS is admitted
for further evaluation of hematemesis. Induction therapy was anti-thymocyte
globulin. Medications are tacrolimus and mycophenolate mofetil.
On physical examination, the blood pressure is 130/80 mmHg and pulse is 86
beats/min without postural changes. The allograft is nontender and without an
audible bruit. The remainder of the examination is normal.
Laboratory studies show a serum creatinine level of 140umol/L (stable from prior
values), white cell count of 6000/l, haemoglobin level of 12.1 g/dl, and platelet
count of 269,000/l. The tacrolimus trough is 9 ng/ml.
An upper endoscopy shows multiple sessile and pedunculated polyps ranging
from 3 to 13 mm in the gastric body. Biopsies are taken from the largest
polyps and show findings consistent with monomorphic post-transplant
lymphoproliferative disease, diffuse large B-cell type. Large malignant lymphoid
cells are CD20+. In situ hybridization for Epstein-Barr virus (EBV)-encoded RNA is
positive. EBV viral load is positive at 4000 copies/ml.
A positron emission tomography/computed tomography scan of the chest,
abdomen, and pelvis showed increased F-18 fluorodeoxyglucose (FDG)
uptake, numerous retroperitoneal and mesenteric lymph nodes worrisome for
malignancy, and three subcentimeter FDG-avid foci in the left hepatic lobe and
the spleen suspicious for malignancy.

In addition to reducing immunosuppression, which one of the following is the


most appropriate treatment for this patient?
A No additional therapy
B Rituximab with chemotherapy
C Chemotherapy
D Rituximab alone
E Ganciclovir alone

Return to question 1 Return to contents


A nswer
104. E - 44 Months
105. B - Rituximab with chemotherapy

Answer 104
E - 44 Months
Options A, B, C and D are incorrect. Median survival from dialysis onset in AL
amyloidosis is 44 months.
Median survival from dialysis onset in AA amyloidosis is 69 months

Disease natural history Time from presentation


to ESRD

Primary Amyloidosis (AL) Untreated 1 Year

Secondary Amyloidosis (AA) Untreated 6 Years

Ref:Gillmore JD et al,JASN 2010,20:444-51, Pinney JH et al,JCO,2011,29:674-81

Answer 105
B - Rituximab with chemotherapy

The patient has CD20+ monoclonal post-transplant lymphoproliferative


disorder (PTLD), warranting therapy with rituximab and chemotherapy.
PTLD is a lymphoproliferative disorder most commonly of B-cell origin that
usually occurs in the setting of EBV infection in patients receiving T-cell
immunosuppression. Patients that are EBV nave and receive a kidney from
an EBV-seropositive donor are at the highest risk, although EBV-negative
disease can occur.

Options A, C, D and E are incorrect.

Return to contents 206


A
PTLD can be classified into four subtypes Treatment
as follows
Early lesions, including plasmacytic Reduction of
infiltration and an infectious immunosuppression (RIS)
mononucleosis-like PTLD; these disorders
are polyclonal
Polymorphic PTLD, polyclonal or RIS and rituximab
monoclonal lymphocytic infiltrates not
meeting full criteria for lymphoma (CD20+)
Monomorphic PTLD, a form of B-cell or T/ RIS +Rituximab if CD20+
NK-cell lymphoma (used alone if not a candidate
for broader treatment or if
symptoms are minimal). In
combination with chemotherapy
such as (cyclophosphamide,
doxorubicin, vincristine
Hodgkin lymphoma-like PTLD RIS + chemotherapy as per
protocol for Hodgkin lymphoma
Alogrithm showing management of PTLD
PTLD

Immunosuppression

Early/polymorphic disease Monomophic disease


(CD20+ or -)

Response No response CD20- polymorphic


disease

Observe CD20+ Polymorphic Chemotherapy


disease +-
Rituximab
Rituxmab

+- Surgery/radiotherapy for
localised disease

Return to question 1 207 Return to contents


A nswer

Immunosuppression should be reduced for most patients with PTLD, and


lesions that are CD20+ are generally treated with anti-CD20 monoclonal
antibodies (rituximab). Additional chemotherapy is offered if there are
systemic symptoms.

Monomorphic PTLD, when CD20+, as in this patient, is treated with both


rituximab and chemotherapy. CD20 monomorphic PTLD is treated with
combination chemotherapy.

Antiviral prophylaxis with ganciclovir has been shown to reduce the


incidence of PTLD but is not established as treatment.

Prognostic factors in PTLD

Good prognosis Poor prognosis


Age<50 yrs Age>50 yrs
Early or polymorphic disease Monomorphic disease
EBV positive EBV negative
Decrease LDH with treatment Persistent increase LDH

Return to contents
Question 106 Q
A 58-year-old lady referred through GP on Friday evening with 4 weeks history
of fatigue, joint pain and skin rash. She was known asthmatic since childhood
and had history of angina for which she take aspirin. GP prescribe her on
ibuprofen for joint pains, which she took for last 4 weeks.
Urine dipstick shows 3+ blood and 2+protein, UPCR 615
Serum creatinine 255 umol/L, eGFR 15% (baseline eGFR >90% one month ago)
Full Acute kidney injury workup was requested.

Ultrasound kidneys - Normal.

You stopped her ibuprofen


On Saturday morning lab informed you with some more results

PANCA+ve, MPO 244 (0.0-3.5IU/mL), C ANCA -ve,PR3 -ve


antiGBM antibodies 114(0-7u/ml)

Patient now complains of shortness of breath and become oligoanuric.

Which one of the following is the most appropriate step in her further
management?

A Urgent Kidney biopsy


B Continue with pulsed methylprednisolone until kidney biopsy results
available
C Start Plasma exchange immediately
D Pulsed methylprednisolone and do plasma exchange followed by iv
cyclophosphamide
E Start Dialysis immediately

Return to question 1 Return to contents


A nswer D

Options A, B, C and E are incorrect. Option D is the correct answer.


Pulsed methylprednisolone and do plasma exchange followed by iv
cyclophosphamide.

This Patient has double positive vasculitis.


Biopsy cannot be done urgently as she was on aspirin (need to hold at least 7
days prior to biopsy).
Option D had a beneficial effect on renal and patient outcomes compared
with treatment without plasma exchange. Anti-GBM antibody titers should
be monitored regularly and apheresis should be stopped when none are
detectable, typically after 1014 treatments.
Double positive patients show similar outcome to anti-GBM disease, but may
relapse like AAV (ANCA associated vasculitis)
Johnson JP, Moore J Jr., Austin HA 3rd., Balow JE, Antonovych TT, Wilson CB:
Therapy of anti-glomerular basement membrane antibody disease: Analysis of prognostic
significance of clinical, pathologic and treatment factors. Medicine (Baltimore) 64: 219227, 1985.

Return to contents
Question 107 Q
A 61-year-old male with ANCA +ve vasculitis was treated with iv
cyclophosphamide for 3 months and azathioprine for 1 year.
He presented with haemoptysis and kidney biopsy showed necrotizing
cresentic glomerulonephritis.
Over the 12 months course of treatment the extrarenal manifestations of
vasculitis disappeared.The patient eGFR was stable at 50ml/min.
Now patient coming back with another episode of haemoptysis.
There were 5 RBC/HPF.

Chest CT revealed nodular opacities and cavities.

Which of the following is the most appropriate step in his further management?

A Do urgent bronchoscopy
B IV Cyclophosphamide
C IV pulse Methylprednisolone
D Start Plasma exchange
E IV Rituximab

Return to question 1 Return to contents


A nswer A - Do urgent bronchoscopy

Options B, C, D and E are incorrect. Option A is the correct answer. There is


another disease processes rather than just original one.
Patient eGFR is stable, Urine dipstick normal, the best next step is to do a
bronchoscopy to rule out any lung pathology.

Bronchoscopy shows underline aspergillosis, which is treated with anti fungal


agents.

Cyclophosphamide dose calculation


Calculate dose according to body surface area.
The following link will take you to Hall's calculator for BSA:

http://www.halls.md/body-surface- area/bsa.htm
(also contains equations for calculating lean body weight).

Dose adjustment for reduced renal function is as follows:

If GFR < 30ml/min then recommended dose is 0.5g/m2 and


Dialysis. Remember that cyclophosphamide is removed very efficiently
by dialysis but accumulates in patients with a reduction of renal function.
All cyclophosphamide doses should be administered after dialysis and if
possible further dialysis treatments withheld for 36-48 hours.

Maximum single dose of cyclophosphamide is 1200mg.

Return to contents
Question 108 Q
Which of the following feature helps in differentiating acute Allergic interstitial
nephritis from atheroembolic disease?

A Eosinophilia
B Eosinophiluria
C Presence of Maculopapular /Morbiliform skin rash
D Urinalysis Hematuria and Pyuria
E Low C3 level

Return to question 1 Return to contents


A nswer
C - Presence of Maculopapular/
Morbilliform skin rash

Options A, B, D and E are incorrect. Option C is the correct answer.


Presence of Maculopapular/Morbilliform skin rash.

All of the above features are present in both conditions except option C that is
more typical of acute allergic interstitial nephritis where as presence of livedo
reticularis on lower extremities is highly indicates atheroembolic disease.

Table: Difference Between Acute allergic interstitial nephritis and


Atheroembolic disease.

AIN Atheroembolic disease


Eosinophilia YES YES
Eosinophilia YES YES
Rash Maculopapular/ Livedo reticularis/
Morbilliform Digital infarct(Blue toe
syndrome)
Complements-Low C3 YES YES
Urinalysis Hematuria/pyuria Hematuria/pyuria
Multi organ dysfunction Very Rare Common

Precipitating event Drug exposure-(3/7 Radiographic procedure


days)/Infection/Systemic Coronary cath/
disease Aortogram
(preceeding 1-2 weeks)

Return to contents
Questions 109 & 110 Q
Question 109
The most common cells in the infiltrate of drug induced acute Interstitial
Nephritis is?

A Eosinophils
B T Cells
C Monocytes
D Neutrophils
E Plasma cells

Question 110
Which of the following kidney biopsy features are consistent with
the diagnosis of BK Virus Nephropathy?

A Interstitial fibrosis and tubular atrophy


B Arterial intimal thickening and hyaline arteriosclerosis
C Tubular inflammation with marked neutrophilic infiltrate
D Intranuclear inclusion bodies which stain positive for large T antigen
E A, B and D

Return to question 1 Return to contents


A nswer 109 - B

Answer 109
B - Tcells

Options A, C, D and E are incorrect. Option B is the correct answer.

Drug induced Acute interstitial nephritis Characterized by predominant


involvement of the renal interstitial compartment by
Interstitial edema Interstitial cellular infiltrate
Composition of cellular infiltrate -T lymphocytes (70%- both CD4 and CD8)
Monocytes (15%) Eosinophils (variable based on drug compound) B cells
(7%) Neutrophils Granuloma formation

Acute interstitial Nephritis- Presentation


Presentation Frequency in renal biopsy series
Acute kidney injury 100%
Acute kidney injury requiring dialysis 40%
Arthralgia 45%
Skin rash 22%
Pyrexia 36%
Non visible haematuria 67%
Visible haematuria 5%
Proteinuria 93%
Nephrotic range proteinuria 2.5%
Nephrotic syndrome 0.8%
Eosinophilia 35%
Eosinophiluria 66%

Causes of Acute interstitial Nephritis


Aetiological agents Frequency in renal biopsy series
Medications 50-75%
Multisystem Inflammatory Disease 5-15%
Infection 1-10%
Unknown 10-30%

Ref: Praga M, Gonzalez E. Acute interstitial nephritis. Kidney Int. 2010;77(11):956-61

Return to contents
110 - E A
Answer 110
Options A, B, C and D are incorrect. Option E is the correct answer. Kidney
biopsy is the gold standard for the diagnosis of BK virus nephropathy. The
histology of BK virus nephropathy is characterized by tubular atrophy and
fibrosis with an inflammatory lymphocytic infiltrate that can be mistaken
for acute cellular rejection. The presence of intranuclear inclusion bodies
which stain positive for the large T antigen is pathognomonic for BK virus
nephropathy (Papanicolaou et al). Owing to patchy involvement, a negative
biopsy cannot rule out early BK virus nephropathy. Most important prognostic
parameters in determining the outcome of BK virus nephropathy is the degree
of Interstitial fibrosis and tubular atrophy.

Differential Diagnosis of BK Virus Nephropathy in Kidney Transplant

Interstitial Nephritis

Acute rejection

Acute tubular necrosis

Chronic allograft Nephropathy (IFTA)

Return to question 1 Return to contents


A nswer

BK Virus Light Microscopy

The presence of intranuclear inclusion bodies which stain positive for the large
T antigen is pathognomonic for BK virus nephropathy

Return to contents
Question 111 Q
A 30-year-old female with HIV and normal kidney function started on HARRT
(Highly Active Anti Retroviral Therapy) comprising Lamivudine,Tenofovir and
Ritonavir. She was also started on trimethoprim (cotrimoxazole) prophylaxis
against pneumocystis.
She was reviewed in clinic 3 months later.

Labs
serum sodium 136 mmol/L (137144)
serum potassium 5.5 mmol/L (3.54.9)
serum creatinine 330 mol/L (60110)
urinalysis trace protein, trace blood
MSU WCC <10, RBC <10, hyaline casts and no growth

Which one of the following is the most likely aetiology in her clinical scenario?

A HIV associated Nephropathy


B Trimethoprim causing acute interstitial nephritis
C Tenofovir nephrotoxicity
D Ritonavir toxicity
E Lamivudine nephrotoxicity

Return to question 1 Return to contents


A nswer C - Tenofovir nephrotoxicity

Options A, B, D and E are incorrect. Option C is the correct answer. Tenofovir


is a nucleoside analogue of adenosise-5-monophosphate used as an inhibitor
of viral reverse transcriptase in the therapy of HIV infection. It is well known
that tenofovir is associated acute tubular necrosis in association with severe
lactic acidosis in patients treated with this drug. The toxicity is thought to
be mediated through mitochondrial DNA depletion and is potentiated by
concomitant therapy with stavudine or didanosine. Urine microscopy is not
suggestive of allergic interstitial nephritis.

Perazella MA. Acute renal failure in HIV-infected patients: A brief review of common causes. Am
J Med Sci 319:385-391; 2000. Coca S, Perazella MA. Rapid communication: acute renal failure
associated with tenofovir: evidence of drug induced nephrotoxicity. Am J Med Sci 324:342-4; 2002.

HIV Drugs and Renal Disease


Abacavir AIN (Acute Interstitial Nephritis)
Didanosine/Lamivudine/Stavudine Fanconi syndrome, acidosis
Fanconi syndrome,
Tenofovir
Nephrogenic diabetes insipidus
Indanavir Crystals
Ritonavir AKI (Acute Kidney Injury)
Adenofovir ATN (Acute tubular necrossi)

Return to contents
Questions 112 & 113 Q
Question 112
Which one of the following factor is least favourable when considering
extracorporeal therapy for a removable substance?

A Toxic blood levels (if a threshold of toxicity can be defined)


B Serious clinical manifestations despite optimal support
C Where delayed action is possible for example ethylene glycol,
methanol, paraquat poisoning
D Metabolic effects for example severe acidosis
E Large volume of distribution of distribution and molecular weight

Question 113
Conditions for which there is strong evidence for benefit of plasma exchange
do not include is?

A Multiple Myeloma
B ANCA associated systemic vasculitis
C Anti-GBM antibody disease
D ABO incompatible kidney transplantation
E Thrombotic thrombocytopaenic purpura

Return to question 1 Return to contents


A nswer
112. E - Large volume of distribution
and large molecular weight

Options A, B, C and D are incorrect. Option E is the correct answer.


Large volume of distribution and Large molecular weight.
All of the above factors affect drug removal by extracorporeal therapy except
option E. Low molecular weight and low volume of distribution enhance drug
removal by extracorporeal therapy.

Factors that Enhance Drug Removal by Extracorporeal Therapy

Drug Related Factors Therapy Related Factors

Low Molecular weight (<500Da) Large surface area of dialysis


membrane

Low protein binding(<80%) High flux dialysis

Low volume of distribution(<1L/Kg) High blood and dialysate flow

Water soluble Increases ultrafiltration rate

Fast redistribution from peripheral Increased time on dialysis


compartment into blood

Return to contents
113. A - Multiple Myeloma A
Options B, C, D and E are incorrect. Option A is the correct answer.
In multiple myeloma there is only some evidence,Plasma exchange is not
routinely used There is only some evidence, not routinely used.
Negative randomised controlled trial in 2005.

Light-chain removal by plasmapheresis in myeloma-associated


renal failure.
Cserti C, Haspel R, Stowell C, Dzik W. Transfusion. 2007;47(3):511.
Plasma exchange when myeloma presents as acute renal failure:
a randomized, controlled trial.
Clark WF, Stewart AK, Rock GA, Sternbach M,
Sutton DM, Barrett BJ, Heidenheim AP, Garg AX, Churchill DN, Canadian Apheresis
Group Ann Intern Med. 2005;143(11):777

Return to question 1 Return to contents


A nswer

Indications for Plasma Exchange


Plasma exchange is the treatment of choice for
HUS/TTP these conditions with FFP (Fresh Frozen plasma)
as replacement.

Recent evidence suggest that PEX has advantage


Severe crescentric ANCA
over methylprednisolone if serum creatinine >500
positive vasculitis
umol/L

Levy 2001 study showed intensive therapy


including PEX worked well. Several advise use
Anti GBM disease
only if pulmonary haemorrhage or recoverable
renal function
Can be use in severe lupus cerebrates in
SLE combination with cyclophosphamide. Probable no
benefit
Anecdotal and small series reports that it can
Hyperviscosity syndromes
benefit these patients.

Multiple myeloma No benefit

No RCT performed to showed benefit. but clinical


reports suggest benefit .IgM has half life day of
Cryoglobulinaemia
5 days. expert opinion for treatment is 3 week .
dose recommend 4 L exchanges. (Madore)

Please note, there is no role of plasma exchange in TTP secondary to


Malignancy, Chemotherapy and bone marrow transplantation.

Return to contents
Questions 114 & 115 Q
Question 114
A 25-year-old gentleman admitted with tremors, hyperkalaemia and confusion.
He had a cadaveric renal transplant 2 years ago. His baseline Serum creatinine
is 120 umol/L. He has been started on several new medications recently and
has not been able to attend transplant clinic appointments.

His bloods showed Tacrolimus level of 25, K 7.0,Creatinine of 300 umol/L.

Which one of the following is the most likely culprit drug in his clinical scenario?

A St Johns Wort
B Phenytoin
C Carbamazepine
D Caspofungin
E Clarithromycin

Question 115
The most common renal manifestation of sarcoidosis is

A Hypercalciuria
B Hypercalcaemia
C Tubulointerstitial nephritis
D Nephrolithiasis
E Glomerulonephritis

Return to question 1 Return to contents


A nswer 114. E - Clarithromycin

Answer 114
Options A, B, C and D are incorrect.
Option E is the correct answer.
E - Clarithromycin increases blood levels of calcineurin inhibitors through
CYP450-3A4 inhibition
Common Drug Interactions of CNI (Calcineurin Inhibitors)
Important Drug interactions that decreases CNI (Calcineurin inhibitor) Level
by CYP450 enzyme induction

Anti Tuberculosis drugs


Rifampicin
Rifabutin
Isoniazid
Anticonvulsants
Barbiturates
Phenytoin
Carbamazepine
Herbal
St Johns Wort

Important Drug interactions that Increases CNI Level by CYP450 enzyme


inhibition

Antibiotic - Macrolides (clarithromycin) but not azithromycin


Ca Channel blockers-Non Dihydropyridines(Diltiazem)
mTOR inhibitors- Sirolimus, Everolimus
Antifungal- Ketoconazole, Fluconazole
Grapefruit juice

Statins and CNI


Risk of rhabdomyolysis increased- use pravastatin or fluvastatin
Colchicine and CNI
Increase Risk of Rhabdomyolysis
Allopurinol and Febuxostat with azathioprine
Potentially fatal bone marrow suppression
Antivirals (valaciclovir , valganciclovir) with Mycopheolate mofetil
Approximate doubling of plasma concentration
Competition for tubular secretion

Return to contents 226


115. A - Hypercalciuria A
Answer 115
A - Hypercalciuria

Renal involvement in sarcoidosis. The principal manifestations of renal


involvement in sarcoidosis are the functional abnormalities resulting from
the altered metabolism of calcium as a result of the increased synthesis of
1,25-dihydroxy-vitamin D3 by the macrophages of the granulomatous lesions.
The consequent increased calcium absorption from the gastrointestinal tract
results in the hypercalciuria that can be detected in more than half of patients.
The frequency of hypercalciuria depends on the extent of granulomatous lesions
and on the time of the year, being more common in spring and summer when
exposure to the sun is greatest. Hypercalcaemia is less common and usually
depends on coexistent deterioration of renal function when the capacity of the
kidney to excrete calcium is compromised. In most patients, hypercalciuria is
asymptomatic. Its principal manifestations are inability to concentrate the urine
and polyuria. Nephrolithiasis occurs in about 10% of patients; another
10% develop nephrocalcinosis.
Options B, C, D and E are incorrect.

Return to question 1 227 Return to contents


A nswer

Renal Manisfestation of Sarcoidosis

Calcium Metabolism Percentage


Hypercalciuria 50-60%
Hypercalcaemia 10-20%
Nephrolithiasis 10%
Nephrocalcinosis 5-10%

Tubulointersitial Intersitial Nephritis

Granulomatous 15-40%
Fibrotic 10-20%

Glomerulopathy rare

Membranous
Proliferative
Focal segmental glomerulosclerosis

Obstructive nephropathy rare

Retroperitoneal fibrosis
Retroperitoneal lymphadenopathy

Return to contents
Questions 116 & 117 Q
Question 116
A 49-year-old woman with end-stage kidney disease due to IgA nephropathy
on maintenance haemodialysis is evaluated for possible kidney transplantation.
She was diagnosed with infiltrating ductal carcinoma of the right breast last
year, for which she underwent right mastectomy and radiation therapy. The
axillary nodes were negative for tumour, and she did not require chemotherapy.
She is currently doing well and is without evidence of recurrent disease. Her
sister, who is blood type compatible and in good health, would like to be
evaluated for a living kidney donor.

Physical examination is unremarkable except for post-surgical changes of the


right chest. Laboratory studies are consistent with end-stage kidney disease,
and liver chemistries are all normal.

In the absence of recurrent breast cancer, which one of the following is the
most appropriate management regarding candidacy for kidney transplantation
for this patient?

A Proceed 2 years after breast cancer treatment


B Proceed immediately with kidney transplantation
C Proceed 3 years after breast cancer treatment
D Proceed 5 years after breast cancer treatment
E Kidney transplantation is indefinitely contraindicate

Question 117
The DOPPS study (Dialysis Outcomes and Practice Patterns Study) observed a
45% higher risk of death in patients with pre-dialysis sodium levels less than

A <145 mEq/dL
B <140 mEq/dL
C <137 mEq/dL
D <130 mEq/dL
E < 125 mEq/dL

Return to question 1 Return to contents


A nswer
116. D - Proceed 5 years after
117. C - <137mEq/dL

Answer 116
D - Proceed 5 years after breast cancer treatment

Kidney transplantation should be delayed 5 years following treatment of


breast cancer for patients. A 2-year waiting period may be appropriate for
ductal carcinoma in situ. The rate of recurrence for previously treated breast
cancer after kidney transplantation is approximately 23% and depends on the
tumour characteristics and stage. For most patients, this high rate of recurrence
occurred even when the waiting time exceeded 2 years after treatment. Factors
associated with increased risk for recurrent breast cancer include invasive
disease, nodal involvement, bilateral disease, and inflammatory carcinoma.
Recurrence of breast cancer after kidney transplantation has a 76% mortality.
Options A, B, C and E are incorrect.

Answer 117
Options A, B, D and E are incorrect.
Option C is the correct answer.
C <137mEq/dL
The DOPPS (Dialysis Outcomes and Practice Patterns Study) observed a 45%
higher risk of death in patients with pre-dialysis sodium levels less than 137
mEq/dL

Return to contents
Questions 118 & 119 Q
Question 118
Which one of the following statement is most appropriate about refractory
peritoneal dialysis (PD) Peritonitis?

A Failure of effluent to clear after 2 days of antibiotic


B Failure of effluent to clear after 3 days of antibiotic
C Failure of effluent to clear after 5 days of antibiotic
D Failure of effluent to clear after 10 days of antibiotic
E Failure of effluent to clear after 14 days of antibiotic

Question 119
A 45-year-old male who had a terminal ileostomy for five years presented with
a renal colic.
A plain abdominal x-ray shows no stones.

Investigations
Serum sodium 138 mEq/L
Serum urea 7.2 mmol/L
serum Potassium 3.6 mmol/L
serum bicarbonate 17 mmol/L
serum albumin 39g/L
serum calcium 2.4 mmol/L
serum phosphate 0.9 mmol/L

Which one of the following is the most likely aetiology for his symptoms?

A Oxalate stone
B Calcium stone
C Uric acid stone
D Cystine stone
E Staghorn calculi

Return to question 1 Return to contents


A nswer
118. C - Failure of effluent to clear
after 5 days of antibiotic

Answer 118
Options A, B, D and E are incorrect.
Option C is the correct answer.

Definitions of Recurrent, Relapsing and Repeat PD peritonitis

Interval between 2 Causative organism for the


Type of Peritonitis
episodes of peritonitis first and second infection

Recurrent <4 week Different

Same or cultures from one of


Relapse <4 week
the 2 episode is sterile

Repeat >4 week Same

Failure of effluent to
Refractory clear after 5 days of
antibiotic therapy

Adapted from Li PK, Szeto CC, Piraino B, Bernardini J, Figueiredo AE, Gupta A, Johnson DW,
Kuijper EJ, Lye WC, Salzer W, Schaefer F, Struijk DG; International Society for Peritoneal Dialysis:
Peritoneal dialysis-related infections recommendations: 2010 update. Perit Dial Int 30: 393423, 2010

Return to contents
119. C - Uric acid stone
A
Answer 119
Options A, B, D and E are incorrect.
Option C is the correct answer. Uric acid stones are not uncommon in patients
with ileostomies.
The increase enteric losses of fluid and bicarbonate result in the production of
concentrated acidic urine, predisposing to the precipitation of uric acid.
Regular ingestion of sodium bicarbonate supplements usually prevents the
problem.

Appearance of stone on urinary tract imaging

Stone Radiography Ultrasonograph Unenchanced


CT
Calcium
Oxalate
Struvite
Uric Acid
Cystine *
Indinavir

Visible
Not visible
* Stones are faintly radio-opaque or radiolucent

Return to question 1 Return to contents


A nswer

Composition Frequency (%)

Calcium oxalate 60

Calcium oxalate and phosphate 10

Calcium phosphate 10
Magnessium, ammonium phosphate
5-10
(struvite)
Uric acid 5-10
Cystine 1
Others 1

Crystalline composition of renal calculi & their frequency


Morton,A,R et al, CMAJ 2002,166-213-218

Return to contents
Question 120 Q
Treatment with cinacalcet should be stopped if the parathyroid hormone (PTH)
levels in the blood do not fall substantially within?

A 1 month
B 2 month
C 3 month
D 4 month
E 6 month

Return to question 1 Return to contents


A nswer D - 4 months

Options A, B, C and E are incorrect. Option D is the correct answer.


Cinacalcet is not generally recommended for treating people on dialysis who
have hyperparathyroidism because of their kidney disease.
However it is recommended for people on dialysis who
have very high levels of parathyroid hormone in their blood that cant be
lowered by other treatments
cannot have an operation to remove the parathyroid glands
(parathyroidectomy) because of the risks involved.
People who do receive cinacalcet should have regular checks.
Treatment should be stopped if parathyroid hormone levels in their blood do
not fall substantially within 4 months. (response means if>30% PTH reduction).

iPTH > 300pg/ml

Begin Cinacalcet Treatment

Titrate Cinacalcet Dose if iPTH


remains >300pg/ml

iPTH <150pg/ml iPTH = 150 - 300pg/ml iPTH > 300pg/ml


Despite Maximum
Dose of Cinacalcet

Ca and P Ca or P Ca and P Ca or P
controlled high controlled high

Reduce Vit D Maintain Reduce Dose of Increase or Start Adjust


Vit D Vit D Phosphate
Binders

When indicated, the dose of a vitamin D (vit D) sterol was reduced by


approximately 50% in sequential steps until a minimum administered dose was
reached: intravenous calcitriol, 0.5 g three times per week (TIW); intravenous
alfacalcidol, 1 g TIW; intravenous paracalcitol, 2 g TIW; oral calcitriol, 0.25
g TIW; oral alfacalcidol, 0.25 g/d. a The dose of cinacalcet was reduced if a
patient was not receiving vitamin D.
Messa P et al. CJASN 2008;3:36-45

Return to contents
Question 121 Q
A 48-year-old woman maintained on chronic haemodialysis is seen during
dialysis rounds 72 hours following subtotal parathyroidectomy. Postoperatively,
she required a calcium infusion for 24 hours and was discharged on calcium
carbonate 2.5 g twice a day between meals. The serum calcium level at the
time of discharge was 1.95 mmol/L, with a normal serum albumin level.
She complains of generalized weakness and muscle cramps.

On physical examination, A chvostek sign is not elicited. A stat serum


calcium level returns at 1.7 mmol/L. She has been adherent with her calcium
supplementation. The current dialysis prescription calls for a 2-meq/L
(1.63-mmol/L) calcium dialysate level, and she receives 3g of paricalcitol
intravenously with each dialysis session three times weekly.

Which one of the following is the most appropriate therapeutic option


in her case?

A Increase dialysate calcium


B Increase oral calcium
C Change paricalcitol to calcitriol
D Increase dialysate calcium and increase oral calcium.
E Increase dialysate calcium, change paricalcitol to calcitriol, and
increase oral calcium supplementation

Return to question 1 Return to contents


A nswer E - Increase dialysate calcium

Options A, B, C and D are incorrect, option E is the correct answer.

Increase dialysate calcium, change paricalcitol to calcitriol, and increase oral


calcium supplementation

This patient has symptomatic hypocalcaemia following parathyroidectomy,


often referred to as the hungry bone syndrome. Treatment involves
supplementing calcium both orally and by increasing the dialysate calcium
level, as well as addition of calcitriol.

Calcitriol is more effective than paricalcitol at enhancing intestinal absorption


of calcium and phosphorus and is indicated in this patient with symptomatic
hypocalcaemia. Typical doses for calcitriol are 0.54g daily.

Combination therapy with active vitamin D and calcium supplementation is


essential for a positive calcium balance sufficient to ameliorate symptoms and
restore the serum calcium level to normal. Treatment continues until symptoms
resolve and the serum calcium level is restored to the normal range.

Therapy can then be tapered, following the serum calcium level closely.

Adjustment of the dialysate calcium level alone, increasing calcium


supplementation without adding active vitamin D, or use of calcitriol without
calcium supplementation would not be adequate to correct the hypocalcaemia
and symptoms in a timely manner.

Return to contents
Question 122 Q
A 61-year-old man known case of end stage renal disease secondary to ADPKD
on thrice per week maintenance haemodialysis since 4 years presented with a
skin rash.

Which one of the following is the most likely presentation of this skin rash?

A Painful, symmetrical, well demarcated, non-healing ulcers.


B Painless, asymmetrical, ill defined, granulating ulcers.
C Livedo reticularis.
D Hyperkeratotic papules.
E Foot ulcers with callosities at the pressure points.

Return to question 1 Return to contents


A nswer
A - Painful, symmetrical,
well demarcated, non-healing ulcers

Options B, C, D and E are incorrect, option A is the correct answer.

Painful, symmetrical, well demarcated, non-healing ulcers.

Intense pain associated with cutaneous lesions and palpation of firm calcified
subcutaneous tissue is suggestive of calciphylaxis in dialysis patients and
patients with other risk factors for calciphylaxis.

Calciphylaxis clinically presents with severe painful skin lesions (livedo


reticularis, reticulate purpura, violaceous plaques, or indurated nodules) that
demonstrate poor healing and are frequently complicated by blistering and
ulcerations with superimposed infections.
Histologically, calciphylaxis is characterized by calcification, microthrombosis,
and fibrointimal hyperplasia of small dermal and subcutaneous arteries and
arterioles, leading to ischemia and intense septal panniculitis.
Although the pathogenesis of calciphylaxis is not well understood, several risk
factors are implicated.

Hyperphosphataemia and raised calcium-phosphate product secondary to


hyperparathyroidism is an important predisposing factor.

Skin Lesions begin as painful, symmetrical, violaceous discolouration.


Lesions evolve into well demarcated, non-healing ulcers that become necrotic
and gangrenous.
Lesions may have proximal or distal localisation.
Proximal lesions have poorer prognosis.

Image showing Calciphylaxis Lesions evolve


into well demarcated, non-healing ulcers
that become necrotic and gangrenous

Return to contents
A
Calcific Uremic Arteriopathy (Calciphylaxis)

Is a Progressive cutaneous necrosis


There is a Calcium phosphate deposition within vessel walls

Risk Factors
Dialysis >6-7 yrs
Female sex
Obese
Diabetes
Hyperphosphataemia
Hypercalcaemia
Hyperparathyroidism
Adynamic bone disease
Steroids,Warfarin, active Vit D, Iron therapy, Calcium based supplements,
calcium based phosphate binders, Hypoalbuminemia, Heparin injections.

Mortality is > 50 %
Common areas affected
Lower extremities
Proximal localisation
Distal localisation
Abdomen
Breasts

Return to question 1 Return to contents


A nswer

Differential diagnosis of calciphylaxis

Condition Features of Clinical Mimic Features of Calciphylaxis

Atherosclerotic vascular Symptoms of claudication, Can be proximal or distal


disease weak peripheral pulses, distal distribution, severe pain,
distribution, abnormal ankle- dermal arteriolar calcification
brachial index on skin biopsy

Cholesterol Usually in acral distribution, may Can be proximal or distal


embolization have features associated with distribution, dermal arteriolar
renal or gastrointestinal ischemia, calcification on skin biopsy
cholesterol clefts on skin biopsy

Nephrogenic systemic Brawny plaques, thickened Severe pain, dermal arteriolar


fibrosis skin, history of exposure to calcification on skin biopsy
gadolinium, moderate intensity
pain, marked increase in spindle
cells and fibrosis on skin biopsy

Oxalate vasculopathy Acral distribution, history Can be proximal or distal


of calcium oxalate stones, distribution, calcium deposits
birefringent, yellowish-brown, nonpolarizable
polarizable crystalline material
deposition in dermis and
arteriolar wall on skin biopsy

Purpura fulminans Usually seen in settings such as Unlikely to have diffuse


septic shock or disseminated whole-body distribution,
intravascular coagulation, absence of serologic features
diffuse body distribution, rapid of disseminated intravascular
progression, clinical features of coagulation, dermal arteriolar
shock calcification on skin biopsy

Vasculitis Systemic features of vasculitis, Absence of systemic features


serologic test abnormalities and serologic abnormalities of
(eg, cryoglobulins), no dermal vasculitis (unless autoimmune
arteriolar calcification on skin disease is a trigger for
biopsy, unlikely to have full- calciphylaxis), black eschar,
thickness necrosis or large areas dermal arteriolar calcification
of involvement on skin biopsy

Warfarin necrosis Typically seen within the first 10 d Warfarin exposure of


of warfarin initiation, manifestation prolonged duration when
of paradoxical hypercoagulable calciphylaxis associated with
state created by a transient warfarin therapy, black eschar,
imbalance in procoagulant and dermal arteriolar calcification
anticoagulant pathways, warfarin
on skin biopsy
discontinuation associated with
clinical improvement in most cases

Return to contents
A
Management of Calciphylaxis

Wound management
Wound care team should be involved for recommendations regarding
selection of dressings, chemical debriding agents, frequency of dressing
changes, and negative pressure wound therapy.
Surgical wound debridement should be considered on a case-by-case
basis.
Hyperbaric oxygen therapy can be considered as a second-line
treatment if wounds not improving; claustrophobia, access to treatment,
and cost can be significant limiting factors of this therapy.
Antibiotic administration should be guided by clinical appearance of
lesions and accompanying systemic features.

Pain management
Often narcotic analgesics are required to control severe pain associated
with calciphylaxis.
Fentanyl may be preferred over morphine to minimize potential
hypotension episodes associated with morphine.

Sodium thiosulfate
Intravenous sodium thiosulfate at doses ranging from 12.5-25g in the
last 30 min of each haemodialysis session for patients on 3/wk dialysis
schedule47.
Nausea, metabolic acidosis, hypotension, and volume overload are
potential adverse effects.
Intralesional sodium thiosulfate has been described to aid in the
resolution of calciphylaxis lesions.

Return to question 1 Return to contents


A nswer

Management of mineral bone disease


Serum calcium and phosphorus levels should be maintained in normal
range and serum parathyroid hormone level should be maintained at
150-300ng/mL.
Serum calcium and phosphorus levels should be maintained in normal
range and serum parathyroid hormone level should be maintained at
150-300ng/mL.
Calcium supplements, high dialysate calcium bath, vitamin D
preparations should be avoided and instead cinacalcet to be considered
to treat secondary hyperparathyroidism in patients with calciphylaxis;
surgical parathyroidectomy is indicated in patients with refractory
hyperparathyroidism.
Excessive suppression of parathyroid hormone should be avoided.

Dialysis prescription
Haemodialysis prescription should be optimized to achieve the
recommended NKF-KDOQI goals of adequacy.
NKF-KDOQI clinical practice guidelines for peritoneal dialysis adequacy
should be followed for peritoneal dialysis patients.

Return to contents
Question 123 Q
Which one of the following is least likely to be present in normal pregnancy?

A 50% increase in GFR


B Decreased serum osmolality
C Doubling of urine protein excretion
D Unilateral pelvicalyceal dilatation
E Metabolic alkalosis

Return to question 1 Return to contents


A nswer E - Metabolic alkalosis

Metabolic alkalosis

Options A,B,C,D are correct with regard to normal pregnancy except option E

Increased filtered bicarbonate is not matched by an equal increase in


reabsorption leading to bicarbonaturia and a metabolic acidosis. Within 6
weeks gestation, systemic vascular resistance falls leading to an increase in
cardiac output and circulating volume. Subsequently, renal blood flow increases
by 75% and GFR increased by 50% by 30 weeks gestation.
The upper limit of normal urine protein excretion increases from 150mg/d to
260mg/d in pregnancy.
Smooth muscle relaxation and the effect of the gravid uterus commonly lead to
a degree of hydronephrosis, more commonly on the right than the left. In the
absence of pain, renal dysfunction or recurrent infection this does not require
further investigation during pregnancy

Normal values of Serum Creatinine and urea

Serum Creatinine Serum urea


(umol/L) (mean) (mmol/L) (mean)
Non pregnant women 73 (0.82mg/dl) 4.3 (25mg/dl)
Pregnant women 51 (0.5 mg/dl) 3.3 (20mg/dl)

Return to contents
A
Causes ok AKI in pregnancy

Volume depletion
Hyperemesis gravidarum
Postpartum bleeding
Placental abruption

Sepsis
Septic abortion
Acute pyelonephritis

Severe preeclampsia
Bilateral cortical necrosis
Thrombotic microangiopathies (TTP-HUS)
Acute fatty liver of pregnancy
Urinary tract obstruction from gravid uterus

Return to question 1 Return to contents


A nswer

Normal adaptive changes during Pregnancy

Structural Changes in the Kidney


Increase in kidney size by 1 to 1.5 cm
Dilation of the collecting system, more prominent on the right

Hormonal Changes
10- to 20-fold increase in aldosterone
8-fold increase in renin
4-fold increase in angiotensin

Systemic Hemodynamic Changes


Increased cardiac output by 40% to 50% of normal
Increased plasma volume by 40% to 50% of normal
Drop in SBP by 9 mm and DBP by 17 mm Hg (prominent in second
trimester)
Resistance to pressor effect of angiotensin
Increased production of prostacyclin and nitric oxide

Renal Hemodynamic Changes


Increase in GFR and RPF by 50% above normal
Decrease in glomerular capillary oncotic pressure

Metabolic Changes
Decrease in Blood urea 4.6mmol/L
and serum creatinine (to 35-45 umol/L)
Increase in total body water by 6 to 8 liters
Net retention of 900 mEq of sodium
Decrease in plasma osmolality by 10 mOsm/L
Decrease in serum sodium by 4 to 5 mEq/L
Mild respiratory alkalosis with compensatory metabolic acidosis
(bicarb of 18 to 22 mEq/L)
Decrease in serum uric acid levels (to 2.5 to 4 mg/dL)
Glucosuria irrespective of blood glucose levels

Return to contents
Question 124 Q
A 68-year-old man known case of CKD stage 5 secondary to diabetic
nephropathy (eGFR 8ml/min) and with a working mature left AVF was seen
in the clinic 3 days ago when he was presented with uraemic symptoms and
was advised to start dialysis at that time but he refused and went home.
Now he presented to accident and emergency with Nausea and vomiting
Investigations.

Serum sodium 149 mmol/L, Serum potassium 5.8 mmol/L, Blood urea 79
mmol/L, Serum Creatinine 800 umol/L (eGFR 4 ml/min), Serum Bicarbonate
8mmol/L.

He is admitted for initiation of haemodialysis, at the end of 2 hours he rapidly


became unresponsive, there were no localizing neurological signs. His blood
pressure was 160/80mmHg(unchanged).

Which one of the following is the most appropriate step in his


further management?

A Arrange for a CT Brain to be undertaken after dialysis


B Stop Dialysis immediately
C Start an intravenous labetolol infusion
D Stop the dialysis heparin
E Call ICU

Return to question 1 Return to contents


A nswer B - Stop Dialysis immediately

Options A, C, D and E are incorrect, option B is the correct answer.

Stop Dialysis immediately.


He is likely to have Dialysis disequilibrium syndrome (DDS) and continuing
dialysis may lead to death. Intracranial bleeds are more common in
haemodialysis patients than those on peritoneal dialysis but stopping the
heparin alone is not adequate given the risk of DDS.

Dialysis disequilibrium syndrome is characterized by neurologic symptoms of


varying severity, from headache and dizziness to seizure and coma. It is seen in
patients new to dialysis therapy with very high serum urea nitrogen levels (>70
mg/dL [>60 mmol/ L]) and is caused by the rapid lowering of plasma osmolality
and intracellular movement of water in the brain causing cerebral edema.
Dialysis disequilibrium syndrome rarely is encountered now because dialysis
therapy is initiated in a gradual fashion, with short treatments and low blood
flow rates.

Return to contents
QA
Capillary Cerebrospinal Brain
fluid

Start of Urea Urea


Dialysis

Urea Urea Urea


during
Dialysis
Urea Urea
Urea
Osmolality Osmolality Idiogenic osmoles
Osmolality
H2O H2O
end of
Dialysis

Pathogenesis of dialysis disequilibrium syndrome caused by increased removal


of plasma urea, with slower removal from CSF and brain tissue, thus setting up
an osmotic gradient with passage of water from the relatively hypotonic plasma
to the relatively hypertonic brain, causing brain edema.

Return to question 1 Return to contents


A nswer

Dialysis Disequilibrium syndrome risk factors

RISK FACTORS SYMPTOMS/SIGNS


New ESRF or missed dialysis Headache
Serum urea >60mmol/L Nausea
Older age Disorientation
Paediatric patients Restlessness
Metabolic acidosis Asterixis
Pre-existing CNS disease Blurred vision
Confusion
Seizures

Prevention - requires slow reduction of serum urea by short, low efficiency


dialysis repeated frequently

Return to contents
Questions 125 &126 Q
Question 125
Which one of the following statement is most appropriate about visceral
peritoneal membrane ?

A It comprises of 20% of total surface area


B It comprises of 40% of total surface area
C It comprises of 60% of total surface area
D It comprises of 70% of total surface area
E It comprises of 80% of total surface area

Question 126
A 56-year-old man on peritoneal dialysis since 6 years presented to you with
complain of poor ultrafiltration. His baseline 2.5% dextrose PET (Peritoneal
Equilibration test) test showed that D/P creatinine was 0.62 and 4 hrs
Ultrafiltration (UF) 450 ml.

Which one of the following will be a useful test to establish the diagnosis?

A 4.25% Dextrose PET


B Repeat 2.25% Dextrose PET
C CT abdomen with intraperitoneal contrast
D Rapid in and out PD dwell to check drain volume
E Do nothing reassure patient everything ok

Return to question 1 Return to contents


A nswer
125. E - 80% of total surface area
126. A - 4.25% Dextrose PET

Answer 125
Options A, B,C and D are incorrect, whereas option E is the correct answer.
Visceral peritoneal memberane comprise of 80% of total surface area.

Peritoneal serosal memberane is lined by mesothelium. surface area


approximates body surface area. Parietal peritoneum comprises of 20% of
total surface area and visceral peritoneum 80%.

Answer 126
Options B, C, D and E are incorrect, option A is the correct answer.
4.25% Dextrose PET.
This Patient has a Peritoneal memberane failure.
Remember rule of 4 when working for peritoneal membrane failure.
4.25% Dextrose PET (as per ISPD guidelines), 4 hrs dwell, UF>400ml.
CT abdomen with intraperitoneal contrast is done when you suspect leak.
Option D is done when you suspect volume overload.

Causes of volume overload in PD


Change in residual kidney function
Change in transport
Compliance
Mechanical
new comorbid condition

Causes of too little out


Loss of residual renal function
Mechanical
Poor PD Ultrafiltration.

Definition of ultrafiltration failure


Clinical
Failure to maintain volume homeostasis despite fluid restriction and use of
>3 hypertonic exchanges /day

Laboratory
<400 ml UF(Drain volume <2400 ml after 4 hrs dwell with 2 L- 4.25% or
3.86% dialysate

Return to contents
Questions 127 & 128 Q
Question 127
A 20-year-old male visited to your clinic for evaluation of recurrent urinary
tract infections and currently he is on prophylaxis for urinary tract infections.
Ultrasound of kidneys was done recently and he was told stones in the kidney.
His father is diagnosed for kidney stones. CT scan with stone protocol
was done.

Which one of the following is the most likely diagnosis?

A Acute pyelonephritis
B Chronic pyelonehritis with right atrophic kidney
C Stones obstructing the left kidney
D Emphysematous pyelonephritis
E None of the above.

Question 128
Which one of the following is the most common cause of death in patients
whose hypertension is under control?

A Aortic dissection
B Coronary artery disease
C Left ventricular dysfunction
D Renal failure
E Stroke

Return to question 1 Return to contents


A nswer
127. B - Chronic pyelonehritis
with right atrophic kidney
128. B - Coronary artery disease

Answer 127
Options B, C, D and E are incorrect, option A is the correct answer.
Chronic pyelonehritis with right atrophic kidney Patient has right kidney
atrophic secondary to chronic pyelonephritis most likely the etiological factor
is stones but it does not shows the features of obstruction. Emphysematous
pyelonephritis is more common in diabetic patients and radiology findings
does not fit with it.

Answer 128
Options A, C, D and E are incorrect, option B is the correct answer.

Coronary artery disease


Coronary artery disease is the most common cause of death in patients whose
hypertension is under control. Hypertension is the strongest risk factor for
stroke, and 1 of the 3 main risk factors for coronary atherosclerosis (the other
risk factors are smoking and hyperlipidemia). Hypertension can also lead to
renal failure at a young age. Uncontrolled hypertension causes increased
left ventricular pressure, which over time can cause cardiomegaly and left
ventricular dysfunction.

Return to contents
Questions 129 & 130 Q
Question 129
A 39 year old male of Asian origin is known to have kidney stone disease, he's
admitted to accident and emergency with a right renal angle pain. CT scan with
a stone protocol is done. See the image below.

Which one of the following is the most likely diagnosis in this patient?

A Calcium phosphate stone


B Staghorn stone
C Uric acid stone
D Xanthine stone.
E Oxalate stone

Question 130
KDIGO Guidelines recommends that all of the following drugs be
contraindicated in the management of lupus nephritis in pregnancy except?

A Mycophenolate mofetil
B Hydroxychloroquine
C Cyclophosphamide
D ACE inhibitors
E ARB (angiotensin receptor blockers)

Return to question 1 Return to contents


A nswer
129. C - Staghorn stone
130. B - Hydroxychloroquine

Answer 129
Option A, B, D and E are incorrect, option C is the correct answer.

Staghorn stone

The CT scan image is showing a staghorn stone (also known as struvite stone)
at the pelvi-ureteric junction of the right kidney, obstructing the kidney.
Staghorn stones are often caused by infections (urease splitting organisms) and
they most commonly occur after a urinary tract infection that has lasted a long
time. Struvite stones are more common in women than in men.

Answer 130
Hydroxychloroquine

KDIGO guidelines suggest that women be counselled to delay pregnancy until


a complete remission of lupus nephritis has been achieved.
Data suggest that active Lupus nephritis( LN) or LN in partial remission is
associated with an increase in fetal loss and an increased rate of kidney relapse
during pregnancy.

Options A,C,D,E are teratogenic.


Hydroxychloroquine, azathioprine, and corticosteroids have been used safely
during pregnancy in patients with systemic lupus; low-dose aspirin may
decrease fetal loss in systemic lupus.

Return to contents
Questions 131 & 132 Q
Question 131
Which one of the following statements regarding the use of digoxin in dialysis
patients with atiral fibrillation is correct?

A. Digoxin use in incident dialysis patients is associated with increased


mortality when pre-dialysis potassium levels are <3.3 mEq/L
B. Digoxin use in incident dialysis patients is associated with increased
mortality when pre-dialysis potassium levels are <4.3 mEq/L
C. Both digoxin and potassium compete for the same Na+/Cl- transporter on
the cardiac myocyte
D. Hyperkalemia potentiates digoxin toxicity
E Digoxin use is contraindicated in dialysis patients

Question 132
A 54-year-old black man admitted with mild dry eyes, dry mouth, diffuse
abdominal pain, diffuse lymphadenopathy, arthralgia and acute kidney injury
Labs shows

Na 136 Calcium 9.8


K 3.1 Amylase 750(Normal<250)
Cl 110 C3 65(Normal>80)
Hco3 16 C4 14(Normal>20)
Blood urea 25 ANA 1:40
Serum Creatinine 300 CBC 16% Eosinophils

Kidney biopsy shows diffuse interstitial nephritis

Which one of the following is the most likely diagnosis in this patient?
A Sarcoidosis
B Sjogreen Syndrome
C Ig4 related Systemic disease
D TINU(Tubulointersitial nephritis uveitis)
E SLE

Return to question 1 Return to contents


A nswer
131. C - Both digoxin and potassium
132. C - Ig4 related Systemic disease

Answer 131
Options A, B, D and E are incorrect. Option C is the correct answer. Both
digoxin and potassium compete for the same Na+/Cl- transporter on the
cardiac myocyte. Digoxin is commonly used in dialysis patients with heart
failure and/or atrial fibrillation. However, caution is advised while prescribing
this medicine. In a retrospective study by Chan et al, digoxin use in incident
dialysis patients was associated with increased mortality, especially in patients
with pre-dialysis potassium levels less than 4.3 mEq/l. Both potassium and
digoxin compete for the Na+/K+-ATPase on the cardiac myocytes, causing
hyperkalaemia to decrease the effectiveness of the drug and hypokalaemia
to potentiate its toxicity.

Answer 132
Options A, B, C and D are incorrect. Option C is the correct answer.

Ig4 related Systemic disease


IgG4 Related systemic disease
New Syndrome IgG4 RSD (Related Systemic Disease) Initially
diagnosed as a cause of autoimmune pancreatitis (AIP) Now known to
cause systemic organ dysfunction
Pathology Intense plasma cell infiltration An expansile, destructive
fibroinflammatory process IgG4 granular staining along the TBM 83%
withextra-renal organ involvement

Common Presenting Clinical Findings


Middle age / older malePancreatitis / Pancreatic mass Salivary gland
mass / insufficiency Generalized lymphadenopathy (80%)
Hypothyroidism Pneumonitis

Laboratory Findings
Elevated IgG4 60% of all patients 5% incidence in the normal population
Hypocomplementemia (C3 or C4) 56-78%
Peripheral Eosinophilia 33-48%
Positive ANA 30%
Parenchymal lesions 78%
Treatment
90% Steroid responsive
Ref: Raissian Y.J Am Soc Nephrol 2011 Jul;22(7):1343-52
Cornell L. Curr Opin Nephrol Hypertens. 2012 May;21(3):323-8

Return to contents
Question 133 Q
An 80-year-old man was brought to the emergency department due to severe
agitation and vomiting which was worsening over the last two weeks. He
developed stroke 6 months ago and was bed bound. Previous medical history
of Diabetes mellitus and Hypertension, which was reasonably, controlled.
On examination, he is conscious but agitated, afebrile with blood pressure
of 110/70. He has some residual weakness of right side.

Laboratory investigations showed. Blood Urea 21, Serum Creatinine 110


umol/L, plasma Na 168 and K 4.5.

CT.Scan of brain was normal. Fluid Deficit was calculated by emergency


physician to correct hypernatraemia.

Patient started on IV 0.45% (half normal saline)

What should be the rate of correction of IV Fluid to avoid complications


in this patient?

A 0.25 mmol/hour
B 0.5 mmol/hour
C 0.75 mmol/hour
D One mmol/hour
E Two mmol/hour

Return to question 1 Return to contents


A nswer B - 0.5 mmol/hour

Options A, C, D and E are incorrect. Option B is the correct answer. In case of


hypernatremia rapid correction more than 0.5 mmol/hour causes seizures and
cerebaral edema, ideally the correction rate is 0.5 mmol/hour for next 48 hours
and daily correction targeted not more than 8-12 mmol/day.
Hypernatraemia

ECFV increased Salt excess


salt ingestion
hypertronic NaCI
Insufficient water intake hypertonic Na bicarb

ECFV
Decreased Normal
Hypotonic fluid defecit Pure Water defecit
Urine sodium
Urine osmolality
urine osmolality
>20 mEq/L <10mEq/L <700 mOsm/L >700 mOsm/L
<700 mOsm/L >700 mOsm/L
Renal hypotonic Extrarenal hypotonic Renal water loss Extrarenal water loss
fluid loss fluid loss nephrogenic DI or hypodipsia
diuretics Skin Central DI insensible loss
Osmotic diuresis GIT Prmary hypodipsia
Secondary hypodipsia
The main causes of Hypernatraemia are:
Reduced water intake (e.g. coma, dysphagia, extreme depression). Because
hypernatraemia is an extremely strong stimulus to thirst, reduced water intake
is almost always involuntary. Increased losses of hypotonic fluid. Usually both
are present, though either alone can be sufficient.increased loss via gut, skin or
respiratory tract. Cholera syndromes (likely to be sodium depleted too); severe
sweating, etc Increased loss in urine caused by impaired ability to concentrate
urine (diabetes insipidus, central, nephrogenic or drug-induced) If circulating
volume is reduced hypernatraemia is exacerbated by reabsorption of sodium
due to aldosterone secretion.
Clinical features include thirst, oliguria and concentrated urine. More severe
cases may result in confusion and weakness, and possibly tachycardia, and finally
hypotension and coma. Plasma urea is usually increased. These findings, along
with a urine osmolality of over 600 mosm/kg confirm water depletion.
Treatment is by oral replacement of water in mild cases; 5% dextrose (i.v.) in
moderate cases; and a combination of 5% dextrose and 0.9% (150mmol/l) saline
(i.v.) if dehydration is severe, as volume expansion requires salt as well as water.
Return to contents
Question 134 Q
A 26-year-old man was referred to the renal clinic with a 3 month history of
tiredness, weight gain and leg swelling. Dipstick urinalysis showed 4+protein
and 3+ blood. He had no previous medical problems and was taking no
medication. On examination (see image 1 & 2) his blood pressure was 160/95
mmHg and he had pitting oedema in his legs up to mid-thigh level.
Investigations:
haemoglobin 12.1g/L (13 - 18)
serum urea 8.1 mmol/L (2.5 - 7)
serum creatinine 130 umol/L (60 - 110)
serum albumin 27g/L (37 - 49)
serum cholesterol 8.0 mmol/L (5.2)
urine protein:creatinine ratio 980mg/mmol (<2.5)

All serology and vasculitis screen was normal.


A renal biopsy was undertaken. The electron microscopy (see image 3) findings
are shown below.

Which one of the following is the most


likely diagnosis in his clinical scenario?

A Post streptococcal Glomerulonephritis


B Diabetic Nephropathy
C MPGN Type 2(Dense deposit disease)
D Lupus Nephritis
E Light chain deposit disease

Return to question 1 Return to contents


A nswer C - MPGN Type 2 (Dense deposit disease)

Options A, B, D and E are incorrect, option C is the correct answer.


Image 1. (Face) is showing a partial lipodystrophy (Loss of subcutaneous fat
over cheeks bone), which is seen in MPGN Type2. (Dense deposit disease)
Image 2. (Fundus) showing Drusen, which is associated with MPGN Type 2.
Image 3. (electron microscopy) shows a characteristic Ribbon like distribution
of dense deposits.

Dense deposit disease (DDD)/membranoproliferative glomerulonephritis


type II (MPGNII) is characterized by proteinuria, acute nephritic syndrome, or
nephrotic syndrome. It most frequently affects children between ages five and
15 years. DDD/MPGNII can be associated with acquired partial lipodystrophy
(APL). Drusen, whitish-yellow deposits within Bruch's membrane of the retina
often develop in the second decade of life; they initially have little impact on
vision, but cause vision problems from subretinal neovascular membranes,
macular detachment, and central serous retinopathy in about 10% of
affected individuals.

The definitive diagnosis of DDD requires renal biopsy.

Kidney biopsy finding of Dense Deposit Disease

Light microscopy Immunofluorescence Electron


Microscopy
Dense Deposit Disease Varied appearance Granular capillary Glomerular
(Membranoproliferative of glomeruli ranging loop staining for basement
glomerulonephritis from mesangial complement 3 membrane
Type II) hypercellularity,
ribbon like
membranoproliferative
very dense
glomerulonephritis
to endocapillary deposits
proliferation.

Prognosis in DDD not the best. Spontaneous remissions are uncommon and
about 50% of affected individuals develop end-stage renal disease (ESRD)
within ten years of diagnosis.

Return to contents
Questions 135 & 136 Q
Question 135
An 81-year-old Caucasian man with a history of diabetes, coronary artery
disease, hypertension, and proteinuria is treated by a general practitioner
with an angiotensin-converting enzyme inhibitor (ACEI) along with other
medications that are appropriate for his medical conditions.

The patient is referred to you and brings an Internet printout that refers to the
risks and benefits of combining the ACEI with an angiotensin receptor
blocker (ARB).

Which one of the following statements is correct for describing the effects of
the combination of ACEI and ARB compared with ACEI treatment alone?

A The risk for cardiovascular death will be reduced.


B There will be an increase in risk for allergic reactions.
C The rate of renal disease progression will be slowed.
D There is increased risk for hypotension and acute renal failure.
E There will be a reduction in risk for sudden cardiac death

Question136
A 35-year-old male admitted with nephrotic range proteinuria and microscopic
hematuria. All serologies results are negative.

A Kidney biopsy is done

Immunofluorescence microscopy showed the staining pattern seen in image for


IgG(3+), IgA(3+), kappa light chain(3+), lambda light chain(3+), C3(2+), C1q(0).

Which one of the following is the most


likely diagnosis in his clinical scenario?
A Lupus nephritis, WHO Class II
B Post-infectious
glomerulonephritis
C Membranoproliferative
glomerulonephritis.
D IgA nephropathy
E Hepatitis C infection

Return to question 1 Return to contents


A 135. D - There is increased risk for hypotension
nswer 136. D - IgA nephropathy

Answer 135
Options A, B, C and E are incorrect.
Option D is the correct answer. There is increased risk for hypotension and
acute renal failure.

Telmisartan Alone and in combination with Ramipril (ONTARGET) study


indicated that with combination therapy compared to single agent treatment,
the risk for cardiovascular death will not be reduced (choice A), there will not
be a reduction in risk for sudden cardiac death (choice E), or that the rate of
renal disease progression will be slowed (choice C). There is no evidence that
there will be an increase in risk for allergic reactions with combined treatment
(choice B). But the studies do indicate that combination therapy does increase
risk for hypotension and acute renal failure (choice D).

Answer 136
Options A, B, C and E are incorrect, option D is the correct answer.
IgA nephropathy.
The image shows a granular mesangial pattern of staining.
IgA and IgG can be codominant in IgA nephropathy. Absence of C1q staining
and negative serologies are supportive of IgA nephropathy. Lupus nephritis
would show a full house pattern and C1q would be positive.

Patterns of immunofluorescence for exams

Return to contents
Question 137 Q
A 71-year-old man who received a kidney transplant for end-stage kidney
disease due to diabetes mellitus is evaluated 2 weeks after receiving treatment
for acute gout of the left metatarsophalangeal joint with a course of oral
corticosteroids. He is now asymptomatic. He has a history of gout, previously
averaging one flare per year, but over the last 3 months he has experienced
three episodes of symptomatic gout involving his hands, knees, and feet, most
recently 1 month ago. He does not drink alcohol. Medications are tacrolimus,
mycophenolate mofetil, aspirin, losartan, and atorvastatin.

On physical examination, there is no active synovitis. Tophi are seen


on the right second and third fingers.

Laboratory studies reveal a stable serum creatinine level of 430umol/L, eGFR


of 20 ml/min per 1.73 m2, and a serum uric acid level of 511 umol/L. (Normal
range British Society of Rheumatologist (BSR) <300 umol/L)

Which one of the following is the most appropriate treatment for this patient?
A Change losartan to enalapril
B Allopurinol 50 mg daily
C Change tacrolimus to cyclosporine
D Colchicine 0.6 mg daily
E Probenecid

Return to question 1 Return to contents


A nswer B - Allopurinol 50 mg daily

Options A, C, D and E are incorrect. Option B is the correct answer.


Allopurinol 50 mg daily
This patient has hyperuricemia and polyarticular gout complicated by tophi
now flaring with increasing frequency. Urate-lowering therapy is indicated to
decrease progressive joint destruction and episodes of symptomatic gout. The
American College of Rheumatology (ACR) recommends the use of the xanthine
oxidase inhibitors allopurinol or febuxostat as the first-line urate-lowering
approach in gout. In the setting of kidney transplantation, simultaneous use of
both allopurinol and febuxostat in patients receiving azathioprine should be
avoided because of enhanced risk of myelosuppression. Patients maintained
on azathioprine requiring xanthine oxidase inhibitor therapy should generally
be switched to mycophenolate mofetil.
Allopurinol should be started at a dose of 50 mg daily in stage G4 or greater
CKD, titrating the dose by 50-mg increments every 34 weeks until the
serum uric acid level is <356 umol/L. A maximum dose of 200 mg is often
recommended when the eGFR is <20 ml/min per 1.73 m2. However, the ACR
has suggested that the dose can be increased to >300 mg daily, even with
renal impairment, with close monitoring for pruritus, rash, increases in liver
chemistries, or myelosuppression. Pegloticase, recombinant urate oxidase, is
recommended when there is severe gout that is refractory to urate-lowering
therapy or for individuals who are intolerant of urate-lowering agents.
Probenecid, a uricosuric agent, is unlikely to significantly lower the serum
uric acid level in this patient and is not recommended when the creatinine
clearance is <50 ml/min or in patients with a history of nephrolithiasis.
Colchicine is an anti-inflammatory agent that has increased potential for
toxicity in patients with CKD and when used following transplantation.
Given the increased serum uric acid level and recurrent gout in this patient,
colchicine would not be the first choice of therapy. For gout flare prophylaxis
in such a patient, a dose of 0.3 mg daily, not 0.6 mg daily as indicated, or
less is recommended. Concurrent therapy with the p-glycoprotein inhibitors
tacrolimus and cyclosporine inhibits the metabolism of colchicine, increasing
the risk for myoneuropathy, and close monitoring for related symptoms is
warranted.
The ACR has in fact recommended addition of losartan as one of several
possible uricosuric agents when the target serum uric acid level has not been
attained with xanthine oxidase inhibition.
Cyclosporine is associated with an increased risk of gout relative to tacrolimus,
and thus a change from tacrolimus to cyclosporine would not be justified.

Return to contents
Question 138 Q
A 20-year-old man is referred to the outpatient clinic for evaluation of
hypertension. He has no siblings. His father was diagnosed with hypertension
at a young age.
He does not take any medications. On physical examination, his blood pressure
is 184/90 mmHg, and his heart rate is 88 beats/min. His weight is 60 kg. The
physical examination is otherwise normal.
Laboratory data show a haemoglobin level of 14.2 g/dl, serum creatinine level
of 90 umol/L, serum potassium level of 4.9 mEq/L, supine normal sodium
diet serum aldosterone level of 487 pmol/L (reference range, 50138 pmol/L,)
seated morning plasma renin activity of 0.1 nmol/L per hour [reference range,
0.83 nmol/L per hour)].
Direct genetic testing confirms the presence of hybrid 11 -hydroxylase and
aldosterone synthase genes.

Which ONE of the following is the BEST initial treatment for this patient?
A Hydralazine
B Amlodipine
C Prednisone
D Low sodium diet
E Bendroflumethiazide

Return to question 1 Return to contents


A nswer A - Severe tubulitis

Options A,B,D &E are incorrect. Option C is the correct answer.


This patient has glucocorticoid remediable aldosteronism (GRA), and treatment
with prednisone is the best therapy for this condition. GRA is a rare autosomal
dominant form of hyperaldosteronism in which aldosterone hypersecretion can
be reduced with physiologic doses of glucocorticoids. Patients with GRA have
adrenocorticotrophic hormone (ACTH)-sensitive aldosterone production in the
zona fasciculata of the adrenal gland. The mutation in patients with GRA is a
fusion of the promoter region of the gene for CYP11B1 (11-hydroxylase) and
the coding sequences of CYP11B2 (aldosterone synthase), resulting in ACTH-
dependent activation of the aldosterone synthase. GRA should be suspected
in individuals with onset of hypertension under the age of 21, particularly when
there is a consistent family history. Onset of hypokalaemia after initiation of a
thiazide diuretic is another important clue to the diagnosis.
The plasma potassium level is normal in >50% of cases of GRA. It has been
proposed that this finding is secondary to the normal circadian rhythm of ACTH
release, which leads to an increase in aldosterone secretion above normal for
only part of the day. The plasma aldosterone level is typically modestly elevated,
and plasma renin activity is suppressed. However, the aldosterone:renin
ratio is typically not as high as with primary aldosteronism caused by an
adenoma. Genetic testing for the chimeric gene has eliminated the need for
the dexamethasone suppression test. The primary therapy for this condition
is the use of the smallest possible dose of glucocorticoid to suppress ACTH
secretion and limit the stimulation of aldosterone while at the same time striving
to avoid iatrogenic Cushings syndrome. Treatment with thiazide diuretics may
cause marked hypokalaemia secondary to the increased sodium delivery to the
aldosterone-sensitive potassium secretory site in the cortical collecting duct.
Alternatively, treatment with mineralocorticoid receptor antagonists or amiloride
can be used, but these agents are typically used only if hypertension cannot be
controlled with glucocorticoids alone.
The use of Calcium channel blockers and hydralazine does not address the
pathophysiologic defect .
A low sodium diet may help but is unlikely to be sufficient to control
hypertension in these patients.
It is also worth noting that patients with GRA are at elevated risk for cerebral
aneurysms, and it has been recommended that patients with genetically proven
GRA should undergo screening MR angiography at puberty and every 5 years
thereafter.
References
Rich GM, Ulick S, Cook S, Wang JZ, Lifton RP, Dluhy RG: Glucocorticoid-remediable aldosteronism in
a large kindred: Clinical spectrum and diagnosis using a characteristic biochemical phenotype. Ann
Intern Med 116: 813820, 1992

Return to contents
Question 139 Q
A 78-year-old man was found collapsed on the floor by his carer after an
overnight mechanical fall. He had a background history of Hypertension and
dyslipidaemia. He was on amlodipine, Lisinopril and atorvastatin therapy.

Investigations

Serum sodium 157 mmol/L 135-145


Serum Potassium 6.0 mmol/L 3.5-5.5
Blood urea 9.6 mmol/L 2.5-7.8
Serum creatinine 371 umol/L 60-115
Adjusted calcium 2.81 mmol/L 2.2-2.6
CK Raised

Which one of the following metabolic changes is seen in the early phase of
Rhabdomyolysis?

A Hypokalaemia
B Hypocalcaemia
C Hypercalcaemia
D Hypophosphataemia
E Hyperuricaemia

Return to question 1 Return to contents


A nswer B - Hypocalcaemia

Options A, C, D and E are incorrect, option B is the correct answer. This


patient has Rhabdomyolysis. Hypocalcaemia occurs in the early phase of
rhabdomyolysis because of an influx of calcium into a damaged myocytes,
which is then deposited. Decreased bone responsiveness to the PTH has
also been reported in patients with rhabdomyolysis. During the recovery
phase, serum calcium levels return to normal and may rebound to significantly
elevated levels due to the release of calcium from injured muscle, mild
secondary hyperparathyroidism from the acute renal failure, and an increase in
calcitriol.
Other manifestations of rhabdomyolysis include fluid and electrolyte
abnormalities, many of which precede or occur in the absence of acute kidney
injury, and hepatic injury. Hypovolemia, hyperkalaemia, hyperphosphatemia,
hypocalcaemia, hyperuricemia, and metabolic acidoses may be seen.
Hyperkalaemia may result in cardiac dysrhythmias. Later complications include
acute kidney injury (AKI), hypercalcaemia, compartment syndrome, and, rarely,
disseminated intravascular coagulation.1,25-dihydroxyvitamin D).

Ref:
1. Llach F, Felsenfeld AJ, Haussler MR. The pathophysiology of altered calcium metabolism
in rhabdomyolysis-induced acute renal failure. Interactions of parathyroid hormone,
25-hydroxycholecalciferol, and 1,25-dihydroxycholecalciferol. N Engl J Med 1981; 305:117.
2. Akmal M, Bishop JE, Telfer N, et al. Hypocalcemia and Hypercalcaemia in patients with
rhabdomyolysis with and without acute renal failure. J Clin Endocrinol Metab 1986; 63:137.

Return to contents
Question 140 Q
A 32-year-old man attends the clinic for an evaluation of kidney cysts and
haematuria. He was in his usual state of health until he helped friends move
furniture. He subsequently developed right flank pain and haematuria. His
family history is unknown as he was adopted.
He is on no medications. On physical examination, the blood pressure
is 135/80 mmHg and his heart rate is 70 beats/min. The remainder
of the examination is unremarkable.
Laboratory data show a serum creatinine level of 90 umol/L. Urinalysis shows a
pH of 5.5, 2+ blood, no protein, and 610 nondysmorphic red blood cells/HPF.
Kidney ultrasound shows enlarged kidneys (14 cm, right kidney; 13.7 cm, left
kidney) with five cysts in the right kidney and four cysts in the left kidney.
No kidney stones are seen. There is no hydronephrosis.

Which one of the following is the most appropriate next step in the diagnostic
evaluation of this patient?

A A volumetric MRI for measuring total kidney volume


B No further testing is necessary; he has autosomal dominant polycystic kidney
disease (ADPKD) because he has at least two cysts per kidney
C Genetic testing for PKD1 and PKD2 mutations
D Computed tomography (CT) scans or magnetic resonance
imaging (MRI) of the abdomen to evaluate for kidney and liver cysts
E Repeat kidney ultrasound

Return to question 1 Return to contents


A nswer D - Computed tomography

Options A, B, C and E are incorrect, option D is the correct answer.

Computed tomography (CT) scans or magnetic resonance imaging (MRI) of


the abdomen to evaluate for kidney and liver cysts

Further testing first with imaging of the kidneys and liver with CT or MRI is
indicated for this patient. An ultrasound finding of at least two or more cysts per
kidney is sufficient to make a diagnosis of ADPKD in patients with a family history
of ADPKD by the Ravine Criteria or by the revised diagnostic criteria. However,
these criteria do not apply to patients without a documented family history of
ADPKD. In this patient with an unknown family history, more evidence is required
to make the diagnosis. Ten cysts per kidney would help establish the diagnosis.
Other findings of ADPKD, such as liver cysts, may be helpful in establishing the
diagnosis. Total kidney volume is useful for monitoring progression of ADPKD
but not for establishing a diagnosis.
Genetic testing may be considered, but testing may not always be fully
informative. In some cases, no mutation is found, or a new mutation is found
that may or may not be known to be pathogenic. Furthermore, in the absence of
available affected family members, the easier, linkage analyses cannot be done.
Repeat ultrasound measurements or volumetric MRI may be in following
progression of ADPKD.

Genes and Proteins od Autosomal Polycystic Kidney Disease (ADPKD)

Disease Frequency Chromosomes Gene Locus Protein Function


ADPKD 1:1000 16p13.3 PKD1 Polycystin Regulates
1, which intracellular
colocalizes cAMP,
with polycystin mTOR,
2 in the planar
primary cilium polarity

1:15,000 4q21.2 PKD2 Polycystin Regulates


2, which intracellular
colocalizes Ca
with polycystin levels
1 in the through ER
primary cilium Ca release,
activates Ca
and ER
channels

Return to contents
Question 141 Q
A 55-year-old man with a history of alcohol dependence is admitted for
alcohol detoxification. He develops symptoms of alcohol withdrawal 1
day into his hospitalization and is treated with intravenous diazepam, with
dosing based on the Clinical Institute Withdrawal Assessment (CIWA) score.
Three days into the hospitalization, he develops worsening confusion and
hypoxia. Medications are subcutaneous heparin, multivitamin, intravenous
thiamine, and intravenous diazepam. On physical examination, his blood
pressure is 102/75 mmHg, pulse is 106 beats/min, temperature is 36.3C,
and respiratory rate is 15 breaths/min. He is hypersomnolent, inattentive,
and disoriented to time and place.
The remainder of the examination is normal.
Laboratory Admission Hospital day 3
Serum
Sodium (mEq/L) 152 147
Potassium (mEq/L) 3.6 4
Chloride (mEq/L) 121 98
Total CO2 (mEq/L) 26 20
Blood urea (mmol/L) 33 25
Creatinine (umol/l) 97 129
Glucose (mmol/l) 3.8 10
Albumin (g/dl) 3.5
Osmolality (mOsm/kg) 336 415
Lactate (mEq/L) - 7
Ethanol (mg/dl) 74 0
Ketones Negative Negative
Arterial blood gas (ambient air)

pH - 7.28
PCO2 (mmHg) - 45
PO2 (mmHg) - 85

Which one of the following is the most likely cause of this patients metabolic
abnormalities?
A Sepsis
B Alcoholic ketoacidosis
C Ethylene glycol
D Diazepam infusion
E Isopropyl alcohol

Return to question 1 Return to contents


A nswer D - Diazepam infusion

Options A, B, C and E are incorrect, option D is the correct answer.


Diazepam infusion.

High-dose diazepam infusion can cause propylene glycol toxicity, which is


the most likely cause of this patients increased osmolal gap and increased
anion gap metabolic acidosis. Propylene glycol is used as a vehicle for several
pharmaceutical preparations, including lorazepam, phenobarbital, diazepam,
and etomidate. Propylene glycol is metabolized by alcohol dehydrogenase
into both the D- and L-isoforms of lactic acid. Toxicity of propylene glycol
manifests as hyperosmolarity, lactic acidosis, and in some cases AKI. This
patient developed hyperosmolarity (osmolal gap, 108 mOsm/kg) and
increased anion gap metabolic acidosis from the accumulation and toxic
effects of propylene glycol.
Propylene glycol should be considered in the differential diagnosis when there
is both an increased anion gap metabolic acidosis and an increased osmolal
gap in individuals receiving infusions in which propylene glycol is used as a
solvent. The metabolic abnormalities usually resolve with discontinuation of the
offending agent, but renal replacement therapy may become necessary when
there is severe metabolic acidosis or severe AKI. The role of fomepizole
is unproven in propylene glycol toxicity.
Although this patient is at high risk for sepsis, the rise in the serum lactate
would be insufficient to explain the magnitude of the increased osmolal gap
in this patient.
Toxic ingestions of methanol and ethylene glycol can also cause increased
anion gap metabolic acidosis and an elevated osmolal gap. Although the
osmolal gap was increased on admission, these findings can be accounted for
by the elevated ethanol level. Ingestion of ethylene glycol usually presents with
an increased osmolal gap, often >25 mOsm/kg. Ethylene glycol is metabolized
to glycolic acid, causing increased anion gap metabolic acidosis. The increased
osmolal gap resolves once the parent alcohol is fully metabolized and therefore
may not be present in patients who present late. The onset of the increased
osmolal gap in this patient while hospitalized is not consistent with the
expected time course of a declining osmolal gap and worsening increased
anion gap metabolic acidosis characteristic of ethylene glycol toxicity.
Isopropyl alcohol toxicity may give rise to an increased osmolal gap, but would
not account for the increased anion gap metabolic acidosis.

Return to contents
Question 142 Q
A 37-year-old man with diabetes mellitus complicated by microalbuminuria
is seen for a routine follow-up and is found to have asymptomatic
Hypercalcaemia.

His laboratory data are as follows:


24-hour urine collection
SSerum studies
Calcium 70 mg
Na 140 mEq/L
Creatinine 1.4 g
Potassium 4.3 mEq/L
Creatinine 114 umo/L
Calcium 3 mmol/L
Albumin 4.0 g/dl
Phosphorus 0.77 mmol/L
Intact parathyroid hormone (PTH) 110 pg/ml
25-OH vitamin D 48 ng/ml

Which one of the following is the most likely diagnosis in his clinical scenario?

A 1,25-dihydroxyvitamin D production from granulomas


B Familial hypocalciuric Hypercalcaemia
C Adenoma of the parathyroid gland
D Activating mutation of the calcium-sensing receptor
E Secondary hyperparathyroidism

Return to question 1 Return to contents


A nswer B - Familial hypocalciuric Hypercalcaemia

Options B, C, D and E are incorrect, option B is the correct answer.


Familial hypocalciuric Hypercalcaemia.
This patient likely has familial hypocalciuric Hypercalcaemia.

The elevated serum calcium with an unsuppressed PTH level in this patient
suggests that PTH secretion is not appropriately regulated by systemic calcium
levels. The differential diagnosis includes autonomous secretion of PTH by
an adenoma or a defect in the calcium-sensing receptor. These entities can
be differentiated by an assessment of urinary calcium excretion. Primary
hyperparathyroidism is typically confirmed by high urine calcium and/or a
high urine Ca/Cr clearance. The latter can be calculated as: (24-h Urine Ca
Serum Cr)/(24-h Urine Cr Serum Ca)and is generally >0.02 in primary
hyperparathyroidism. This patient, however, has hypocalciuria and a urine Ca/
Cr clearance of <0.01, indicating avid renal reabsorption of calcium, despite
systemic Hypercalcaemia. This rare autosomal dominant condition, caused
by an inactivating mutation of the calcium-sensing receptor, is referred to as
familial hypocalciuric Hypercalcaemia. The serum phosphate level in this entity
is typically normal or mildly decreased.
Secondary hyperparathyroidism occurs in CKD when the GFR falls below the
6070 ml/min per 1.73 m2 range. A rise in the serum PTH level to 110 pg/ml
would be unusually high at this patients level of kidney function in the absence
of a concomitant condition such as vitamin D deficiency. Laboratory findings
in early secondary hyperparathyroidism include normal to low serum calcium
levels and not Hypercalcaemia as noted in this patient. Serum phosphorus
levels are typically normal to increased and not low as in this case. No obvious
secondary causes are evident in this patients laboratory data, and the unusual
elevation in serum calcium level should prompt an evaluation for alternative
explanations for the elevated PTH. Hypercalcaemia may complicate secondary
hyperparathyroidism, but this usually occurs in patients with advanced CKD
or end-stage kidney disease. Common causes in these settings include use
of vitamin D sterols or calcium-based phosphate binders, as well as severe
parathyroid hyperplasia and markedly increased serum PTH levels,
usually >800 pg/ml.
Extrarenal production of vitamin D would cause Hypercalcaemia with a
suppressed PTH level.

Return to contents
Question 143 Q
A 59-year-old man is seen in a follow-up for management of increasing ascites
and oedema complicating alcohol-related liver disease and cirrhosis. He has
been sober for the last 4 months and is undergoing evaluation for a possible
future liver transplantation.
Medications are spironolactone 100 mg daily and furosemide 40 mg daily, and
the patient states that he has remained adherent with this regimen. Despite
this, the patient notes increased ascites, leg oedema, and has gained 3 kg in
weight over the last month.
On physical examination, he is afebrile, with a blood pressure of 98/50
mmHg, pulse of 90 beats/min, respiratory rate of 16 breaths/min, and oxygen
saturation of 92% on room air. He is alert and oriented.
The abdomen is nontender, and there is moderate ascites.

There is 1+ pretibial edema bilaterally.


Laboratory data

Serum Urine
Na 134 mEq/L Na 102 mEq/L
K 3.9 mEq/L K 60 mEq/L
Chloride 108 mEq/L
Total CO2 22 mmol/L
Blood urea2.14 mmol/L
Creatinine 53 umol/L

Which one of the following is the most appropriate next step in treating this
patient?

A Restrict fluid to<1L/day


B Admit for albumin infusions
C Increase spironolactone
D Add a thiazide diuretic
E Dietary sodium restriction

Return to question 1 Return to contents


A nswer E - Dietary sodium restriction

Options A, B, C and D are incorrect, option E is the correct answer.


Dietary sodium restriction.

The urine sodium level >78 mEq/L and urine sodium:potassium ratio >1
indicates excessive dietary sodium intake in this patient that likely exceeds the
recommended 88 mEq/d (2000 mg). Dietary sodium restriction would therefore
be the next best step in managing this patients ascites, edema, and positive
sodium balance.
The management of ascites and edema in cirrhotic patients that have low
effective circulating volume can be a complicated matter. Careful choices
must be made about the use of diuretics (or combination of diuretics) and the
extent of diuresis necessary to balance alleviation of ascites with preservation
of renal perfusion. Patients with detectable ascites and edema require diuretic
therapy in combination with dietary sodium restriction. Diuretic therapy should
generally include an aldosterone antagonist such as spironolactone plus a
loop diuretic. The loop diuretic not only promotes natriuresis but also counters
hyperkalaemia resulting from aldosterone antagonism.
Addition of a thiazide diuretic would be a consideration if the urine sodium
level was 78 mEq/L and the urine sodium:potassium ratio was <1. Care
must be taken to avoid hypokalaemia, which possibly can precipitate hepatic
encephalopathy. Similarly, an increase in spironolactone or furosemide could
be considered if the urine sodium:potassium ratio was <1, reflecting increased
diuretic resistance.

Fluid restriction alone would not ameliorate this patients sodium overload
and would therefore be less effective in treating this patients hypervolemia.
The degree of Hyponatraemia in this patient is not sufficient to warrant fluid
restriction to <1 L/d. Moreover, most cirrhotic patients find it difficult to adhere
to this degree of fluid restriction because of increased thirst.
Infusion of albumin can facilitate urinary sodium excretion in patients with
decreased renal perfusion and prerenal azotemia. Albumin infusions would
be indicated for the management of acute kidney dysfunction, particularly
following overdiuresis, but would not be appropriate for this patient with stable
kidney function and sodium excess, secondary to excessive dietary sodium
intake.

Return to contents
Question 144 Q
This is a kidney biopsy electron microscopy image of a 44-year-old female
patient.

Which one of the following is the most likely Diagnosis for this patient?

A -Interferon therapy
B Fabrys disease
C Post infectious glomerulonephritis
D IgA nephropathy
E Cryoglobulinemia

Return to question 1 Return to contents


A nswer A - interferon therapy.

Options B, C, D and E are incorrect, option A is the correct answer.


- interferon therapy. The image shows tubuloreticular inclusions (arrows),
which are associated with the following conditions.

1 Lupus nephritis
2 -interferon therapy
3 HIV
4 Viral infections

KDIGO for Glomerulonephritis


Do not use immunosuppressive therapy in patients:
1. SCr persistently >3.5 mg/dl (>309 mcmol/l) (or an eGFR <30 ml/min per
1.73m2) AND reduction of kidney size on ultrasound (e.g. <8 cm in length)
2. OR those with concomitant severe or potentially life- threatening infections.
(Not Graded)
3. Do not use steroid monotherapy nor MMF

Return to contents
Question 145 Q
A 55-year-old male admitted with hypertension and 6.2 gram proteinuria/day.
Kidney biopsy done, See the image.

Which one of the following is the most likely diagnosis?

A HIV
B Indinavir toxicity
C Functional and adaptive changes in the kidney
D Primary FSGS (Focal segmental glomerulosclerosis)
E Pamidronate toxicity.

Return to question 1 Return to contents


A nswer
C - Functional and adaptive
changes in the kidney

Options A, B, D and E are incorrect, option C is the correct answer.

Kidney biopsy showing a perihilar variant of focal segmental


glomerulosclerosis The peri-hilar variant is associated with
secondary causes such as reduced nephron mass, hypertension,
and other structural and functional adaptive responses.

The five morphological variants of FSGS are given in figure below.

Return to contents
A
Difference Between Primary and Secondary FSGS

Primary FSGS Secondary FSGS


Usually abrupt onset of nephrotic Less proteinuria; slow onset
syndrome
Normal-sized glomeruli, less Glomerular hypertrophy in unaffected
parenchymal atrophy glomeruli Focal interstitial fibrosis/
tubular atrophy and global
glomerulosclerosis

Diffuse global podocyte foot process Less prominent and segmental


effacement podocyte foot process

No IC, TRI, or other causes Evidence of a secondary cause


(IC, crescents, TRI, DM, Fabry, Alport,
HTN) effacement

IC-Immune complex, TRI-Tubuloreticular inclusion

Return to question 1 Return to contents


A nswer

KDIGO Guidelines for Treatment of FSGS

FSGS with non-nephrotic proteinuria.


ACE/ ARB ONLY

FSGS with nephrotic proteinuria.


Prednisone-at a daily single dose of 1 mg/kg (maximum 80mg) or
alternate-day doseof 2 mg/kg, given for a minimum of 4 weeks

FSGS with nephrotic proteinuria.


CNIs be considered as first-line therapy for patients
with relative contraindications
Intolerance to high-dose corticosteroids

Steroid-resistant FSGS
cyclosporine at 3-5 mg/kg/d in divided doses to be given for at least
4-6 months

Return to contents
Question 146 Q
A 60-year-old man is seen in the clinic for progressive CKD secondary to Biopsy
proven focal segmental glomerulosclerosis. The trends of his laboratory data
are as follows:

12 months ago Today


eGFR = 30 ml/min eGFR = 22 ml/min
Serum calcium = 2.27mmol/L Serum calcium = 2.15 mmol/:
Serum phosphorus = 1.3 mmol/L Serum phosphorus = 1.7 mmol/L
Intact parathyroid hormone
(PTH) = 90 pg/ml Intact PTH = 130 pg/ml

Which one of the following best describes fibroblast growth factor-23 (FGF-23)
metabolism in this patient?

A FGF-23 is causing increased PTH synthesis and activity


B FGF-23 production is reduced because of decreased nephron mass
C FGF-23 is increasing 1--hydroxylaseexpression
D FGF-23 levels have risen abruptly after serum phosphate levels increased
E FGF-23 resistance has developed due to reduced nephron mass and
decreased klotho expression

Return to question 1 Return to contents


A nswer E - FGF-23 resistance has developed

Options A, B, C and D are incorrect, option E is the correct answer.


FGF-23 resistance has developed due to reduced nephron mass and
decreased klotho expression.

Levels of FGF-23 increase as CKD progresses, but resistance develops as renal


mass decreases and klotho levels fall. FGF-23 has recently been recognized as
a critical mediator of phosphorus homeostasis. Produced by bone osteocytes
and osteoblasts (and not from the kidney), PTH, calcitriol, and alterations in
phosphate and calcium turnover in bone result in increased FGF-23 secretion.
Klotho, a type I membrane receptor, colocalizes with the FGF-23 receptor,
decreasing Na/Pi IIa cotransporter expression in the proximal tubule after
FGF-23 binding. As a result, there is decreased proximal tubular re-absorption
of phosphorus and increased renal phosphorus excretion. As CKD progresses,
FGF-23 levels rise, normalizing serum phosphorus levels by inducing
phosphaturia. Thus, FGF-23 levels are elevated before serum phosphorus
levels rise.

FGF-23 suppresses rather than increases PTH and 1--hydroxylase.

NKF KDOQI guidelines for Bone metabolism and disease in Chronic kidney
disease.

CKD STAGE GFR(ml/min) PTH (pg/ml) Calcium (mmol/L) Phosphorus (mmol/L)

3 30-59 35-70 Normal 0.87-1.48


4 15-29 70-110 Normal 0.87-1.48
5 and
5D <15 or 150-300 2.1-2.37 1.13-1.78
dialysis

Indications for Parathyroidectomy in CKD-Mineral Bone Disease

Tertiary or autonomous hyperparathyroidism: Failure of hyperplastic,


overactive glands to suppress adequately in response to optimal
medical treatment
Manifests clinically as increase PTH >500pg/mL, with persistent
hypercalcaemia
Phosphorus and alkaline phosphatase are usually raised

Return to contents
Question 147 Q
A 17-year-old girl was referred for investigation of intermittent visible
haematuria. On examination she appeared small for her age but there were
no other abnormal physical findings.

Investigations:

serum creatinine 98 mol/L (60110)


eGFR 89ml/min
serum corrected calcium 2.3 mmol/L (2.22.6)
serum phosphate 1.3 mmol/L (0.81.4)

X-ray KUB nephrocalcinosis

DMSA/MAG3 renogram split function R 53%, L 42%


normal handling of isotope

24-h urinary calcium 2.82 mmol (2.57.5)


24-h urinary oxalate 1.54 mmol (0.15-0.45)
24-h urinary phosphate 13.3 mol (15-50)
24-h urinary L-glyceric acid elevated

Which one of the following statement(s) is correct in her clinical scenario?

A The enzyme defect is alanine:glyoxalate aminotransferase (AGT)


B Progression to end stage renal disease is likely
C Systemic oxalosis is likely to occur
D The metabolic defect results from conversion of glyoxalate to oxalate
E Liver Kidney transplantation is treatment of choice

Return to question 1 Return to contents


A nswer
D - The metabolic defect results from
conversion of glyoxalate to oxalate

Options A, B, C and E are incorrect. Option D is the correct answer.


Diagnosis is type II primary hyperoxaluria.
Type I PHO is more severe and is caused by AGT deficiency. Glycolate
excretion is increased and as GFR falls systemic oxalosis occurs. Type II the
enzyme defect is the cytosolic enzyme glyoxalate reductase/D-glycerate
dehydrogenase which converts glyoxlate to glycolate. Patients with this
disorder excrete increased amounts of L-glyceric acid as well as oxalate.
In type I primary hyperoxaluria if pyridoxine therapy fails combined liver-kidney
transplantation may become necessary.
Type II does not usually progress to ESRD

Normal oxalate Metabolism


Protein
Oxalate metabolism Gelatin

GO Glycolate hydroxyproline
Glycolate hydroxypyruvate
oxidase
GRHPR
Glyoxylate reductase
B-6 Glyoxylate Hydroxypyruvic reductase
Glycine
Glycerate

Protein AGT
Alanine LDH
Glyoxylate Lactaye
Yeast
Amino dehydrogenase
Fungi
transferase
Oxalate Ascorbate
Arabinose
Oxalate diet
diet Ethylene glycol

Type 1 PHO - AGT deficiency. More severe phenotype


pyridoxine coenzyme for AGT.
If fails then Liver-kidney Transplant
Type II PHO - GR deficiency
unlikely to progress to ESRD

Return to contents
Question 148 Q
A 55-year-old woman undergoes a prolonged hysteroscopy complicated by
uterine perforation, extravasation of irrigant solution, and successful emergent
surgical repair. Postoperatively, she is promptly extubated, and her pain
is controlled with intravenous narcotics.

On physical examination, she is afebrile. Her blood pressure is 120/60


mmHg, with a pulse of 84 beats/min and a respiratory rate of 14 breaths/min.
She is awake, alert, and oriented; in no acute distress; and has appropriate
tenderness at the site of the surgical intervention. Her breathing is even and
unlabored, and she has no edema.
One hour after surgery, laboratory studies are repeated, and the results are
compared with her baseline data:

Preoperative laboratory data 1-Hour postoperative laboratory data


Na 136 mEq/L Na 120 mEq/L
Glucose 5.0mmol/L Blood Glucose 4.0 mmol/L
Blood urea 4.2mmol/L Blood urea 4.9mmol/Ll
Creatinine 79 umol/L Creatinine 0.7 mg/dl (62 mmol/L)
Serum osmolality 276 mOsm/kg

Which one of the following is the most appropriate treatment?

A Hypertonic (3%) saline


B Normal saline
C Close monitoring without additional intervention
D Loop diuretic
E Discontinuation of her pain medications

Return to question 1 Return to contents


A nswer
C - Close monitoring
without additional intervention

Options A, B, D and E are incorrect, option C is the correct answer.


Close monitoring without additional intervention.

No additional intervention is indicated for this patient at this time. The patient
experienced a drop in the serum sodium level after prolonged exposure to an
irrigant solution used for hysteroscopy. The clinical context provides several
possible etiologies for hyponatraemia. The use of an irrigant solution for
hysteroscopy and the increased exposure due to visceral perforation are the
most obvious, but she was also exposed to narcotic medications, pain, nausea,
and surgical stress, all of which can stimulate anti-diuretic hormone secretion.
Determining which of these is causative requires a careful review of her
examination and laboratory findings.
On physical examination, she is stable, without neurologic deficiency, and
grossly euvolemic. The drop in the serum sodium level is accompanied by a
slight reduction in the serum osmolality and, importantly, a rather impressive
increase in the osmolal gap at 27 mOsm/kg. An increase in ADH would give
Hyponatraemia with a low serum osmolality; however, it cannot account for
the osmolal gap. This raises concern for an unmeasured solute acting as an
effective osmole. It is important to note that, although this often leads to a
normal or elevated serum osmolality, glycine and sorbitol irrigant solutions are
actually hypotonic with an osmolality of 200 and 165 mOsm/kg, respectively.
In this context, hypertonic saline has been cautiously used in those with
neurologic complications and significant hypoosmolality or in those with a drop
in serum sodium level by >5 mEq/L when mental status cannot be assessed
due to anesthesia. Patients with neurologic symptoms, severe hyponatraemia,
and normal or near normal serum osmolality, however, are probably best
treated with haemodialysis. Loop diuretics have been used in those with
concomitant volume overload and pulmonary edema. However, in the absence
of neurologic symptoms, impaired kidney function, or volume overload, the
serum sodium levels will correct without intervention as the retained solute
(glycine or sorbitol) is taken up intracellularly, metabolized, and excreted.

Return to contents
Question 149 Q
A 55-year-old woman is admitted with 10 gram proteinuria.
eGFR >60ml/min. All immunology, vasculitic screen and serology results were
negative. A kidney biopsy image under polarized light is shown below.

Which one of the following is the most likely aetiology ?

A Ethylene glycol toxicity


B Light chain deposition disease
C Myoglobinuria
D Amyloidosis
E Myeloma cast nephropathy.

Return to question 1 Return to contents


A nswer A - Ethylene glycol toxicity

Options B, C, D and E are incorrect, option A is the correct answer.


The image show oxalate crystals which polarize brightly.

Causes of oxalate crystals


1. Ethylene glycol toxicity
2. Excess vitamin C intake
3. Dietary excess e.g. rhubarb, star fruit
4. Gastrointestinal bypass surgery

Urine oxalate
crystals- Envelope shape

Urinary investigations and targets for patients with stone formation

Investigations Target
Volume 2-3L/day
pH Between 5.5-7.0
Creatinine clearence Normal
Excretion
1. Calcium <0.1 mmol/kg/day
2. Sodium <100-150mmol/day

3. Uric acid
Women <4.5mmol/day
Man <4.8 mmol/day

4. Oxalate Minimize(<450umol/day)
5. Citrate Maximize(>0.6mmol/day)
6. Magnesium >3mmol/day

Values based on 24 hr urine collection.


Creatinine clearance needs to be measured to ensure that 24 hr urine
collection is adequate and the patient has good renal function.

Return to contents
Question 150 Q
A 59-year-old man attends his physician after a fall. He describes losing his
balance and stumbling into a wall before falling to the ground. He denies head
trauma or other injury from the fall. His wife notes he has become unsteady
over the last week and increasingly confused.

The physical examination is notable for a blood pressure of 118/70 mmHg


and regular pulse with a rate of 74 beats/min. On neurologic examination,
he is lethargic and has limited attention and recall. The remainder of the
examination is unremarkable.

Laboratory data

Sodium 117 mEq/l


Potassium 3.3 mEq/l
Chloride 85 mEq/l
Total CO2 25 mEq/l
Blood urea 3.9 mmol/L
Creatinine 62 umol/L
Plasma osmolality 250 mOsm/kg
Urine osmolality 400 mOsm/kg

Urinalysis: specific gravity, 1.021; pH 5; no blood; no protein; no cells;


and no casts.

Which one of the following is the most appropriate treatment in his


clinical scenario?

A Fluid restriction alone


B Tolvaptan (vasopressin receptor antagonist)
C Desmopressin
D Hypertonic (3%) saline bolus
E Transfer Patient to ICU

Return to question 1 Return to contents


A nswer D - Hypertonic (3%) saline bolus

Options A, B, C and E are incorrect, option D is the correct answer.


D - Hypertonic (3%) saline bolus

Hyponatraemia accompanying hypo-osmolality in individuals with clinically


apparent normal extracellular fluid volume reflects a state of water overload.
After excluding myxedema coma or hypocortisolism, the differential diagnosis
includes the syndrome of inappropriate antidiuresis or water intoxication. This
patients inappropriately concentrated urine with an osmolality of 400 mOsms/
kg is consistent with the syndrome of inappropriate ADH secretion (SIADH).
Treatment of symptomatic Hyponatraemia requires an increase in the serum
sodium by approximately 46 mEq/L to relieve symptoms. This can usually be
accomplished through administration of a 100-ml bolus of hypertonic saline.
Additional boluses may be required to achieve resolution of symptoms or a rise
in the serum sodium level by approximately 46 mEq/L. Notwithstanding this
small rapid increase to alleviate symptoms, most experts currently advocate
for a goal of increasing the sodium level by a cumulative total of 68 mEq in
the first 24 hours, 1214 mEq/L by 48 hours, and 1416 mEq/L by 72 hours
in patients with chronic Hyponatraemia that has become symptomatic to
avoid overly rapid correction. Limits not to be exceeded to avoid osmotic
demyelination syndrome are <10 mEq/L in the first 24 hours, <18 mEq/L by
48 hours, and <20 mEq/L by 72 hours.

Vasopressin receptor antagonists that block the ADH effect leading to urinary
dilution, loop diuretics that lower urine osmolality, and fluid restriction alone
are all treatments for SIADH, but would be most appropriate for chronic
management or patients who do not have clinically overt neurologic symptoms
indicative of increased intracranial pressure.

Desmopressin is a vasopressin analog that results in urinary concentration


and water retention, resulting in worsening water overload and Hyponatraemia,
and is therefore contraindicated.

Desmopressin has been used in combination with hypertonic saline to prevent


inadvertent overcorrection of Hyponatraemia, but its use as primary
therapy for symptomatic Hyponatraemia without hypertonic saline would not
be appropriate.

Return to contents
A
Major causes of hypernatraemia

Unreplaced water loss


(which requires an impairment in either thirst or access to water)
Insensible and sweat losses
Gastrointestinal losses
Central or nephrogenic diabetes insipidus
Osmotic diuresis
Glucose in uncontrolled diabetes mellitus
Urea in high-protein tube feedings
Mannitol

Hypothalamic lesions impairing thirst or osmoreceptor function


Primary hypodipsia
Reset osmostat in mineralocorticoid excess

Water loss into cells


Severe exercise or seizures

Sodium overload
Intake or administration of hypertonic sodium solutions

Return to question 1 Return to contents


A nswer

Causes of Hyponatraemia

Disorders in which ADH levels are elevated


Effective circulating volume depletion
True volume depletion
Heart failure
Cirrhosis
Thiazide diuretics

Syndrome of inappropriate ADH secretion, including reset osmostat


pattern
Hormonal changes
Adrenal insufficiency
Hypothyroidism
Pregnancy

Disorders in which ADH levels may be appropriately suppressed


Advanced renal failure
Primary polydipsia
Beer drinker's potomania

Hyponatraemia with normal or elevated plasma osmolality


High plasma osmolality (effective osmols)
Hyperglycemia
Mannitol
High plasma osmolality (ineffective osmols)
Renal failure
Alcohol intoxication with an elevated serum alcohol concentration
Normal plasma osmolality
PseudoHyponatraemia (laboratory artifact)
High triglycerides
Cholestatic and obstructive jaundice (lipoprotein X)
Multiple myeloma
Absorption of irrigant solutions
Glycine
Sorbitol
Mannitol

Return to contents
Question 151 Q
A 70-year-old female admitted with a 1 month history of back pain.
She took ibuprofen for last 1 month for pain relieve.
Her vital signs (including BP) were in normal range, and her physical
examination was notable only for 1-mm pitting edema to the ankles.
Laboratory workup revealed 11g/d proteinuria on 24-hour collection, Serum
Creatinine of 116 umol/l, total cholesterol of 9.0 mmo/l, and albumin of 26 g/dl.
The following tests were requested and awaiting results: serum protein
electrophoresis, urine protein electrophoresis, antinuclear antibody, C3 and C4,
hepatitis B and C,serological tests for syphilis, Myeloperoxidase (MPO) and PR3
ANCA. Renal ultrasound was unremarkable, including no evidence of renal vein
thrombosis, and ultrasound of the legs showed no deep venous thrombosis.
She was started on lisinopril, atorvastatin, and furosemide.

A Kidney Biopsy was undertaken


Look at the image below.

Which one of the following is the most likely diagnosis in this patient?

A AL Amyloidosis
B AA Amyloidosis
C Monoclonal light chain cast nephropathy
D Acute tubular necrosis
E Normal Light microscopy

Return to question 1 Return to contents


A
nswer
C - Monoclonal light chain cast nephropathy.

Options A, B, D and E are incorrect.

Kidney biopsy Image shows tubular casts with multi nucleated giant cell
reaction (arrow).

Classification of Monoclonal Immunoglobulin Deposition

Organized Crystalline Light chain Cast nephropathy

Light chain Fanconi syndrome

Fibrillar AA or AL amyloidosis

Fibrillary GN

Microtubular Immunotactoid GN

Cryoglobulinaemia

Disorganized Granular LCCD


HCDD

Light chain deposition disease (LCCD), Heavy chain deposition disease (HCDD)

Return to contents
A
Difference Between Amyloid and Fibrillar deposition

Amyloid Fibrillar

Fibrils 10-12 nm Fibrils 10-20 nm

Extra renal Glomerular involvement Rare extra glomerular renal


common-(Interstitum, vessels) involvement

Extra renal involvement common Extra renal involvement rare

Congo red positive Congo red negative

Return to question 1 Return to contents


A nswer

Electron microscopy of Fibrils

10 nm 20nm

Amyloidosis Fibrillary GN

Return to contents
Questions 152 & 153 Q
Question 152
Occurrence of severe hypocalcaemia following parathyroidectomy may be
predicted by pre-operative levels of ?

A Serum Calcium level


B Intact Parathyroid Hormone
C Alkaline phosphatase
D All of the above
E None of the above

Question 153
Urease producing bacteria are associated with the formation of?

A Calcium stones
B Uric acid stone
C Cystine stone
D Magnesium ammonium phosphate stone (Struvite)
E Oxalate stones

Return to question 1 Return to contents


A nswer
152. D - All of the answers are correct
153. D - Magnesium ammonium
phosphate stone (Struvite)

Answer 152
In a recent retrospective chart review, lower preoperative calcium levels and
higher preoperative intact parathyroid hormone, phosphorus, and alkaline
phosphatase levels were all found to be independent predictors of severe
hypocalcaemia following parathyroidectomy.

Tsai WC, et al. Int Urol Nephrol. 2015;47(7):1203-1207.

Answer 153
D - Magnesium ammonium phosphate stone (Struvite).
Options A, B, C and E are incorrect.
Option D is the correct answer.
Struvite (magnesium ammonium phosphate) stones are typically large
stones associated with urease producing bacteria and an alkaline urine. Urea
breakdown produces excess ammonium and hydroxyl ions, a rise in urinary pH,
and a decrease in phosphate solubility thus encouraging the precipitation of
insoluble magnesium ammonium phosphate.

Common bacteria producing urease


Proteus
Haemophilus
Pseudomonas
Klebsiella
Yersinia
Staphylococcus epidermidis
Citrobacter, Serratia and Ureaplasmaurealycium

Note: E.coli does not produce urease

Return to contents
A
Dietary and pharmacological management of kidney stones according to
urinary abnormalities.

Risk factors for calcium nephrolithiasis include

1 Low daily fluid intake


2 High dietary sodium and oxalate
3 Hyperoxaluria
4 Hypercalciuria, hypocitraturia, and hyperuricosuria

Return to question 1 Return to contents


A nswer

Common images of Stone crystals for Exam

Return to contents
Questions 154 & 155 Q
Question 154
The treatment of choice for transient diabetes insipidus of pregnancy is?

A AVP (Atrial vasopressin)


B Desmopressin
C Tolvaptan
D Demeclocycline
E none of the above

Question 155
Cystatin C is a low molecular weight protein filtered at the glomerulus and
metabolized by the tubules.

Which one of the following factors has been shown to affect cystatin C levels
independent of GFR(Glomerular Filtration rate)?

A Hyperthyroidism
B Hypothyroidism
C Steroid use
D Advanced age
E All of the above

Return to question 1 Return to contents


A nswer
154. B - Desmopressin
155. D - Advanced age

Answer 154
Options A, C, D and E are incorrect.
Option B is the correct answer.
Transient diabetes insipidus of pregnancy is caused by increased expression
of vasopressin from the placenta. This results in a greater breakdown of
vasopressin. Desmopressin (DDAVP), however, is not a substrate for this
enzyme and remains effective.

Tolvaptan is a vasopressin antagonist and will act as an aquaretic, thus


exacerbating diabetes insipidus.
It is a category C drug for use in pregnancy.

Demeclocycline inhibits adenylatecyclase activity, which is downstream


of AVP binding to the V2 receptor and has been described as correcting
Hyponatraemia in SIADH.
However, it is potentially nephrotoxic with variable efficacy.
It is a category D drug in pregnancy due to its effect on skeletal formation, and
should not be used.

Answer 155
Options A, B, C and E are incorrect.
Option D is the correct answer.
All of the mentioned factors have been associated with alterations in cystatin C
production. Manetti et al showed that cystatin C concentrations were increased
in overt hyperthyroidism and decreased in mild hypothyroidism. Rule et al
demonstrated that patients on chronic steroids had higher measured GFR by
19% as compared to patients off steroids with the same cystatin C level. Lastly,
cystatin C has been demonstrated to increase with advanced age independent
of affecting GFR.

Return to contents
Questions 156 & 157 Q
Question 156
A 84-year-old man is referred for evaluation of proteinuria and edema. Other
than a remote history of tobacco abuse (he quit 15 years ago), he is healthy
with no other comorbidities. He takes no regular medications.

Laboratory data show a serum creatinine level of 110umo/L, serum albumin


level of 27, and serum cholesterol level of 5.3.
Twenty-four-hour urine collection shows 2600 mg of urinary protein excretion
per day. C3 is in the normal range at 85 mg/dl (reference range, 55120 mg/
dl), and antinuclear antibodies, HIV, hepatitis B surface antigen, and hepatitis C
antibody are all negative.
Kidney biopsy shows findings consistent with membranous nephropathy.
Immunofluorescence is negative for antiphospholipase 2 receptor (PLA2-R)
antibodies.

Which one of the following is the most appropriate next step in this patient
management?

A Age and risk factor appropriate cancer screening


B Warfarin prophylaxis
C Plasmapheresis
D Alternating monthly cycles of cyclophosphamide and prednisone
E Check serum antiPLA2-R antibodies

Question 157
Which one of the following is not a cause of metabolic alkalosis in end stage
renal disease (ESRD) patients on dialysis?

A Malnutrition
B Hypercapnea
C Nasogastric suction.
D Massive blood transfusion
E Plasmapheresis with fresh frozen plasma

Return to question 1 Return to contents


A nswer
156 - A
157 - B

Answer 156
Option A is the correct answer.

Options B,C,D &E are incorrect. Age- and risk factorappropriate cancer
screening is indicated for patients with membranous nephropathy (MN). When
a patient is diagnosed with MN, the first determination is whether the lesion is
primary (idiopathic) or secondary to a systemic condition. Approximately 25%
of individuals with MN over age 65 will have a malignancy diagnosed within 1
year of diagnosis of MN.

Hence, a diagnosis of MN should prompt age- and risk factorappropriate


cancer screening. In a 70-year-old man with prior tobacco abuse, this screening
should include at least a non-contrast chest computed tomography scan,
colonoscopy, prostate examination, and serum prostate specific antigen (PSA).

Answer 157
B - Hypercapnea

Options A, C, D and E are incorrect.


Option B is the correct answer.

In patients with ESRD, the kidney does not play a role in acid-base
homeostasis. Chronic hypercapnea does not produce a secondary elevation
in serum bicarbonate level since this compensatory response requires
kidney function. In the absence of kidney function, metabolic alkalosis can
occur from either exogenous administration of alkali or loss of HCL from the
gastrointestinal tract (vomiting, nasogastric suction).

Malnourished dialysis patients with poor protein intake and low muscle mass
have low endogenous acid production resulting in metabolic alkalosis.

Packed red cells and fresh frozen plasma are anticoagulated with citrate, which
generates bicarbonate upon metabolism.

Return to contents
Question 158 Q
A 57-year-old female recently started on peritoneal dialysis with low volumes
Patient denies abdominal pain but peritoneal dialysis sister informed she has
poor drain out problem.
You requested a plain abdominal X-ray (see image)

Which one of the following is the most likely cause in her case?

A Constipation
B Kink in catheter
C Fibrin clot in catheter
D Hernia
E Perforation of viscus

Return to question 1 Return to contents


A nswer A - Constipation

Options B, C, D and E are incorrect.


Option A is the correct answer.

X-ray shows faecal loading in large bowel and tip of catheter flip up
Most common cause of poor drain out of PD fluid is constipation.
Heavily loaded loops of bowel prevent the movement of fluid through the
peritoneal cavity, resulting in pools of located fluid and hence poor drainage.
Constipation is more common in-patient when starting PD, at the time when
they are more at risk of catheter malfunction.
Regular exercise also helps to avoid constipation and encourages the catheter
to remain in the pelvis.

Common causes of constipation in PD

Reduce fibre in diet


Reduced exercise
Use of phosphate binders and iron supplements

Return to contents
Questions 159 & 160 Q
Question 159
A first haemodialysis session should only reduced blood urea by?

A 10%
B 20%
C 30%
D 40%
E 50%

Question 160
You are doing a round in haemodialysis unit and you found one of your patient
is having a KT/V of 1.2.

His average fistula blood flow rate is between 350- 400ml/min.

Which one of the following is the most appropriate next step in his
management to increase his KT/V?

A Increase dialysis time


B Use higher blood flow rate
C Use bigger dialyser
D Do nothing
E None of above

Return to question 1 Return to contents


A nswer
159. C - 30%
160. A - Increase dialysis time

Answer 159
Options A, B, D and E are incorrect.
Option C is the correct answer.

A first haemodialysis session should only reduce blood urea by 30%.


Major concern is the dialysis disequilibrium syndrome, which occurs from
overcorrection of uraemia.

First Haemodialysis prescription:

Duration of first session of dialysis- 2 hrs


Second session is typically 3 hr and the third session is 3.5-4 hrs.
Blood flow is usually 150 ml/min. larger patients may require a slightly
longer session 2.5 hrs with blood flow of 250ml/min.
Ultrafiltration - No more than 2 Litres should be removed during a first
dialysis session.
Heparin free
Avoid use of mannitol.
High-risk patients of DDS may be given sodium valproate.

DDS (Dialysis Disequilibrium syndrome)

Answer 160
Options A, B, C all can increase KT/V but best option is A. Increasing dialysis
time to 20% will increase the KT/V.

Return to contents
Questions 161 & 162 Q
Question 161
Which one of the following statement is incorrect about haemodialysis
patients (HD)?

A Nocturnal haemodialysis has shown to improves sleep apnea symptoms


B Treatment of sleep apnea has been shown to improve survival in HD patient
C There is high prevalence of sleep apnea in HD patients
D Sleep apnea is HD associated with adverse outcome
E None of above

Question 162
A 26-year-old male presented with 4gm proteinuria and moderate renal
insufficiency. His kidney biopsy showed mild mesangial matrix expansion and
mesangialhypercellularity.
Immunofluorescence stains were done for IgG, IgA, IgM, C3, C1q, kappa light
chain and lambda light chain. Only C3 was positive in mesangial areas and
capillary loops.
See the images of kidney biopsy.

What clinical marker is most likely to be abnormal?

A Factor H levels
B Anti-nuclear cytoplasmic antibody
C Anti-nuclear antibody
D SCL-70
E Hgb-A1c

Return to question 1 Return to contents


A nswer
161 - B
162 - A

Answer 161
B - Treatment of sleep apnea has been shown to improve survival in HD
patient.
Options A, C, D and E are incorrect.
Option B is the correct answer.

Prevalence of sleep apnea in symptomatic haemodialysis patients (Restless


legs, morning headaches, personality changes, and daytime sleepiness) is 73%.
Kimmel, AmJMed-1989

Estimated overall prevalence in HD is 21-47% as compared to 3% in general


population.

Pathophysiology of sleep apnea in dialysis.

Volume overload leads to pulmonary congestion during recumbency.


Destabilization of central ventilatory control and underlying comorbid
conditions (DM, CVD, Obesity), which are independent risk factors for sleep
apnea.

Nocturnal HD improves symptoms of sleep apnea but there is not specific


treatment.

Answer 162
A - Factor H levels
Options B, C, D and E are incorrect.
Option A is the correct answer.
The biopsy represents C3 glomerulopathy . The glomerulus shows mild
mesangial matrix expansion and mesangialhypercellularity.

The differential diagnosis of predominantly C3 staining in glomeruli is

1. Complement disorders such as factor H deficiency


2. Post infectious glomerulonephritis

Return to contents
Question 163 Q
A 60-year-old male statuspost cadaveric kidney transplant, 2 months ago
presented with an acute rise in serum creatinine of 140 umol/L(Baseline was 82
umol/L).He denies any history of genitourinary or systemic symptoms.
On examination there is no graft tenderness.
Systemic examination was unremarkable.
His immunosuppressive medications include (prednisolone, cyclosporine and
Mycophenolate mofetil).
Kidney ultrasound showed no obstruction. His cyclosporine trough level result
is pending Graft biopsy is done.
Look at the biopsy image.

Which one of the following is the most likely diagnosis in his clinical scenario?

A Acute Calcineurin inhibitor toxicity


B Antibody mediated rejection
C Cellular rejection
D Acute tubular injury.
E Fungal infection

Return to question 1 Return to contents


A nswer
A - Acute Calcineurin
inhibitor toxicity

Options B, C, D and E are incorrect.


Graft biopsy is showing a Tubular isometric vacuolization thatis indicative of
calcineurin inhibitor toxicity.
Isometric vacuolation Differential Diagnosis.

Acute CNI (calcineurin inhibitor) Toxicity


Use of Plasma expanders (mannitol,dextran)
IV IG(Immunoglobulin)
Radiolabeled contrast media

Both cyclosporine and tacrolimus have the same nephrotoxic effects with
indistinguishable histological lesions.

Differential diagnosis of Rising Creatinine in kidney transplant

1. Ureteric obstruction
2. Transplant artery disease
3. Drug toxicity- Tacrolimus/cyclosporine
4. IFTA- Interstitial fibrosis tubular atrophy
5. Acute rejection
6. Infections- Viruses, BK
7. Recurrent primary disease

Return to contents
Questions 164 & 165 Q
Question 164
See the electron microscopic kidney biopsy image of a 31-year-old male
patient with rheumatoid arthritis on long-term chloroquine treatment.
The patient does not have angiokeratomas on examination.

What is the most likely cause?

A Fabry disease
B Amyloidosis
C Chloroquine toxicity
D Lecithin cholesterol acyltransferase
deficiency
E Scleroderma

Question 165
The kidney biopsy image provided shows a nodular glomerulosclerosis.

Which one of the following is the most likely aetiology?

A Fabry disease
B IgG4 associated diseases
C Henoch Schonlein purpura
D Monoclonal light chain deposition disease
E Cocaine abuse

Return to question 1 Return to contents


A nswer
164. C - Chloroquine toxicity
165. D - Monoclonal light chain
deposition disease

Answer 164
Options A,B,D & E are incorrect.
Option C is the correct answer. The image shows myeloid bodies in tubules.
Both Fabry disease and chloroquine toxicity can cause this. However the
obvious association here is chloroquine toxicity.

Answer 165
Options A, B, C and E are incorrect.
Option D is the correct answer.
Causes of Nodular glomerulosclerosis

1. Monoclonal light chain deposition disease.


2. Chronic cigarette smoking.
3. Diabetic nephropathy
4. Amyloidosis.

Return to contents
Question 166 Q
Look at the electron microscopy image of a kidney biopsy

What is the most likely diagnosis?

A Diabetic nephropathy
B Post-infectious glomerulonephritis
C Membranous nephropathy
D Fabry's disease
E MPGN type III

Return to question 1 Return to contents


A nswer B - Post-infectious glomerulonephritis

Options A, C, D and E are incorrect. Option B is the correct answer.

The arrow points to a subepithelial hump which is characteristic of post


streptococcal glomerulonephritis.

The subepithelial deposits are frequently accompanied by less conspicuous


mesangial and subendothelial deposits and do not elicit the spike formation
seen in membranous nephropathy.

Kidney biopsy finding of Post infectious glomerulonephritis

Light microscopy Immunofluorescence Electron


Microscopy
Post-infectious Diffuse Capillary loop IgG Sub-epithelial
glomerulonephritis endocapillary and C3 humps
proliferation

Return to contents
Question 167 Q
An 18-year-old male presented with microscopic haematuria and a recent onset
of non-nephrotic range proteinuria.
Family history is negative for renal disease.
Physical examination reveals a BP of 145/80 mmHg, height of 5 ft, weight of
201 lb, and no Lower limb edema. Laboratory evaluation discloses a serum
creatinine of 140umol/l, 24-hour urine protein of 2 g, albumin of 32 g/dl, and
bland urine sediment. Serum complements are in the normal range. Serologic
workup is negative for anti-nuclear antibody, hepatitis B surface antigen,
hepatitis C antibody, and HIV. The kidneys measure 13.6 and 13.5 cm by
ultrasound. A renal biopsy is performed.

Electron microscopic image of a kidney biopsy is provided.

See the image


Which one is the most likely diagnosis in his scenario?

A Membranous nephropathy
B Membranoproliferative glomerulonephritis
C Minimal change nephropathy
D Alport disease
E IgA nephropathy

Return to question 1 Return to contents


A nswer D - Alport disease

Options A, B, C and E are incorrect.


Option D is the correct answer.
Diagnostic features on Kidney Biopsy
The electron microscopy image shows thickened basement membranes
with lamellar changes ( basket weave pattern) in the glomerular basement
membrane, indicative of Alport disease

Light Immunofluorescence Electron Microscopy


microscopy
Alport disease Normal, FSGS. Negative Thickened glomerular
basement membranes
with lamellar changes(
basket weave pattern).
Thin glomerular
basement membranes
in10-20%.

Presentation
Classically, persistent microscopic haematuria is the earliest manifestation
of disease and persists throughout childhood into adolescence. Occasional
episodes of gross haematuria may occur. With disease progression,
subnephrotic or nephrotic range proteinuria develops and is accompanied by
declining renal function. Most males with X-linked disease develop ESRD, often
by age 30.
Extrarenal manifestations of Alports
Sensorineural hearing loss and anterior lenticonus (of the eye), owing to the
presence of diseased collagen IV networks in the Organ of Corti and lens
capsule. Genetic testing for HN has recently become available and involves
se- quencing of the 3, 4, and5 chains of type IV collagen in an effort to identify
disease causing mutations.

Return to contents
Question 168 Q
A 53-year-old male with history of diabetes mellitus and alcoholic liver disease.
He presented with cellulitis due to MRSA (Meticillin-Resistant Staphylococcus
aureus). He was treated with antibiotics however he developed acute renal
failure.
Urine analysis showed RBC casts.
Image of a glomerulus is provided. Immunofluorescence staining showed
positivity in mesangium and capillary loops for IgA and C3.

Which one is the most likely diagnosis in his scenario?

A Henoch-Schonlein purpura.
B IgA nephropathy superimposed on diabetic nephropathy.
C Post infectious glomerulonephritis associated with Staph. Aureus infection.
D Hepatitis C associated glomerulonephritis.
E Rheumatoid arthritis

Return to question 1 Return to contents


A nswer
C - Post infectious glomerulonephritis
associated with Staph.
Aureus infection

Options A, B, D and E are incorrect.


Option C is the correct answer.
Staph. Aureus infection is associated with glomerulonephritis that
can mimic IgA nephropathy. The glomerular patterns can vary from
mesangialhypercellularity, through membranoproliferative glomerulonephritis
and diffuse endocapillary proliferation.
Ref: Staphylococcus infection-associated glomerulonephritis mimicking IgA
nephropathy. Clin J Am SocNephrol. 2006 Nov; 1(6):1179-86

Differential Diagnosis of Diffuse Proliferative Glomerulonephritis

1. Membranoproliferative Mesangial and subendothelial deposits with


glomerulonephritis GBM duplication Glomerular involvement may
be variable IgG or IgM with light chains (may be
monoclonal)

Mesangial and subendothelial deposits with


2. Lupus nephritis (Class
GBM duplication Glomerular involvement may be
IV-G)
(>50% of tuft in >50% of glomeruli) Usually a few
subepithelial depositsFull-house IF stainin

Mesangial and endocapillary proliferation


3. Acute postinfectious
Capillary wall IgG, C3 predominantly Mesangial
glomerulonephritis
and large ('humplike') subepithelial deposits No
GBM duplication

4. IgA nephropathy Mesangial capillary wall IgA staining May


(uncommon) see organized mesangial deposits by EM

5. Immunotactoid Microtubules (2055 nm) by EM Most are


glomerulonephritis monoclonal IgG

6. Fibrillary Fibrillary deposits (1329 nm) IgG, C3 common


glomerulonephritis by IF (smudgy to pseudolinear) Polyclonal light
chains

Return to contents
Questions 169 & 170 Q
Question 169
50-year-old female with breast cancer, admitted with nephrotic syndrome.
Kidney biopsy is undertaken. Look at the image.

What is the most likely aetiology?

A Elevated anti-neutrophil cytoplasmic antibodies.


B Bisphosphonate therapy
C Anti-glomerular basement membrane disease
D Lithium toxicity
E Positive anti-nuclear cytoplasmic antibody

Question 170
Withdrawal of dialysis will lead to death within?

A 1-2 days
B 3-4 days
C 5-6 days
D 7-14 days
E 15- 21 days

Return to question 1 Return to contents


A nswer
169. B - Bisphosphonate therapy
170. D - 7-14 days

Answer 169
Options A, C, D and E are incorrect.
Option B is the correct answer.

The image shows collapsing focal segmental glomerulosclerosis with shrunken


capillary loops surrounded by hypertrophied podocytes.
Associations of collapsing glomerulopathy include:

1. HIV associated nephropathy


2. Bisphosphonate therapy
3. Parvovirus B-19
4. African American heritage (APOL1 gene)

Answer 170
Options A, B, C and E are incorrect.
Option D is the correct answer.
Withdrawal of dialysis will lead to death within 7-14 days in most cases but
this is very dependent on residual renal function.

Discussion about withdrawal of dialysis should include


An assessment of patient ability to make a decision
Reversible factors should be addressed
Full discussion with dialysis team
Full involvement of the family

Return to contents
Question 171 Q
Which one of the following statements is correct as per K/DOQI guidelines
about vascular access?

A Permanent vascular access should be in order of preference in elbow then


wrist
B AVF (Arteriovenous fistula) more likely to be useable when they have flow is
>600 ml/min, fistula length 6cm and no more than 6 mm deep from skin.
C AVF (Arteriovenous fistula) more likely to be useable when they have a flow
400 ml/min, fistula length 5cm and no more 6 mm deep
D AVF (Arteriovenous fistula) patency should be 1 year
E AVG (Arteriovenous graft) patency >3 yrs

Return to question 1 Return to contents


A nswer B

AVF (arteriovenous fistula) more likely to be useable when they have flow is
>600 ml/min, fistula length 6cm and no more than 6 mm deep from skin.
Options A, C, D and E are incorrect.
Option B is the correct answer.

K/DOQI Guidelines

AVF more likely to be useable when they have flow>600 ml/min,


6cm long, and no more 6 mm deep
Permanent vascular access should be in order of preference is wrist then elbow
AVF patency should be >3 year
AVG patency >2 yrs

Indications for Fistula Doppler veins arms (R/L)

Routine assessment of maturation at 8 weeks post op


Cannulation problems
Fistula seems too deep to needle
Suspected significant haematoma over access
Suspected axillary vein thrombosis (sudden swelling of fistula arm)

Return to contents
A
Complications of AVF surgery.
(First 8 weeks).

Failure (approximately 10% by first 6 weeks post surgery)


Failure to mature (good thrill but not needle able)
Steal syndrome
Venous hypertension

Complications of AVF surgery


(Beyond 8 weeks)

Steal syndrome
Venous hypertension
Poor flow
Stenosis
Aneurysmal
Infection
High output cardiac failure

Return to question 1 Return to contents


A nswer

Guidance for Bleeding AVFistula Post Haemodialysis


Vascular Access (VA)

Return to contents
Question 172 Q
65-year-old male on maintenance haemodialysis three times per week
presented to accident and emergency with progressive swelling of left arm.
Currently he is dialysing with left Brachiocephalic AVF (arteriovenous fistula)
that was created 10 weeks ago. On examination his AVF looks humpier.
Previously he was dialysing with left internal jugular permanent catheter.

Look at the images

What is the most likely diagnosis?

A Central vein stenosis


B Cellulitis of hand
C Steal syndrome
D High output cardiac failure
E Aneurysm of AVF

Return to question 1 Return to contents


A nswer A - Central vein stenosis

Options B, C D and E are incorrect.


Option A is the correct answer.
Patient has central venous stenosis and has symptoms of venous
hypertension (swelling hand, dilated veins).
Next step is to do Fistulogram and central venogram.
The treatment is angioplasty of central stenosis with or without stenting
but if it become a recurrent event then needs ligation of AVF and creation of
new access.

K/DOQI Guideline- Indications for Fistulogram

Swelling of whole fistula arm

Prolonged bleeding >10 mins post dialysis on more than one occasion
despite optimisation of anticoagulation regime

Increase in size of aneurysms

Persistent problems with scabs >3mm diameter

Unable to achieve dialysis blood flow of at least 300 ml/min

25% fall from baseline in either achieved blood flow on dialysis or fistula
flow (transonic measurement within first 1.5 hrs of HD)

Recirculation >5% on 2 consecutive dialysis sessions

Dynamic venous pressure >150 mmHg when measured using 15G


needles and blood flow 200 ml/min in the first 2 - 5 mins of dialysis (rising
trend over time is more useful than a single measurement so compare
with baseline).

Unexplained fall in 2 consecutive URR measured on a 4 hour dialysis


session

Return to contents
Questions 173 & 174 Q
Question 173
Which allele is associated with increased genetic susceptibility to anti-GBM
antibody disease?

A HLA-DR1
B HLA-DR15
C HLA-DR7
D HLA-DP1
E HLA-DP 2

Question 174
Which of the following statements is incorrect?

A Presence of ANCA positivity in patients with anti-GBM glomerulonephritis


(double positive) alters treatment outcomes.
B Double positive patients more often present with c-ANCA than p-ANCA.
C Double positive patients are more likely to have systemic manifestations of
vasculitis on presentation.
D Patients who present with anti-GBM disease and 100% crescents on
kidney biopsy requiring dialysis have a lower likelihood of kidney recovery
than patients who present without requiring dialysis when treated
aggressively with plasmapharesis, steroids, and cyclophosphamide.
E None of above

Return to question 1 Return to contents


A nswer
173. B - HLA-DR15
174. B

Answer 173
Options A, C, D and E are incorrect.
Option B is the correct answer.
Genetic factors, along with pulmonary infections, have been associated the
development of anti-GBM disease. There is evidence of increased susceptibility
of developing anti- GBM disease in patients with HLA-DR15.
(HLA-DRB1*1501 allele), which is especially true in the white, Chinese, and
Japanese populations. Similarly, studies have shown a reduced risk of anti-GBM
disease in patients with HLA-DR1 and DR7.

Answer 174
B. Double positive patients more often present with c-ANCA than p-ANCA.
Options A,C,D and E are incorrect Option B is the correct answer.
Patients who are double positive for ANCA and anti-GBM have a better overall
survival. These double positive patients also have more systemic manifestations
of vasculitis on presentation, and are more often p-ANCA or anti-MPO
positive. Prognosis for kidney recovery in patients presenting with anti-GBM
glomerulonephritis and 100% crescents on kidney biopsy when requiring
dialysis is very poor.

Return to contents
Questions 175 & 176 Q
Question 175
What is the preferred GFR estimation equation for use in morbid
obesity individuals?

A MDRD equation
B CKD EPI (creatinine)
C Cockcroft Gault
D None of the above
E All of above

Question 176
What is the KDOQI recommended pre-dialysis serum bicarbonate
level in adults?

A 18 mEq/L
B 20 mEq/L
C 22 mEq/L
D >22mEq/L
E None of above

Return to question 1 Return to contents


A nswer
175 - D
176 - C

Answer 175
D. None of the above
Options A,B,C and E are incorrect
Each of the estimating equations for GFR has shown inconsistencies in
accurately determining GFR (either under- or over-estimating GFR). For
instance, Friedman et al demonstrated that neither creatinine- nor cystatin C
based equations (Cockcroft Gault or MDRD equations) correlated with GFR as
measured by iohexol in 44 obese individuals with normalkidney function,
with both equations overestimating GFR. Similarly, Verhave et al demonstrated
that both the MDRD and Cockcroft Gault equations were not accurate in
estimating GFR in subjects with BMI >30 kg/m2. Lastly, Bouquegneau et
al showed that the CKD-EPI (creatinine based) did not outperform MDRD
equation in 366 patients with a mean BMI >36 kg/m2)

Answer 176
C - 22 mEq/L.
Options A, B, D and E are incorrect.
Option C is the correct answer.

The KDOQI guidelines suggest that pre-dialysis HCO3 should be around


22 mEq/L.

Return to contents
Question 177 Q
You receive an ABO incompatible renal transplant patient with an acute rise in
serum creatinine. Kidney biopsy was done.

Immunofluorescence image of complement (C4d stain) is given below.

Which is the correct statement?

A Glomerular capillary loop C4d staining is diagnostic of acute antibody


mediated rejection.
B The image shows diffuse tubular basement membrane staining for C4d
C C4d stain is diagnostic of cellular rejection.
D C4d stain does not correlate with acute rejection in ABO incompatible
renal transplants.
E None of above

Return to question 1 Return to contents


A nswer D

C4d stain does not correlate with acute rejection in ABO incompatible renal
transplants.

Options A, B, C and E are incorrect, option D is the correct answer.

The image shows diffuse Peritubular capillary staining for C4d staining by
immunofluorescence. Peritubular capillary C4d does not correlate with AMR in
ABO incompatible patients.
Specificity of peritubular capillary staining for C4d is in association with HLA
antibody mediated rejection and not ABO mediated rejection. In ABO-
incompatible grafts, 80% of protocol biopsies and 59% performed for graft
dysfunction show C4d staining in peritubular capillaries. Note that glomerular
capillary loop C4d staining (as opposed to peritubular capillary staining) has
no significance.
(Ref: Am J Transplant. 2006 Aug;6(8):1829-40)

Return to contents
Question 178 Q
25-year-old female presented with acute renal failure and the following biopsy
image. Serology, and other tests are not available.

Image of H+E stain is provided

Which of the following test supports kidney biopsy diagnosis?

A Anti-glomerular basement membrane antibody


B Anti-phospholipid antibody
C Blood eosinophil count.
D Anti-neutrophil cytoplasmic antibody
E Serum protein electrophoresis

Return to question 1 Return to contents


A nswer B - Antiphospholipid antibody

Options A, C, D and E are incorrect.


Option B is the correct answer.

The image shows severe small arterial and arteriolar microangiopathy.


Causes of Thrombotic microangiopathy include:

1. Hemolytic uremic syndrome


2. Thrombotic thrombocytopenic purpura
3. Scleroderma
4. Antiphospholipid antibody syndrome
5. Factor H deficiency
6. Malignant hypertension
7. Drugs such as calcineurin inhibitors
8. Antibody mediated rejection

Return to contents
Question 179 Q
A 41-year-old Asian man presents with a 10 days history of lowgrade fever,
myalgias, cough, and decreased oral intake, more recently accompanied by
oliguria and a single episode of gross haematuria.
The patient was seen 5 days earlier and given a course of ciprofloxacin, without
symptomatic improvement. Laboratory evaluation reveals acute kidney injury
with a creatinine level of 900 umol/l, K 7.9 (with ECG changes) , albumin 25 g/
dl, hematocrit 30%, white blood cell count 11,500, and platelet count 352,000.
Past medical history is significant only for chronic lower back pain.
There is no history of hypertension, diabetes, obesity, or tobacco use.
The patient denies use of NSAID or overcounter medications.
Physical examination reveals a BP of 140/89 mmHg and no edema or skin rash.
Urine dipstick reveals 2+protein and 3 +blood.
Serologic results include a negative antinuclear antibody and normal C3 and
C4 complement levels.
After 2 sessions of haemodialysis, Kidney biopsy is undertaken.
Additional serologies are pending at the time of renal biopsy .
On day 3 patient condition is detoriated and he was transfer to the critical care
bed.

Histopathologist call you with results

What is the most likely Kidney biopsy diagnosis?

A Memberanoproliferative glomerulonephritis
B Cresenteric glomerulonephritis
C IgA Nephropathy
D Membranous nephropathy
E Cytomegalovirus (CMV) infection

Return to question 1 Return to contents


A nswer B - Cresenteric glomerulonephritis

The biopsy shows classic crescetntic glomerulonephritis, the glomerulus is full


of cellular crescents, chest X-ray shows diffuse alveolar haemorrhage. This is a
severe form of rapidly progressive glomerulonephritis.
This patient is an example of pulmonary renal syndrome.
Options A, C, D and E are incorrect.

Immunologic causes of pulmonary-renal syndrome

Primary systemic vasculitis


Granulomatosis with polyangiitis (Wegeners)
Microscopic polyangiitis
Eosinophilic granulomatosis with polyangiitis (Churg Strauss)
Anti-GBM disease (Goodpastures)
Lupus vasculitis
IgA vasculitis (HenochSchonlein)
Cryoglobulinaemia
Behcets disease
Rheumatoid vasculitis
Drugs e.g. propylthiouracil, carbimazole, penicillamine, hydralazine,
cocaine

Commonest causes Pulmonary Renal syndrome

1. Vasculitis-80%
2. Goodpastures syndrome-10%
3. SLE-5%
4. Others-5%

Diagnosis of Alveolar haemorrhage

Haemoptysis/dyspnoea
Progressive anaemia
Hypoxia
Chest x-ray -diffuse airspace shadowing
Raised CO transfer coefficient
BAL (Broncho alveolar lavage) - frank blood/haemosiderin laden macrophages
Lung biopsy - pulmonary capillaritis

Return to contents
A
Current approach to treatment of severe ANCA associated Vasculitis
INDUCTION
(Therapy to achieve remission)
Prednisolone (+/-) iv Methyl Pred
Cyclophosphamide iv/oral
Or consider Rituximab
Plasma exchange (60ml/kg-7 sessions)
MAINTENANCE
(Therapy to maintain remission (switch 3-6 months)
Low dose prednisolone
Convert cyclophosphamide to azathioprine 1.5 mg/kg/day (max 200 mg) or
MMF 1 g bd is an alternative in patients intolerant of azathioprine, or
consider Rituximab.
Maintenance therapy is not recommend in patients who are dialysis dependent and
have no extrarenal manifestations. Relapse rate are lower in patients with ESRD.

Initial therapy of crescentic glomerulonephritis based on pathogenic category


and dialysis dependency. Dialysis dependency defined as requiring dialysis
treatment of renal failure for >1 week

Category Dialysis independent Dialysis dependent


Anti GBM Oral or iv glucocorticoids Conservative
Oral cyclophosphamide
Plasmapheresis
Immune complex Oral or iv glucocorticoids Oral or iv glucocorticoids
Oral or iv cyclophosphamide Oral or iv cyclophosphamide

ANCA associated Oral or iv glucocorticoids Oral or iv glucocorticoids


Oral or iv cyclophosphamide Oral cyclophosphamide
Plasmapheresis
Dual antibody Same as for anti GBM Same as for ANCA
Idiopathic Same as for ANCA Same as for ANCA

Vasculitis and kidney


1. Clinical features of renal involvement (e.g. haematuria, proteinuria, active
urinary sediment, renal failure)
2. Serological assessment (ANCA testing)
3. Histopathological evidence (positive renal biopsy)

Return to question 1 Return to contents


A nswer

Presentation
Rapidly progressive glomerulonephritis (RPGN)
Rising creatinine + crescents presents on biopsy (oliguria) Urine
Haematuria+RBCCast+Proteinuria
Kidneys normal size

Following are indications for Plasma exchange in RPGN


Plasma exchange- Current recommendations
Indications
Creatinine > 500mol/L (5.8mg/dl)
Diffuse alveolarhaemorrhage, hypoxia ,or coexisting anti GBM antibodies
7 exchanges within 14 days 60ml/kg
Volume replacement -with albumin(4.5%), coagulation factors(FFP) if risk of
haemorrhage or if there has been a recent renal biopsy or Octaplus
BSR guidelinesfor ANCA vasculitis: Ntatsaki et al, Rheumatology 2014

Renal biopsy confirms diagnosis and has prognostic value.


Kidney Biopsy Findings of Crescentic Glomerulonephritis

Light Immunofluorescence Electron


microscopy Microscopy
Crescentic Extracapillary Depends on type. Electron dense
glomerulonephritis proliferation of 1) Linear IgG stain for deposits in
cells (crescents) anti-GBM disease. immune complex
2) Immune complexes glomerulonephritis.
for Lupus, IgA Negative or few in
nephropathy etc. other types.
3) Negative or
few deposits n
ANCA associated
pauci-immune
glomerulonephritis.

Return to contents
Question 180 Q
A 60-year-old male known case of alcoholic liver cirrhosis was admitted
with left ankle fracture. He was known to have CKD stage 3. He was taking
spironolactone, which was stopped on admission, and he had moderate
volume ascites.

Labs on admission

Sodium 123 mmol/L (137-144)


Potassium 5.9 mmol/L (3.5-4.9)
Urea 9.5 mmol/L (2.5-7.0)
Creatinine 170 umol/L(60-110)

Urine dipstick-trace protein


US Kidney- bilateral normal size kidneys with no hydronephrosis
Septic screen -ve

After 48 hrs of fluid resuscitation, including human albumin solution his blood
results were as follows

Sodium 128 mmol/L(137-144)


Potassium 5.6 mmol/L(3.5-4.9)
Urea 7.1 mmol/L (2.5-7.0)
Creatinine 140 umol/L(60-110)

What is the next best step for this patient management?

A Renal biopsy
B Start haemodialysis
C Drainage of ascites
D Terlipressin
E Administer 3 L of IV colloid

Return to question 1 Return to contents


A nswer D - Terlipressin

Options A, B, C and E are incorrect.


Option D is the correct answer.

One should ensure that the patient is adequately filled and there is an absence
of hypovolaemia prior to diagnosing Hepatorenal syndrome and starting
terlipressin. This patient has been fluid resuscitated, yet this has not reduced
his creatinine level to <133 umol/L.

The European association for the study of liver (EASL) has defined hepatorenal
syndrome as The occurrence of renal failure in a patient with advanced
liver disease in the absence of an identifiable cause of renal failure. Thus the
diagnosis is essentially one of exclusion of other causes of renal failure.

Return to contents
A
International Ascites club (IAC) guidelines:
Criteria for the diagnosis of Hepatorenalsyndrome (2005)

Cirrhosis with ascites


Absence of hypovolaemia- no improvement in serum creatinine levels
(decreased to <133umol/L) after atleast 2 days with diuretic withdrawal (if on
diuretics) and volume expansion with albumin.T he recommended dose of
albumin is 1g/kg/day upto 100 g/day.
Absence of shock- renal failure in the setting of ongoing bacterial infection,
but in the absence of septic shock, can now be regarded as HRS.
No current or recent treatment with nephrotoxic drugs
Absence of parenchymal kidney disease as indicated by proteinuria >500
mg/day, microhaematuria (>50 RBC per high power field) and or abnormal
renal US.

Difference between type 1 and type 2 Hepatorenal syndrome

Type 1 Type 2

Rapidly progressive Moderate renal failure with slow progression


Commonly precipitated
by bacterial infection SBP Occurs in patient with refractory ascites

Not a terminal event of Survival better than type 1 HRS


hepatic failure, but
potentially reversible

Return to question 1 349 Return to contents


A nswer

Terlipressin

Initial dose of 0.5-1 mg every 4-6 hrs


If there is no early response (defined as <25% decrease in serum creatinine
after 3 days of treatment), the dose can be increased in a stepwise manner
up to maximum of 2mg/4 hourly(12 mg in 24 hrs)
If after 14 days of treatment, the creatinine level does not fall below
133umol/L stop terlipressin.
use it with caution in patients with IHD, HTN, cardiac arrhythmia to
atherosclerosis
It is contraindicated in pregnancy

A combination of albumin and terlipressin has been shown to reverse


hepatorenal syndrome in about 60% of cases of type 1 HRS.I n type 2 HRS has
not been widely studied.

Liver transplantation remains best treatment for type 1 and type 2 HRS,
although many patients still require dialysis afterwards.

RRT with haemofiltration and haemodialysis may be useful in patients who fail
to respond to vasoconstrictor therapy and who fulfil criteria for renal support.

Salerno F,Gerbes A et al- diagnosis, prevention and treatment of hepatorenal syndrome in cirrhosis
GUT 2007

Return to contents
Question 181 Q
50-year-old female admitted with a positive Rheumatoid factor and with acute
nephritic syndrome.

Look at the image of kidney biopsy

What is the most likely cause?

A Monoclonal gammopathy
B Cryoglobulinemia
C HIV
D Parvovirus B-19
E Factor H deficiency

Return to question 1 Return to contents


A nswer B - Cryoglobulinemia

Options A, C, D and E are incorrect.


Option B is the correct answer.
The image shows membranoproliferative glomerulonephritis with intracapillary
loop cryoglobulin PAS positive deposits (arrows)

Eculizumab in C3 Glomerulonephritis and Dense Deposit Disease

glomerular endocapillary proliferation and neutrophil


infiltration

No significant diminution in staining for C3 or C5b-9

No resolution of glomerular dense deposits after 1 yr of


therapy

Eculizumab appears to bind to and deposit within glomeruli,


basement membrane and vessel walls of treated patients

Return to contents
Question 182 Q
A 25-year-old female received a cadaveric kidney transplant 2 years ago.
She admitted with a worsening graft function. Her immunosuppession
medications include (Prednisolone, Cyclosporine and Mycophenolate mofetil),
Cyclosporine trough level and all other serology and immunology results are
pending.
Ultrasound transplant kidney shows no hydronephrosis.
Graft biopsy was done

Look at the image

Which is the most likely diagnosis in her clinical scenario?

A Haemolytic-Uraemic syndrome
B Cyclosporine toxicity
C Polyomavirus infection
D Acute tubular injury
E Acute vasculitis

Return to question 1 Return to contents


A nswer B - Calcineurin inhibitor toxicity

Options A,C ,D and E are incorrect.


Option B is the correct answer.

The image shows a glomerular hilar arteriole with peripheral hyaline nodules
(arrows)which is indicative of chronic calcineurin inhibitor toxicity.

Teaching notes- Kidney biopsy features of calcineurin inhibitor toxicity

Light microscopy Immunofluorescence Electron


Microscopy

Calcineurin inhibitor Acute: thrombotic Negative Not applicable


toxicity microangiopathy,
glomerular
and arteriolar
thrombi, isometric
vacuolization of
tubular epithelial
cells.
Chronic: arteriolar
peripheral hyaline
nodule formation.

Return to contents
Question 183 Q
66-year-old male admitted with acute on chronic renal failure, Had a recent
AAA (abdominal aortic aneurysm) repair done 2 weeks ago.
Kidney biopsy done.

Look at the image.

What is the most likely diagnosis?

A Thrombotic microangiopathy
B Endothelial inflammation
C Cholesterol crystal embolization
D Acute tubular injury.
E Acute interstitial nephritis

Return to question 1 Return to contents


A nswer C - Cholesterol crystal embolization

Options A, B, D and E are incorrect. Option C is the correct answer.


The image shows an artery with needle shaped cholesterol crystal
emboli (arrow).

Cholesterol embolization suggested with


Presence of embolic lesions
AKI delayed to weeks after procedure (3-8 weeks)
Livedoreticularis
Eosinophilia
AKI not resolving
Hypocomplementemia (Low C3C4)

Risk Factors for Atheremboli renal disease

Atherosclerosis
Systemic hypertension
Cigarette smoking
Hypercholesterolemia
DM
Age>55 yrs
White race Trauma to atheromatous Plaque
Invasive vascular procedure involving aorta proximal to origin of
renal arteries
Thrombolytic therapy
Blunt trauma Prevention of healing of the eroded plaque
Anticoagulant therapy
Thrombolytics

Return to contents
A
Differential Diagnosis of Cholesterol emboli renal disease

1. Contrast nephropathy, the renal failure occurs within 48 to 72 h after


the dye infusion and generally resolves within 4 to 7 d.

2. Ischemic acute renal failure is diagnosed by its immediate onset, its


association with hypotension, and the lack of systemic manifestations
such as skin rash, eosinophilia, and hypocomplementemia

Livedoreticularis Blue toes

Return to question 1 357 Return to contents


A nswer

Hollenhorst plaque in retinal artery (Cholesterol crystal arrow)

Return to contents
Questions 184 & 185 Q
Question 184
A 79-year-old Caucasian man presents with florid nephrotic syndrome and
acute renal failure. The patient has a 24-hour urine protein of 20.0 g, serum
albumin level of 13g/dl, 3+ peripheral edema, bland urine sediment, and an
increase in creatinine from 80umol/l to 420umol/l over 2 weeks.
Kidney biopsy is undertaken
See the image

What is the most likely diagnosis?

A Membranoproliferative glomerulonephritis.
B Minimal change nephropathy.
C Thin glomerular basement membrane disease.
D Membranous nephropathy.
E Diabetic nephropathy.

Question 185
According to KDIGO guideline which of the following statement is incorrect
about use of other therapies in the management of Acute Kidney Injury?

A Guideline suggest achieving a total energy intake of 20-30 Kcal/Kg/day in


patients with any stage of AKI.
B Guidelines recommend using low dose dopamine to treat or prevent AKI
C Guideline suggest not using fenoldopam to prevent or treat AKI
D Guideline suggest not using atrial natriuretic peptide (ANP) to prevent or
treat AKI
E In the absence of hemorrhagic shock guidelines suggest using IV Crystalloid
fluids rather than colloids as initial management for expansion of
intravascular volume in patients at risk for AKI or with AKI.

Return to question 1 Return to contents


A nswer
184 - D
185 - B

Answer 184
Membranous Nephropathy. Note glomerular basement membrane spikes
by silver stain.
Options A, B, C and E are incorrect.
Option D is the correct answer.

Diagnostic features on Kidney biopsy.

Light microscopy Immunofluorescence Electron


Microscopy

Membranous Thickened Granular capillary Subepithelial


Nephropathy capillary loops loop staining for electron dense
and spike and IgG. deposits
holes by silver
stain

Answer 185
B - Guidelines recommend using low dose dopamine to treat or prevent AKI.
Options A, C, D and E are incorrect.
Option B is the correct answer.

Return to contents
Question 186 Q
Which of the following drug is better removed by haemodialysis?

A Theophylline
B Phenytoin
C Digoxin
D Lithium
E None of above

Return to question 1 Return to contents


A nswer D - Lithium

Options A, B, C and E are incorrect.


Option D is the correct answer.

You will definitely get 1 or 2 questions in the exam about this topic.

Basic principal is drugs or toxins which can be removed by dialysis are of


Low MW <500 Da
Low degree of protein binding
Water-soluble
Small volume of distribution<1Litre/Kg
Increased clearance by dialysis over native clearance

Drugs and Toxins Removed by Haemodialysis

Methanol and Ethylene glycol


Lithium
Aspirin (acetylsalicylic acid)
Barbiturates
Star fruit poisoning (Averrhoacarambola)

Drugs and Toxins removed by Haemoperfusion

Theophylline- better removed by perfusion than by dialysis, consider if


levels >30 ug/ml or if liver disease or cardiac toxicity

Phenytoin (highly protein bound well removed by perfusion- only needed


if severe cerebral toxicity (levels usually >30ug/ml)

Digoxin- severe poisoning accompanied by hypokalaemia, hyperkalaemia


or alkalosis initial treatment with oral charcoal and correction of electrolyte
disturbances. Poorly removed by dialysis. Digoxin Fab antibody fragments
also effective therapy for severe intoxication.

Paraquat- survival dependent on initial plasma level. Perfusion or dialysis


2nd line therapy after repeated oral adsorbents.

Tricyclic antidepressants and phenothiazines- very poorly removed by


extracorporeal techniques (highly protein bound)

Return to contents
Question 187 Q
Which of the following advice is unlikely to be beneficial When counselling a
female transplant patient with regards to conception and pregnancy?

A It would be best to wait at least 1 year after live donor transplantation to


avoid complications arising from immunotherapy and rejection
B It would be best to wait at least 2 years after deceased transplantation to
avoid complications arising from immunotherapy and rejection
C The renal allograft should be functioning well, with a stable serum creatinine
level < 133 micromol/L and urinary protein excretion <500 mg/day
D Kidney transplant recipients who wish to conceive should change
from MMF (mycophenolate mofetil) to Azathioprine, if there are no
contraindications to the switch.
E MMF (mycophenolate mofetil) should be discontinued at least two weeks
prior to attempted conception

Return to question 1 Return to contents


A nswer E

Mycophenolate mofetil (MMF) should be discontinued at least 6 weeks prior to


attempted conception.
Options A, B, C and D are incorrect.
Option E is the correct answer.
MMF causes first trimester pregnancy loss and congenital malformations,
including cleft lip and palate, and anomalies of the distal limbs, heart,
esophagus, and kidneys.
Sifontis NM, Coscia LA, Constantinescu S, et al. Pregnancy outcomes in solid organ transplant
recipients with exposure to mycophenolate mofetil or sirolimus. Transplantation 2006; 82:1698.
Immunosuppressants Contraindicated in Pregnancy

Mycophenolate mofetil (MMF)


Mycophenolic acid
Sirolimus
Methotrexate
Cyclophosphamide
ATG
OKT3
Gold
Chlorambucil
D-Pencillamine
Leflunamide
Rituximab
Abatacept

Immunosuppressants safe in Pregnancy

Prednisolone
Cyclosporine
Tacrolimus
Azathioprine
Hydroxychloroquine
Sulfasalazine
Etarnercept
(Infliximab)
IVIG

Update:Rheumatology(oxford):2008,Jun 47, suppl 3

Return to contents
Questions 188 & 189 Q
Question 188
Which of the following statements is not true with regards to pregnancy and
SLE?

A Neonatal lupus is a passively transferred autoimmune disease that occurs


in some babies born to mothers with anti-Ro/SSA and/or anti-La/SSB
antibodies
B Maternal SLE is also associated with an increased risk of premature delivery
and intrauterine growth restriction
C Renal disease should be in remission for at least six months prior to
conception
D Risk of foetal loss is increased in women with hypocomplementemia,
increased anti-DNA antibodies, antiphospholipid antibodies, and
thrombocytopenia
E Treatment of thrombocytopaenia includes high-dose prednisolone and
intravenous immune globulin

Question 189
A 27-year-old female known case of antiGBM disease, who is now dialysis
dependent asks you about the possibility of future kidney transplant.
Her recent anti GBM level is (10.6Eu/ml; the normal level is <5 Eu/ml).

what advice you will give about possibility of a future kidney transplant in this
patient?

A Wait for 3 months


B Wait for 8 months
C Wait for 12 months
D Absolute contraindicated in this patient
E Immediate proceed with transplant

Return to question 1 Return to contents


A nswer
188 - C
189 - C

Answer 188
Renal disease should be in remission for at least six months prior to conception.
Options A, B, D and E are incorrect.
Option C is the correct answer.

Wait for at least 12 months.


Options A, B, D and E are incorrect, option C is the correct answer.

Renal disease and Pregnancy- Renal Transplant.

General guidelines

1. Wait 2 years post-transplant (some say 12 months)


2. Stable renal function
3. Minimal proteinuria
4. Minimal or well-controlled hypertension
5. No recent transplant rejection
6. Minimal levels of appropriate immunosuppression

Answer 189
Kidney transplantation can be safely carried out in Goodpasture's disease
after anti-GBM antibodies are no longer detectable. The best advice is to
leave an interval of at least 6 -12 months after the first negative result (this will
depend on the sensitivity of the test being used). This may be a problem if no
treatment was given at the outset, as the time for safe transplantation may then
be 1-2 years away. But these precautions have largely removed the problem of
destruction of transplants caused by return of Goodpasture's disease.

Return to contents
Questions 190 &191 Q
Question 190
Which of the following statements is not true with regards to pregnancy in renal
transplant patients?

A Sirolimus is contraindicated in pregnancy as animal studies have


demonstrated embryo toxicity and fetotoxicity with increased mortality,
reduced foetal weights, and delayed ossification.
B During pregnancy, the hepatic cytochrome P450 enzymes may be inhibited,
which can lead to decreased serum levels of tacrolimus.
C Severe preeclampsia and thrombotic thrombocytopenic purpura-haemolytic
uremic syndrome (TTP-HUS) (due to pregnancy) may be difficult to
distinguish in the pregnant transplant patient, particularly since they both
present with haemolysis and thrombocytopenia
D Patients taking cyclosporine or tacrolimus require more frequent monitoring
of renal function and drug levels during pregnancy.
E Sirolimus may cause impaired spermatogenesis and reduce male fertility

Question 191
Which of the following surgical non-infectious complication of kidney
transplantation is more associated with use of mTOR inhibitors?

A Seromas
B Hematomas
C Lymphocele
D Graft rupture
E none of above

Return to question 1 Return to contents


A nswer
190 - B
191 - C

Answer 190
During pregnancy, the hepatic cytochrome P450 enzymes may be inhibited,
which can lead to decreased serum levels of tacrolimus Options A,C,D and
E are incorrect Option B is the correct answer During pregnancy, the hepatic
cytochrome P450 enzymes may be inhibited, which can lead to increased
serum levels of tacrolimus. The dose may therefore have to be significantly
reduced to prevent toxicity (sometimes by as much as 60 percent).
Answer 191
Lymphocele, option C is the correct answer. Options A, B, D and E are incorrect.
Lymphocele seen in 10% cases post kidney transplantation.
About 5-15% cases are associated with the use of mTOR inhibitors.

Management
Conservative if non obstructing
If obstructing- percutaneous drainage
30% require surgical repair.

Surgical complications-Kidney Transplant


Wound infections
Fluid collection- Vascular (seromas, hematomas)
Lymphocele- -10%
Urinary leaks
Decrease diuresis- urological
compression of ureter
urinary leak
obstruction of urinary tract at any level
Decrease diuresis-
Arterial or venous thrombosis, seen in 1-2% cases
Diagnosis- no flow by doppler ultrasound
Management- immediate surgical exploration

Lymphocele causing hydro-nephrosis.


Anechoic fluid collection shown
inferior to the renal transplant proved
to be a lymphocele upon drainage
(single arrow). The collection results
in hydronephrosis of the transplanted
kidney (double arrow) secondary to
mass effect.

Return to contents
Questions 192 & 193 Q
Question 192
Which of the following is most common cause of mortality after kidney
transplantation?

A Squamous cell carcinoma


B Posttransplant lymphoproliferative disease
C CMV infection
D Myocardial infection
E Diabetes Mellitus

Question 193
Which of the following is most strongly associated with development of
Post Transplant Diabetes (PTDM)?

A Tacrolimus use
B Hepatitis C
C advanced age of recipient
D Obesity
E Advanced donor age

Return to question 1 Return to contents


A nswer
192 - D
193 - C

Answer 192
CVD cardiovascular disease remains most common cause of mortality after
kidney transplant.
Options A, B, C and E are incorrect.
Option D is the correct answer.
Trends in reported causes of death with function during 1 post transplant

Answer 193
C - advanced age of recipient.
Options A, B, D and E are incorrect.
Option C is the correct answer.

PTDM: Independent Risk Factors


Age > 60
Age 49-59
BMI > 30
African American
Tacrolimus
Hepatitis C Antibodies
Corticosteroids
Kasiske et al. AJT 2003 3: 178

Tacrolimus is more diabetogenic than cyclosporine (Tacrolimus has a


dose dependent effect on Beta cells)
Steroid sparing regimens reduces incidence of NODAT
No long term data on any benefits in switching CNI to prevent NODAT
Early post transplant- Insulin, Vildagliptin, consider repaglinide
Long term management - weight loss, diet, moderate exercise, ACEi,
Statins

Return to contents
Questions 194 & 195 Q
Question 194
Which of the following factors is most strongly associated with post transplant
anaemia?

A Use of mTOR inhibitors


B Impaired allograft function
C Use of ACEi or ARBs
D Number of acute rejections
E Use of steroids

Question 195
The immunofluorescence microscopy finding in the image provided is
diagnostic of ?

A Post infectious glomerulonephritis


B Alport syndrome
C Membranous nephropathy
D Fibrillary glomerulonephritis
E Anti-glomerular basement membrane antibody syndrome

Return to question 1 Return to contents


A nswer
194 - B
195 - E

Answer 194
B - Impaired allograft function.
Options A, C, D and E are incorrect.
Option B is the correct answer.

Independent Risk Factors for Anaemia


1. Impaired renal function
2. Use of ACEIs or ARBs (after excluding ~ 5-10% of
patients with post-transplantation erythrocytosis)
3. Use of mycophenolate mofetil or azathioprine (no patients on
sirolimus included in analysis)
Kidney Transplant Recipients (Vanrenterghem Y, et al. Am J Transplant 2003; 7: 835)

Answer 195
E - Anti-GBM disease.
The image shows linear capillary loop staining for IgG.

Options A, B, C and D are incorrect.


Option E is the correct answer.

This immunofluorescence pattern shows positivity with antibody to IgG and


has a smooth, diffuse, linear pattern that is characteristic for deposition of
glomerular basement membrane antibody with Goodpasture syndrome.

Return to contents
Question 196 Q
A 46-year-old Caucasian man is discovered to have microscopic haematuria,
subnephrotic proteinuria, and renal insufficiency. Past medical history is
significant for hypertension for 2 years. There is no history of diabetes and no
family history of renal disease. Physical examination reveals no edema and BP
of 130/92 mmHg on antihypertensive medications. The patient denies any
history of recent infection, skin rash, arthralgias, or abdominal pain. Laboratory
evaluation includes serum creatinine level of 186umol/l, 24-hour urine protein
of 1.8 g, and serum albumin level of 36 g/dl. urine dipstick shows 2+protein
and 1+blood , and microscopic examination of the
urinary sediment reveals 10 red blood cells (RBCs) per high-power field
and no casts. Serum IgA level is mildly elevated, but all other serologies
are negative or normal, including serum complement levels (C3 and C4),
anti- nuclear antibody, hepatitis B surface antigen, hepatitis C antibody,
anti-neutrophil cytoplasmic antibody (ANCA), and antiglomerular basement
membrane (anti-GBM) antibody. No monoclonal spike is detected by serum
protein electrophoresis. The kidneys measure 11.3 and 12.2 cm in length by
ultrasound.
A kidney biopsy was performed
See the electron microscopy image

What is the most likely diagnosis in this patient scenario?

A Membranous nephropathy
B Memberoproliferative glomerulonephritis type 1
C IgA nephropathy
D Post streptococcal GN
E Alport syndrome

Return to question 1 Return to contents


A nswer C - IgA nephropathy

Options A, B, D and E are incorrect. Option C is the correct answer.


The image shows mesangial electron dense deposits in the solid mesangial
areas of the glomerulus (arrows). The most likely diagnosis is IgA nephropathy.
Other conditions such as lupus nephritis can also cause this.

Teaching notes- Kidney biopsy features of IgA Nephropathy


Light microscopy Immunofluorescence Electron
Microscopy

IgA nephropathy Normal glomeruli, Mesangial IgA Mesangial


mesangialhypercellularity, deposits. electron dense
proliferative glomeruli, deposits.
crescentic glomeruli


The Oxford Classification of IgA Nephropathy

Variable Score

Mesangialhypercellularity M0: >3 cells/mesangial area in 50% of glomeruli


M1: >3 cells/mesangial area in >50% of glomeruli
Segmental glomerulosclerosis S0: absent
S1: present
Endocapillaryhypercellularity E0: absent
E1: present
Tubular atrophy/interstitial fibrosis T0: 025%
T1: 2650%
T2: >50%

Adapted from Cattran DC, Coppo R, Cook HT, et al. The Oxford classification of IgA nephropathy:
rationale, clinicopathological correlations, and classification. Kidney Int. 2009;76(5):534545.

Return to contents
Questions 197 & 198 Q
Question 197
A 37-year-old Caucasian woman with a 12-year history of systemic lupus
erythematosus (SLE) but no previous history of renal disease presents with
new onset of haematuria and proteinuria. Laboratory evaluation reveals a
serum creatinine level of 120umol/l , 24-hour urine protein of 2.9 g, serum
albumin of 28 g/dl, white blood cell count of 2.4 109/L, hematocrit of 38.8%,
and platelet count of 122 109/L. Serologic evaluation reveals an anti-nuclear
antibody (ANA) titer of 1:1280, an anti-DNA antibody of 600 IU/ml, low C3 and
C4 complement levels, a positive IgG anticardiolipin antibody, and a negative
lupus anticoagulant.
Examination of the urine sediment discloses 6 to 10 red blood cells per high-
power field, no significant white blood cells, and no cellular casts. Previous
manifestations of SLE included pericarditis, alopecia, a malar rash, and
arthralgias. Physical examination reveals a BP of 122/76, an erythematous facial
rash, and no edema. Medications
include methylprednisolone 10 mg/day and hydroxychloroquine 200 mg/ day.
The kidneys measure 12.4 and 11.9 cm in length by ultrasound. Renal biopsy is
performed.
see the electron microscopy image
Which class of lupus nephritis does
the finding represent?

A WHO Class I
B WHO Class II
C WHO Class III
D WHO Class IV
E WHO class V

Question 198

Sodium removal in haemodialysis is achieved primarily by?


A. Diffusion
B. Convection
C. Convection and diffusion
D. Osmosis
E. None of above

Return to question 1 Return to contents


A nswer
197 - E
198 - C Convection and diffusion

Answer 197
The answer is E - WHO Class V, as the findings are of subepithelial
membranous deposits ( arrows). It is important to be familiar with basics of
WHO and RPS/ISN classifications of lupus nephritis.

Teaching notes- Kidney biopsy features of Lupus Nephritis


Light microscopy Immunofluorescence Electron Microscopy

Lupus Depends on WHO or ISN/ Full house Mesangial,


nephritis RPS class. Ranges from pattern. All subepithelial and
mesangialhypercellularity to immunofluorescence subendothelial
endocapillary proliferation stains including IgG, electron dense
with crescents and necrosis. IgA, IgM, C3 and deposits
Membranous lupus nephritis especially C1q are maybe present.
with will have thickened positive. Subendothelial
capillary loops with spikes. deposits indicate
Wire-loop deposits and activity. Subepithelial
hyaline capillary thrombi deposits indicate
indicate subendothelial membranous deposits.
deposits and active lesions.

Answer 198
Options A,B,D & E are incorrect
Option C is the correct answer
In a dialysis patient who makes no urine, sodium removal occurs through both
diffusion (15-20%) and convection (80%). Convective loss of sodium depends
on the prescribed UF, while diffusive clearance depends on the concentration
gradient between the plasma and dialysate. High dialysate sodium
prescriptions result in less disequilibrium symptoms, more vascular stability, and
lower incidence of muscle cramps. However, patients leave with a higher post
HD sodium concentration (relative hypernatraemia) and increased thirst, and
are prone to greater interdialytic weight gains and ultimately hypertension.

Return to contents
Questions 199 & 200 Q
Question 199
Which one of the following option shows most effective treatment for sexual
dysfunction management in haemodialysis and peritoneal dialysis patients?

A Optimise Dialysis
B Psychological counselling
C Correction of anaemia with ESA
D Use of zinc supplement
E Use of sildenafil

Question 200
Which one of the following drug safely use in dialysis patient for pain control?

A Morphine
B Hydromorphone
C Oxycodone
D Fentanyl
E Demerol

Return to question 1 Return to contents


A nswer 199 - E

Answer 199
Option E is the correct answer

Options A,B,C&D are incorrect. Erectile dysfunction (ED) is the persistent


inability to achieve or maintain an erection sufficient to permit satisfactory
sexual performance, and the resulting stress often impacts interaction with
others (1). End-stage renal failure patients on haemodialysis (HD) are frequently
affected by ED. The prevalence of ED in these patients has been estimated to
be between 71 and 82%.

There are several management guidelines for ED. The UK guidelines suggest
that a detailed history is the most important aspect in the assessment of
patients with ED. Treatment options for men with ED include psychosexual
therapy, drug therapy, transurethral or intracavernosal therapy, treatment with
vacuum-constriction devices, and surgical treatment .

Sildenafil, a selective inhibitor of phosphodiesterase type-5, which is the


predominant isozyme inactivating cyclic guanosine monophosphate (cGMP)
in the corpus cavernosum, has been shown to be an effective, well-tolerated
treatment for non-uremic men with ED.

Its use results in increased smooth muscle relaxation and improved erection
when nitric oxide is released in the presence of sexual stimulation.

Patients not to take sildenafil on dialysis days. There have been reports of
hypotension associated with sildenafil use by dialysis patients.

Oral sildenafil was an effective and safe treatment for ED in the present sample
of selected patients with chronic renal failure on HD.

Ref: Erectile dysfunction. N Engl J Med 342: 18021813, 2000

Return to contents
200 - D A
Answer 200
D is the correct answer.

Option A,B,C & E are incorrect. Pain is the most frequently reported symptom
in the dying patient with chronic kidney disease.

Pain remains under treated in many haemodialysis patients.


The World Health Organization pain ladder is an effective tool in the
management of pain in dialysis patients.
Causes:
Numerous but include uremic neuropathy, diabetic neuropathy, restless leg
syndrome, dialysis related amyloidosis, pain from underlying or comorbid
conditions, bone disease, etc.

Pain control in ESRD

Meperidine should not be used because its metabolite,


normeperidine, accumulates, leading to central nervous system
excitability and seizures.
Prolonged use of morphine in patients with chronic kidney disease
may lead to the accumulation of morphine-3-glucuronide and
morphine-6- glucuronide, which may contribute to the development
of myoclonic jerks.
The accumulation of morphine-6-glucuronidemay also lead to
prolonged narcosis in those with ESRD
For short term use (days), morphine is an excellent analgesic for
dying patients with end stage renal disease and is also useful in the
treatment of dyspnea

It is recommended that morphine and codeine be avoided in


dialysis patients; hydromorphone and oxycodone be used with
caution; fentanyl and methadone appear to be relatively safe to use.

Return to question 1 Return to contents


Radiology for SCE Exam

Images Index

1 Cartoon showing Course of PD Catheter in abdominal wall


2 Cartoon Correct Position of Tenchkoff Catheter
3 X-ray Abdomen - Normal Position of Tenchkoff catheter
4 Malposition of PD Catheter
5 Kink in PD Catheter
6 CAPD Catheter Entrapment
7 PD Dialysate Leak
8 Horse Shoe Kidney
9 Nephrocalcinosis
10 Enlarge Prostate
11 ADPKD (Autosomal Polycystic kidneys)
12 Bladder wall calcification - Schistosomiasis

Return to contents
Image 1
Drawing showing course of
Peritoneal dialysis
catheter in abdominal wall

Return to question 1 Return to contents


Image 2
Diagram shows the correct placement of a straight Tenckhoff
catheter for CAPD, with the tip pointing downward in the
pelvis.

Return to contents
Image 3
Plain Abdominal X-ray shows correct position
of Peritoneal Dialysis catheter, with the tip in
the Pelvis

Return to question 1 Return to contents


Image 4
Plain Abdomen X-ray shows
Malposition of PD catheter tip
in Right Flank

Return to contents
Image 5
Plain Abdominal x-ray shows kink in PD
Catheter

Return to question 1 Return to contents


Image 6
CT Abdomen showing entrapment of PD Catheter

Return to contents Return to contents


Image 8
IVP -Horse Shoe Kidney CT Abdomen- Horse shoe
Kidney

Return to question 1 Return to contents


Image 9
Plain Abdominal Xray: Nephrocalcinosis

Return to contents Return to contents


Image 10
Ultrasound Bladder: Showing Enlarge Prostate

Return to question 1 Return to contents


Image 11
Ultrasound Abdomen &
Below Image MRI Abdomen Showing ADPKD

Return to contents Return to contents


Image 12
CT Bladder Showing: Bladder wall calcification
secondary to Schistosomiasis

Return to question 1 Return to contents


Return to contents Return to contents
(A) A cholesterol crystal made up of plates heaped 1 upon another. Note
the clear-cut edges and corners. (B) Cystine crystals heaped 1 upon another.
Note the irregular hexagonal shape. (C) Birefringent star-like crystals of
ciprofloxacin. {D) Amoxici llin crystals appearing as needles, shocks of wheat,
and "broom bush," all strongly bi refringent under polarized light.
bultc rnlcs, ctc

Return to question 1 Return to contents


Upcoming book published by Tricorn books 2016

150 Renal Replacement Questions


For
Nephrology Specialty certificate Exam

By
Dr Ebadur Rahman
& Dr Raees Farhan Mushtaq

Return to contents Return to contents

Das könnte Ihnen auch gefallen